由买买提看人间百态

boards

本页内容为未名空间相应帖子的节选和存档,一周内的贴子最多显示50字,超过一周显示500字 访问原贴
Parenting版 - 数学教育 一家之言 番外篇
相关主题
分数应用题7,8岁的女孩子暑假在家都干什么呢?
代数小朋友不理解怎么办?鸡兔同笼其实非常简单
教绝对值方程:Khan Academy vs AMC10 答案 vs 分段函数替换?BSO: 儿子解射雕瑛姑九宫格
系统学习的反面是什么?混BBS要抱着贡献和学习的心
有些小学四年级的数学题不太会做老警察问个新问题
AMC 8 成绩 2016如何帮助有一定数学天赋的孩子
singapore math 到底怎么样啊?Q. & A. With Liping Ma
给初中学生家长的一些建议 (转载)什么时候给孩子引入方程概念
相关话题的讨论汇总
话题: 数学话题: 鸡兔同笼话题: amc话题: 孩子话题: 代数
进入Parenting版参与讨论
1 (共1页)
s**********y
发帖数: 509
1
Guess, Check, and Revise 是学好数学的核心
去年回国, 找了本地最富盛名的中学老师, 请教代数算数如何衔接。他的回答让我大
跌眼睛。猜想, 检验, 和修正。 他说他从不叫公式, 放手让小蛙自己试, 试到那
一步算哪一步。 我说这不漫无边际了? 他说自然要加以引导。 秉承一贯风格, 举例
如下:
鸡兔同笼问题。 鸡兔共20只, 脚64只, 鸡若干/兔若干?
鼓励小蛙先猜: 鸡10, 兔10. 脚60. 问题: 先加鸡, 还是先加兔? 答案: 加兔。
加几? 不知。 试试? 鸡:9, 兔:11. 脚62. 嘿:再加一就的。
鸡兔同笼问题。 鸡兔共20只, 脚70只, 鸡若干/兔若干?
还是鼓励小蛙先猜: 鸡10, 兔10. 脚60. 问题: 先加鸡, 还是先加兔? 答案: 加
兔。加几? 不知。 从上一个点试起。 鸡:8, 兔:12. 脚64. 再加一, 再加一。
慢慢也算到15.
鸡兔同笼问题。 鸡兔共20只, 脚46只, 鸡若干/兔若干?
还是重复。 慢慢小蛙发现:鸡加一, 脚减二, 兔加一, 脚加二。 这样只要猜一个
点, 算一下差异, 在除一下就好。
我听了后不觉惶然。 这有啥好处呢?
此位老师说, 我来告诉你禁赛题是如何出出来的。 大家都做鸡图同笼, 你看看如何
解一下问题:
鸡兔同笼问题。 鸡兔共20只, 脚65只, 鸡若干/兔若干?
鸡兔同笼问题。 鸡兔共20只, 脚82只, 鸡若干/兔若干?
鸡兔同笼问题。 鸡兔共20只, 脚36只, 鸡若干/兔若干?
凡事用心猜的, 一眼就知道答案。 凡事套公式的, 算半天兴许还算错了。
我这才明白, 怪不得当年大家挤破头去他的班上。
时间有限, 下次回国一定问问袋鼠怎么学!
r*g
发帖数: 3159
2
这是exactly我当时教娃的步骤。随便猜一个初始解,然后一对一对换,改进答案就行
。不然你要解释为何一开始假设全是鸡或者兔子,其实一开始假设什么都可以。

【在 s**********y 的大作中提到】
: Guess, Check, and Revise 是学好数学的核心
: 去年回国, 找了本地最富盛名的中学老师, 请教代数算数如何衔接。他的回答让我大
: 跌眼睛。猜想, 检验, 和修正。 他说他从不叫公式, 放手让小蛙自己试, 试到那
: 一步算哪一步。 我说这不漫无边际了? 他说自然要加以引导。 秉承一贯风格, 举例
: 如下:
: 鸡兔同笼问题。 鸡兔共20只, 脚64只, 鸡若干/兔若干?
: 鼓励小蛙先猜: 鸡10, 兔10. 脚60. 问题: 先加鸡, 还是先加兔? 答案: 加兔。
: 加几? 不知。 试试? 鸡:9, 兔:11. 脚62. 嘿:再加一就的。
: 鸡兔同笼问题。 鸡兔共20只, 脚70只, 鸡若干/兔若干?
: 还是鼓励小蛙先猜: 鸡10, 兔10. 脚60. 问题: 先加鸡, 还是先加兔? 答案: 加

h*****m
发帖数: 1034
3
说来有趣,不知道古人是怎么想的,为什么要把这类问题用鸡和兔子来表达。反正我刚
开始给我儿子出这个题目时,他觉得非常的奇怪,鸡和兔子头一点也不一样,脚也不一
样,为什么放一起来数?一点都不make sense。
后来我在网上搜了一下,找了一些有更有意义的表达方法。比如说,红铅笔4分钱一支
,蓝铅笔2分钱一支,一共20支铅笔64分钱,红蓝铅笔个多少支?
或者小明一小时吃4 个苹果,小红一小时吃2 个苹果,小明吃了一阵以后小红来接着吃
,他们一共吃了20个小时,共吃了64 个苹果,分别吃了多少个?

【在 s**********y 的大作中提到】
: Guess, Check, and Revise 是学好数学的核心
: 去年回国, 找了本地最富盛名的中学老师, 请教代数算数如何衔接。他的回答让我大
: 跌眼睛。猜想, 检验, 和修正。 他说他从不叫公式, 放手让小蛙自己试, 试到那
: 一步算哪一步。 我说这不漫无边际了? 他说自然要加以引导。 秉承一贯风格, 举例
: 如下:
: 鸡兔同笼问题。 鸡兔共20只, 脚64只, 鸡若干/兔若干?
: 鼓励小蛙先猜: 鸡10, 兔10. 脚60. 问题: 先加鸡, 还是先加兔? 答案: 加兔。
: 加几? 不知。 试试? 鸡:9, 兔:11. 脚62. 嘿:再加一就的。
: 鸡兔同笼问题。 鸡兔共20只, 脚70只, 鸡若干/兔若干?
: 还是鼓励小蛙先猜: 鸡10, 兔10. 脚60. 问题: 先加鸡, 还是先加兔? 答案: 加

w*********y
发帖数: 7895
4
这个需要小学高年级的娃做吧。。。感觉像小学竞赛题啊。

【在 h*****m 的大作中提到】
: 说来有趣,不知道古人是怎么想的,为什么要把这类问题用鸡和兔子来表达。反正我刚
: 开始给我儿子出这个题目时,他觉得非常的奇怪,鸡和兔子头一点也不一样,脚也不一
: 样,为什么放一起来数?一点都不make sense。
: 后来我在网上搜了一下,找了一些有更有意义的表达方法。比如说,红铅笔4分钱一支
: ,蓝铅笔2分钱一支,一共20支铅笔64分钱,红蓝铅笔个多少支?
: 或者小明一小时吃4 个苹果,小红一小时吃2 个苹果,小明吃了一阵以后小红来接着吃
: ,他们一共吃了20个小时,共吃了64 个苹果,分别吃了多少个?

s**********y
发帖数: 509
5
倒还真不是。 会做除法, 就可以了。 对小蛙, 用小一点的数字。 中间猜, 凑,最
后用除来拼的过程是一样的。

【在 w*********y 的大作中提到】
: 这个需要小学高年级的娃做吧。。。感觉像小学竞赛题啊。
c***x
发帖数: 1826
6

赞具体,赞分享。
“Guess, Check, and Revise 是学好数学的核心”
不知道是不是核心 -- 因为我数学学得不深,但是,其实可以是学好很多东西的重要一
步。
因为这个trial and error的过程不断的培养了intuition,虽然在一定阶段后,需要也
应该上代数,上方程,上公式,但是有了通过learning by thinking而发展出来的直觉
,一是帮助理解更高层次的概括和一般表达的来源;二是起到rule of thumb and
cross
-check的作用;三是长期培养,悟性高的孩子也许就自己导出一般规律来了,而这个归
纳的能力是认知上升的一个必要途径。
期待你下次展开讲讲代数的学习。

【在 s**********y 的大作中提到】
: Guess, Check, and Revise 是学好数学的核心
: 去年回国, 找了本地最富盛名的中学老师, 请教代数算数如何衔接。他的回答让我大
: 跌眼睛。猜想, 检验, 和修正。 他说他从不叫公式, 放手让小蛙自己试, 试到那
: 一步算哪一步。 我说这不漫无边际了? 他说自然要加以引导。 秉承一贯风格, 举例
: 如下:
: 鸡兔同笼问题。 鸡兔共20只, 脚64只, 鸡若干/兔若干?
: 鼓励小蛙先猜: 鸡10, 兔10. 脚60. 问题: 先加鸡, 还是先加兔? 答案: 加兔。
: 加几? 不知。 试试? 鸡:9, 兔:11. 脚62. 嘿:再加一就的。
: 鸡兔同笼问题。 鸡兔共20只, 脚70只, 鸡若干/兔若干?
: 还是鼓励小蛙先猜: 鸡10, 兔10. 脚60. 问题: 先加鸡, 还是先加兔? 答案: 加

c***x
发帖数: 1826
7
考考你,如果遇到下面这样的问题,怎么办?

为什么要把鸡和兔子放在一个笼子里?
为什么红铅笔比蓝铅笔贵呢?
为什么小明比小红吃得快呢?

【在 h*****m 的大作中提到】
: 说来有趣,不知道古人是怎么想的,为什么要把这类问题用鸡和兔子来表达。反正我刚
: 开始给我儿子出这个题目时,他觉得非常的奇怪,鸡和兔子头一点也不一样,脚也不一
: 样,为什么放一起来数?一点都不make sense。
: 后来我在网上搜了一下,找了一些有更有意义的表达方法。比如说,红铅笔4分钱一支
: ,蓝铅笔2分钱一支,一共20支铅笔64分钱,红蓝铅笔个多少支?
: 或者小明一小时吃4 个苹果,小红一小时吃2 个苹果,小明吃了一阵以后小红来接着吃
: ,他们一共吃了20个小时,共吃了64 个苹果,分别吃了多少个?

s**********y
发帖数: 509
8
小明和小红一共吃了10小时, 真能吃啊!

【在 c***x 的大作中提到】
: 考考你,如果遇到下面这样的问题,怎么办?
:
: 为什么要把鸡和兔子放在一个笼子里?
: 为什么红铅笔比蓝铅笔贵呢?
: 为什么小明比小红吃得快呢?

d**********h
发帖数: 2795
9
我娃小学三年级上学期学了类似方法:一个班男女共A人,男比女多B,问男女个多少?
当然娃的数学题里是“数字”,不是“代数”,因为没有方程概念。
解题过程就是猜,迭代,期待收敛
这就是典型的“呆仗”,考试一百分,肯定答对八十以上,其他二十或是不会或是没时
间完成了。很显然,这个方法的收敛速度是不可控的,在此能不能“以奇胜”就看水平
了(娃的水平和课外强化的水平)。
即使不能达到自我推导方程的聪慧,也要思考一下如何快速收敛,比如二分法,或者外
推内推估算。
打呆仗的一般都是偏向“缺乏想象力,害怕失败,好奇心不足,安于现状”,通常没有
明显短板,解决现实生活中考试中的百分之八十,九十的问题,剩下的就交给上帝了,
这样也不错。
真正惊才艳艳的是乔布斯,韩信这样的天才,大名鼎鼎的one more thing,当然一般不
指望
回到孩子,除了“以正合”之外,也要鼓励他们跳出来想,创新找取巧的办法。
回到这个数学题,就是要找规律,锻炼总结和抽象。迭代法培养的是潮水兄的人肉计算
器,抽象思维才是数学的节奏。
这个也是教育学习中的一个关键:实物和抽象如何交替前进

【在 s**********y 的大作中提到】
: Guess, Check, and Revise 是学好数学的核心
: 去年回国, 找了本地最富盛名的中学老师, 请教代数算数如何衔接。他的回答让我大
: 跌眼睛。猜想, 检验, 和修正。 他说他从不叫公式, 放手让小蛙自己试, 试到那
: 一步算哪一步。 我说这不漫无边际了? 他说自然要加以引导。 秉承一贯风格, 举例
: 如下:
: 鸡兔同笼问题。 鸡兔共20只, 脚64只, 鸡若干/兔若干?
: 鼓励小蛙先猜: 鸡10, 兔10. 脚60. 问题: 先加鸡, 还是先加兔? 答案: 加兔。
: 加几? 不知。 试试? 鸡:9, 兔:11. 脚62. 嘿:再加一就的。
: 鸡兔同笼问题。 鸡兔共20只, 脚70只, 鸡若干/兔若干?
: 还是鼓励小蛙先猜: 鸡10, 兔10. 脚60. 问题: 先加鸡, 还是先加兔? 答案: 加

c***x
发帖数: 1826
10

You made my day :-)
真的差点笑岔气!

【在 s**********y 的大作中提到】
: 小明和小红一共吃了10小时, 真能吃啊!
相关主题
AMC 8 成绩 20167,8岁的女孩子暑假在家都干什么呢?
singapore math 到底怎么样啊?鸡兔同笼其实非常简单
给初中学生家长的一些建议 (转载)BSO: 儿子解射雕瑛姑九宫格
进入Parenting版参与讨论
c***x
发帖数: 1826
11

说得中正平和,一如既往,不得不赞。
不过在版上air-lifting和抽象思维的一波波大潮下,来一点盘山公路和具体案例的孤
星,也不失为一种很好的平衡。

【在 d**********h 的大作中提到】
: 我娃小学三年级上学期学了类似方法:一个班男女共A人,男比女多B,问男女个多少?
: 当然娃的数学题里是“数字”,不是“代数”,因为没有方程概念。
: 解题过程就是猜,迭代,期待收敛
: 这就是典型的“呆仗”,考试一百分,肯定答对八十以上,其他二十或是不会或是没时
: 间完成了。很显然,这个方法的收敛速度是不可控的,在此能不能“以奇胜”就看水平
: 了(娃的水平和课外强化的水平)。
: 即使不能达到自我推导方程的聪慧,也要思考一下如何快速收敛,比如二分法,或者外
: 推内推估算。
: 打呆仗的一般都是偏向“缺乏想象力,害怕失败,好奇心不足,安于现状”,通常没有
: 明显短板,解决现实生活中考试中的百分之八十,九十的问题,剩下的就交给上帝了,

h*****m
发帖数: 1034
12

这就是我儿子的问题,我无法回答,所以才去找一些其它的例子。
因为红宝石比蓝宝石贵,红铅笔可以画红宝石,蓝铅笔只能画蓝宝石,所以红铅笔比蓝
铅笔贵。
至于红宝石为什么比蓝宝石贵,要去问女人们了。
因为小红在换牙,掉了一颗门牙,新的还没长出来。
话说,我前面这帖子主要是搞笑的,一点都不清真,你回它作甚?

【在 c***x 的大作中提到】
: 考考你,如果遇到下面这样的问题,怎么办?
:
: 为什么要把鸡和兔子放在一个笼子里?
: 为什么红铅笔比蓝铅笔贵呢?
: 为什么小明比小红吃得快呢?

h*****m
发帖数: 1034
13
呵呵,搞笑的。
可以换成工人A每天生产4个东西,过了些天以后工人B来接替,每天只能生产2个东西。
一共生产了20天,生产出了64个东西。
或者,披萨店师傅每小时做多少个披萨也行。

【在 s**********y 的大作中提到】
: 小明和小红一共吃了10小时, 真能吃啊!
c***x
发帖数: 1826
14

现实中,challenge the question的学生往往不少。
So you have to be well-prepared all the time...
与女人没有太大关系,与物以稀为贵关系大点。从审美角度看,我觉得蓝的更好看呢。
难道只许州官搞笑,不许百姓捧腹?

【在 h*****m 的大作中提到】
: 呵呵,搞笑的。
: 可以换成工人A每天生产4个东西,过了些天以后工人B来接替,每天只能生产2个东西。
: 一共生产了20天,生产出了64个东西。
: 或者,披萨店师傅每小时做多少个披萨也行。

d**********h
发帖数: 2795
15
抽象是以实物为前提的,这也是为什么一图抵万言,这也是为什么数学都是从数手指头
开始的
这个如何交替进步就是学问了,而且和娃娃的抽象思维能力有关,超前拖后估计都不成
楼主这个数学特例,小学老师教(我娃三年级美帝数学水平)还成,中学数学老师就有点
那个啥了。或许只是个例子吧

【在 c***x 的大作中提到】
:
: 现实中,challenge the question的学生往往不少。
: So you have to be well-prepared all the time...
: 与女人没有太大关系,与物以稀为贵关系大点。从审美角度看,我觉得蓝的更好看呢。
: 难道只许州官搞笑,不许百姓捧腹?

h*****m
发帖数: 1034
16

佩服佩服,总能上升到理论高度。。
倒也是,百姓不捧腹,州官搞笑也没有意思了。谢谢捧腹!

【在 c***x 的大作中提到】
:
: 现实中,challenge the question的学生往往不少。
: So you have to be well-prepared all the time...
: 与女人没有太大关系,与物以稀为贵关系大点。从审美角度看,我觉得蓝的更好看呢。
: 难道只许州官搞笑,不许百姓捧腹?

c***x
发帖数: 1826
17

潮水主要的担心是如果过早过多的强调了人肉计算器,会影响抽象思维能力,具体机制
参见,比如大脑灰质剪枝理论。
而我觉得这个楼的意义在于,guess,check,revise本身可以是提炼培养抽象思维的重
要一步。
而你说的意思是,最终还是要上升到抽象思维,如何从具体到抽象是认知学家的研究对
象,也是父母的学问和实践。

【在 d**********h 的大作中提到】
: 抽象是以实物为前提的,这也是为什么一图抵万言,这也是为什么数学都是从数手指头
: 开始的
: 这个如何交替进步就是学问了,而且和娃娃的抽象思维能力有关,超前拖后估计都不成
: 楼主这个数学特例,小学老师教(我娃三年级美帝数学水平)还成,中学数学老师就有点
: 那个啥了。或许只是个例子吧

c***x
发帖数: 1826
18

还是赶在滔滔潮水来临之前,赶紧开溜。
毕竟在灌水这件体力+脑力+斗嘴力的事情上,我自叹弗如~~~

【在 c***x 的大作中提到】
:
: 潮水主要的担心是如果过早过多的强调了人肉计算器,会影响抽象思维能力,具体机制
: 参见,比如大脑灰质剪枝理论。
: 而我觉得这个楼的意义在于,guess,check,revise本身可以是提炼培养抽象思维的重
: 要一步。
: 而你说的意思是,最终还是要上升到抽象思维,如何从具体到抽象是认知学家的研究对
: 象,也是父母的学问和实践。

f*c
发帖数: 687
19
以前有人讲过鸡兔问题的经典手筋:
命令所有的兔子都用两条腿站着,再数数地上少了几条腿。。。

【在 s**********y 的大作中提到】
: Guess, Check, and Revise 是学好数学的核心
: 去年回国, 找了本地最富盛名的中学老师, 请教代数算数如何衔接。他的回答让我大
: 跌眼睛。猜想, 检验, 和修正。 他说他从不叫公式, 放手让小蛙自己试, 试到那
: 一步算哪一步。 我说这不漫无边际了? 他说自然要加以引导。 秉承一贯风格, 举例
: 如下:
: 鸡兔同笼问题。 鸡兔共20只, 脚64只, 鸡若干/兔若干?
: 鼓励小蛙先猜: 鸡10, 兔10. 脚60. 问题: 先加鸡, 还是先加兔? 答案: 加兔。
: 加几? 不知。 试试? 鸡:9, 兔:11. 脚62. 嘿:再加一就的。
: 鸡兔同笼问题。 鸡兔共20只, 脚70只, 鸡若干/兔若干?
: 还是鼓励小蛙先猜: 鸡10, 兔10. 脚60. 问题: 先加鸡, 还是先加兔? 答案: 加

B********e
发帖数: 10014
20
故事的讲法是这样的:让鸡一屁股坐在地上

【在 f*c 的大作中提到】
: 以前有人讲过鸡兔问题的经典手筋:
: 命令所有的兔子都用两条腿站着,再数数地上少了几条腿。。。

相关主题
混BBS要抱着贡献和学习的心Q. & A. With Liping Ma
老警察问个新问题什么时候给孩子引入方程概念
如何帮助有一定数学天赋的孩子别鸡兔同笼了,来喝啤酒吧
进入Parenting版参与讨论
s**********y
发帖数: 509
21
一般来说, 诸如此类的技巧, 都是成人用代数的方法解出来后, 再用算数的方法来
加以解释。
如果小蛙自己能找到如此算法, 又能用如此语言表述, 应该是天赋异禀, 才华横溢
(或称之为天才)。
总的来说, 诸如此类的技巧既不能帮助学习下一阶段(袋鼠), 有无助于上一阶段
(算数)。 可以称之为鸡肋。

【在 B********e 的大作中提到】
: 故事的讲法是这样的:让鸡一屁股坐在地上
r*g
发帖数: 3159
22
没错。这本身就是抽象思维,比会列线性方程组还要普适。这是学用计算器,而不是自
己做人肉计算器。
相反,任何其他的小学解法假设全是这全是那,假设鸡兔缩腿什么的,那才是亲自做人
肉计算器,只有锻炼体力的价值。

【在 c***x 的大作中提到】
:
: 还是赶在滔滔潮水来临之前,赶紧开溜。
: 毕竟在灌水这件体力+脑力+斗嘴力的事情上,我自叹弗如~~~

w**d
发帖数: 2334
23
我现在是糊涂了,到底我们要教孩子什么?知识点,还是学习探索的方法?教到最后和
我们差不多?
w**d
发帖数: 2334
24
就lz这个例子,其实出发点是好的。但思路算不得很好。
这和我们生活中碰到新问题了一样,就是不知道解法的时候要先猜一猜,就是要先动动
手,别净胡想了。但乱猜也不好,要有个策略,具体到这个例子,明显先猜全是鸡,或
全是兔,或鸡兔一样多比较好。记得以前读书的时候(可能是高中),常有一堆找规律
的题,如果不能一眼就看出的话,可以先试试n=1,2,3的情况。其实这里的思路
比较类似:就是从特殊情况开始,先找找感觉 (说得不太准确)

【在 w**d 的大作中提到】
: 我现在是糊涂了,到底我们要教孩子什么?知识点,还是学习探索的方法?教到最后和
: 我们差不多?

B********e
发帖数: 10014
25
顺其自然
最重要是培养兴趣
其他都是瞎忙乎
跟我们一样还不够nb吗?哈

【在 w**d 的大作中提到】
: 我现在是糊涂了,到底我们要教孩子什么?知识点,还是学习探索的方法?教到最后和
: 我们差不多?

w**d
发帖数: 2334
26
兴趣怎么培养?娃不喜欢的话想方设法的绕到他喜欢?还是老美直接,就搞体育把身体
弄得棒棒的,数学暴烂也信心满盆,过得非常happy。

【在 B********e 的大作中提到】
: 顺其自然
: 最重要是培养兴趣
: 其他都是瞎忙乎
: 跟我们一样还不够nb吗?哈

d****g
发帖数: 7460
27
综合法和分析法。。
感觉娃两方面的能力都不是天生的,都需要成长.
t******l
发帖数: 10908
28
好了我来了。。。
首先从海空直降的战术上,这个问题并不是鸡兔同笼如何解,而是过于纠结于鸡兔同笼
问题本身可能是错误的。
或者说,这不是海空直降的如何直降的战术问题,而是你是不是把 101 空中骑兵师给
直降在敌人坦克群前方了,直接被屠了。。。或者直降在沼泽地里,就算人都就出来了
,武器基本都给淹了。。。或者干脆降在鸟不拉屎地、前不着村后不着店、呆着虽然不
会S,但也不知道干嘛。。。

【在 c***x 的大作中提到】
:
: 还是赶在滔滔潮水来临之前,赶紧开溜。
: 毕竟在灌水这件体力+脑力+斗嘴力的事情上,我自叹弗如~~~

t******l
发帖数: 10908
29
言归正传,就好比 place value 是海空伞降敌后游击队,其核心是 positional
notation system 及其背后的基本概念。。。过多纠结于十进制还是二进制还是三点八
进制的搞法,基本就是把游击队扔水塘里了。。。
而海空直降 algebra 1,其核心是 algebraic notation system, with its spatial
form of analytic geometric 及其背后的基本概念。。。而过多纠结 simultaneous (
linear) equation 这种特例问题,特别是鸡兔同笼这种特例里的特例问题。这种纯属
把 101 空中突击师一万多号人马,用直升机给统统机降在一个美帝小学的操场上。。
。对了,101 师长刚才发电报来问是不是要叠罗汉?。。。

【在 t******l 的大作中提到】
: 好了我来了。。。
: 首先从海空直降的战术上,这个问题并不是鸡兔同笼如何解,而是过于纠结于鸡兔同笼
: 问题本身可能是错误的。
: 或者说,这不是海空直降的如何直降的战术问题,而是你是不是把 101 空中骑兵师给
: 直降在敌人坦克群前方了,直接被屠了。。。或者直降在沼泽地里,就算人都就出来了
: ,武器基本都给淹了。。。或者干脆降在鸟不拉屎地、前不着村后不着店、呆着虽然不
: 会S,但也不知道干嘛。。。

t******l
发帖数: 10908
30
我个人对 "数学抽象" 的理解,是 space-time-pattern。。。之所以说是 “抽象”,
是因为其不依赖于特定的事物,而是反过来,某一类的事物都依赖于其背后的 “抽象
”。。。或者说,the fabric of cosmos 就是 “抽象” 在物理(mathematical
science)上的具体例子,在这个例子里,cosmos 是实物(虽然对很多人也算抽象了),
而 fabric 则是 cosmos 这个实物的 “数学抽象”。
所以从这个角度说,“数学抽象” 是能够用笔和纸在空间上大略的画出来的。(“大
略” 指通常可能有一定二义性,可能需要解释,当然猴子可能解释了也不明白)。。
。我个人认为,任何无法用笔和纸在空间上大略画出来的,都不是 “数学抽象”。。
。当然 “哲学抽象” 可能画不出大略,话说猴子文科里基本没啥东西能画出大略。。
。比如文科生的 “面朝大海,春暖花开”,画不出来的不一定懂海子,但画出来的一
定不懂海子就是了。当然懂不懂精神病人都不影响吃饭拉大号,其实无所谓。。。

【在 c***x 的大作中提到】
:
: 还是赶在滔滔潮水来临之前,赶紧开溜。
: 毕竟在灌水这件体力+脑力+斗嘴力的事情上,我自叹弗如~~~

相关主题
9岁, 如何报名考amc10代数小朋友不理解怎么办?
求科普: 奥数 vs. Math Olympiad vs. Math Count vs. Math circle vs. 超前学数学教绝对值方程:Khan Academy vs AMC10 答案 vs 分段函数替换?
分数应用题系统学习的反面是什么?
进入Parenting版参与讨论
d****g
发帖数: 7460
31
这就是我隔壁说的手段和目的。。。
手段:鸡兔同笼(靠凑)
目的:练习综合法,分析问题,找规律
手段:鸡兔同笼(靠让鸡趴下)
目的:练习高深的分析法,找数和量的关系。当然需要举一反三,各种练习。所以小学
各种算术题,如果都从头不记题型的分析,不比平面几何容易。。非常考脑子。
手段:鸡兔同笼(靠方程)
目的:练习抽像能力。练习算式变换。
当然什么年龄的娃有什么样的能力。不能拔苗助长。

(

【在 t******l 的大作中提到】
: 言归正传,就好比 place value 是海空伞降敌后游击队,其核心是 positional
: notation system 及其背后的基本概念。。。过多纠结于十进制还是二进制还是三点八
: 进制的搞法,基本就是把游击队扔水塘里了。。。
: 而海空直降 algebra 1,其核心是 algebraic notation system, with its spatial
: form of analytic geometric 及其背后的基本概念。。。而过多纠结 simultaneous (
: linear) equation 这种特例问题,特别是鸡兔同笼这种特例里的特例问题。这种纯属
: 把 101 空中突击师一万多号人马,用直升机给统统机降在一个美帝小学的操场上。。
: 。对了,101 师长刚才发电报来问是不是要叠罗汉?。。。

t******l
发帖数: 10908
32
我觉得手段应该是比如 “刷 MOEMS / AMC,不求甚解。每有会意,便欣然去打游戏”。
我的意思是,刷题不能太纠结于单个 element 的某一种特定的甚解,而是要把眼光放到
整个 set / graph 的 traversal 上。。。也就是所谓的 “快糙猛”。。。

【在 d****g 的大作中提到】
: 这就是我隔壁说的手段和目的。。。
: 手段:鸡兔同笼(靠凑)
: 目的:练习综合法,分析问题,找规律
: 手段:鸡兔同笼(靠让鸡趴下)
: 目的:练习高深的分析法,找数和量的关系。当然需要举一反三,各种练习。所以小学
: 各种算术题,如果都从头不记题型的分析,不比平面几何容易。。非常考脑子。
: 手段:鸡兔同笼(靠方程)
: 目的:练习抽像能力。练习算式变换。
: 当然什么年龄的娃有什么样的能力。不能拔苗助长。
:

d****g
发帖数: 7460
33
这个同意
主要是娃还太小。。
”。
放到
t******l
发帖数: 10908
34
娃还小就等等,不急。。。但我觉得 “快糙猛” 的理念,是首先要考虑规模优势。。
。就好比下面这个 video 里 101st 空中骑兵师的搞法(真实 video),是 UH-60 黑
鹰直升机搞成蝗虫的规模,放下来黑压压一片口径 105mm、射程 13km 的 M119 榴弹炮
。。。另外还直升机机降农用推土机?。。。特么我帝直升机机降轻装师的火力,比委员
长的五大王牌只强不弱,连民工都机械化,轰你没商量。。。而烤肉拉天窗扔水果刀的搞
法,再精装,我觉得规模不行是硬伤。。。
http://www.youtube.com/watch?v=a6uLBsCQDs4

【在 d****g 的大作中提到】
: 这个同意
: 主要是娃还太小。。
: ”。
: 放到

x***1
发帖数: 999
35
对。
就是假设,把这个假设学好,孩子数学就上一个台阶了。
代数是假设,几何更是一种假设。小孩开始学数的时候,脑子里那有数的概念,还不是
假设1就是一跟指头。还有一种叫反证法的,纯粹假设。有关小孩子学概率统计的,也
是假设。
假设全是鸡或兔,假设一个一半,假设一只鸡或兔,都是了不起的开始,这不就是孩子
的抽象思维在工作嘛。
不要假设,这题就太难了,因为“都是成人用代数的方法解出来后, 再用算数的方法
来加以解释”。



【在 s**********y 的大作中提到】
: 一般来说, 诸如此类的技巧, 都是成人用代数的方法解出来后, 再用算数的方法来
: 加以解释。
: 如果小蛙自己能找到如此算法, 又能用如此语言表述, 应该是天赋异禀, 才华横溢
: (或称之为天才)。
: 总的来说, 诸如此类的技巧既不能帮助学习下一阶段(袋鼠), 有无助于上一阶段
: (算数)。 可以称之为鸡肋。

s**********y
发帖数: 509
36
怪不得刚才手机一亮, 果然是潮水兄发帖了。
急学急用, 活学活用, 立竿见影 。 不见太阳怎么办?

【在 t******l 的大作中提到】
: 好了我来了。。。
: 首先从海空直降的战术上,这个问题并不是鸡兔同笼如何解,而是过于纠结于鸡兔同笼
: 问题本身可能是错误的。
: 或者说,这不是海空直降的如何直降的战术问题,而是你是不是把 101 空中骑兵师给
: 直降在敌人坦克群前方了,直接被屠了。。。或者直降在沼泽地里,就算人都就出来了
: ,武器基本都给淹了。。。或者干脆降在鸟不拉屎地、前不着村后不着店、呆着虽然不
: 会S,但也不知道干嘛。。。

w*********y
发帖数: 7895
37
我LG总结说他小学时候做的数学竞赛题对他数学很有帮助,因为都是用算术的方法来解
方程。。。就是你们讨论的这些题。。。



【在 s**********y 的大作中提到】
: 一般来说, 诸如此类的技巧, 都是成人用代数的方法解出来后, 再用算数的方法来
: 加以解释。
: 如果小蛙自己能找到如此算法, 又能用如此语言表述, 应该是天赋异禀, 才华横溢
: (或称之为天才)。
: 总的来说, 诸如此类的技巧既不能帮助学习下一阶段(袋鼠), 有无助于上一阶段
: (算数)。 可以称之为鸡肋。

x***1
发帖数: 999
38
你LG小时候咋做的,有啥心得

【在 w*********y 的大作中提到】
: 我LG总结说他小学时候做的数学竞赛题对他数学很有帮助,因为都是用算术的方法来解
: 方程。。。就是你们讨论的这些题。。。
:
: 溢

w*********y
发帖数: 7895
39
等我LG下班回家,问问他把。

【在 x***1 的大作中提到】
: 你LG小时候咋做的,有啥心得
s******3
发帖数: 344
40
re

【在 s**********y 的大作中提到】
: Guess, Check, and Revise 是学好数学的核心
: 去年回国, 找了本地最富盛名的中学老师, 请教代数算数如何衔接。他的回答让我大
: 跌眼睛。猜想, 检验, 和修正。 他说他从不叫公式, 放手让小蛙自己试, 试到那
: 一步算哪一步。 我说这不漫无边际了? 他说自然要加以引导。 秉承一贯风格, 举例
: 如下:
: 鸡兔同笼问题。 鸡兔共20只, 脚64只, 鸡若干/兔若干?
: 鼓励小蛙先猜: 鸡10, 兔10. 脚60. 问题: 先加鸡, 还是先加兔? 答案: 加兔。
: 加几? 不知。 试试? 鸡:9, 兔:11. 脚62. 嘿:再加一就的。
: 鸡兔同笼问题。 鸡兔共20只, 脚70只, 鸡若干/兔若干?
: 还是鼓励小蛙先猜: 鸡10, 兔10. 脚60. 问题: 先加鸡, 还是先加兔? 答案: 加

相关主题
系统学习的反面是什么?singapore math 到底怎么样啊?
有些小学四年级的数学题不太会做给初中学生家长的一些建议 (转载)
AMC 8 成绩 20167,8岁的女孩子暑假在家都干什么呢?
进入Parenting版参与讨论
t******l
发帖数: 10908
41
不见太阳也可以见彩虹嘛。。。开个玩笑。。。
认真的说,我虽然教了我娃的 pre-algebra,但我不确信是不是教对了。因为 pre-
algebra 教的好不好,本身无法真正被 quantify。一直要到 algebra 1 阶段,或者
AMC 10 阶段,才能知道 pre-algebra 教得好不好。
但我有一句话,不知道当说不当说。。。我想说的是,sell your idea to good math
students / kids, and leave choice to themselves。。。NEVER EVER sell your
idea to 推妈,in any time 。。。否则的话,every small mistake (if it is
mistake) you made, will receive enormous blames, to the level that you
really start to double yourself。。。其实我觉得这可能是 God 的 design:
People who cannot do proof wouldn't doubt their own opinion。。。
当然这也自洽就是了。

【在 s**********y 的大作中提到】
: 怪不得刚才手机一亮, 果然是潮水兄发帖了。
: 急学急用, 活学活用, 立竿见影 。 不见太阳怎么办?

t******l
发帖数: 10908
42
或者更直接一点,如果想赚钱,那就开一个不超过 Russian Math School 的 Center。
否则的话,forget about it。不管你的办法对不对,just don't even think of
change the world。
我整这些都是为了我娃,which is my responsibility。。。Anything beyond it,都
是去垃圾场的单程车票哈哈哈哈哈哈哈。。。

【在 s**********y 的大作中提到】
: 怪不得刚才手机一亮, 果然是潮水兄发帖了。
: 急学急用, 活学活用, 立竿见影 。 不见太阳怎么办?

t******l
发帖数: 10908
43
或者这么形象的说。。。要验证哥们着重长远的想法是不是靠谱,那至少也得等到 AMC
10 的时候,看蝗虫一样的 UH-60 黑鹰直升机,海空直降下来口径 105mm 的 M119 榴
弹炮,是不是把对手轰成稀巴烂的时候,才能见分晓。。。
但哥们我大胆预言,在此之前,很大可能哥们已经被烤肉拉天窗里飞出的,推妈之《攻
克鸡兔同笼之三十六计招招制敌》的短平快型水果刀们,给扎的体无完肤啦啦啦。。。
哥们多保重啊,我先溜号了。。。

【在 s**********y 的大作中提到】
: 怪不得刚才手机一亮, 果然是潮水兄发帖了。
: 急学急用, 活学活用, 立竿见影 。 不见太阳怎么办?

d**********h
发帖数: 2795
44
推妈的reputation这么差 :)
难怪家长纷纷表示自己不是推妈

math

【在 t******l 的大作中提到】
: 不见太阳也可以见彩虹嘛。。。开个玩笑。。。
: 认真的说,我虽然教了我娃的 pre-algebra,但我不确信是不是教对了。因为 pre-
: algebra 教的好不好,本身无法真正被 quantify。一直要到 algebra 1 阶段,或者
: AMC 10 阶段,才能知道 pre-algebra 教得好不好。
: 但我有一句话,不知道当说不当说。。。我想说的是,sell your idea to good math
: students / kids, and leave choice to themselves。。。NEVER EVER sell your
: idea to 推妈,in any time 。。。否则的话,every small mistake (if it is
: mistake) you made, will receive enormous blames, to the level that you
: really start to double yourself。。。其实我觉得这可能是 God 的 design:
: People who cannot do proof wouldn't doubt their own opinion。。。

s**********y
发帖数: 509
45
太认真了, 不过闲来聊聊。对人有点小益, 也就罢了。

math

【在 t******l 的大作中提到】
: 不见太阳也可以见彩虹嘛。。。开个玩笑。。。
: 认真的说,我虽然教了我娃的 pre-algebra,但我不确信是不是教对了。因为 pre-
: algebra 教的好不好,本身无法真正被 quantify。一直要到 algebra 1 阶段,或者
: AMC 10 阶段,才能知道 pre-algebra 教得好不好。
: 但我有一句话,不知道当说不当说。。。我想说的是,sell your idea to good math
: students / kids, and leave choice to themselves。。。NEVER EVER sell your
: idea to 推妈,in any time 。。。否则的话,every small mistake (if it is
: mistake) you made, will receive enormous blames, to the level that you
: really start to double yourself。。。其实我觉得这可能是 God 的 design:
: People who cannot do proof wouldn't doubt their own opinion。。。

s**********y
发帖数: 509
46
潮水兄是一朝被蛇咬, 十年有余悸。
我觉得退妈能把AMC10的题看懂就不错了。 断出不出来36招, 一般也就是3岁能弄个加
法, 5岁背99表, 出来显摆显摆, 再往上, 就心有余, 力不足了。

AMC

【在 t******l 的大作中提到】
: 或者这么形象的说。。。要验证哥们着重长远的想法是不是靠谱,那至少也得等到 AMC
: 10 的时候,看蝗虫一样的 UH-60 黑鹰直升机,海空直降下来口径 105mm 的 M119 榴
: 弹炮,是不是把对手轰成稀巴烂的时候,才能见分晓。。。
: 但哥们我大胆预言,在此之前,很大可能哥们已经被烤肉拉天窗里飞出的,推妈之《攻
: 克鸡兔同笼之三十六计招招制敌》的短平快型水果刀们,给扎的体无完肤啦啦啦。。。
: 哥们多保重啊,我先溜号了。。。

t******l
发帖数: 10908
47
其实别人觉得俺认真的时候,俺是在开玩笑。。。俺现在认真一下。。。
我是一个强调规模的人,所以我对娃教育的看法,也是基于海空立体战的看法。具体而
言,三支力量立体作战,具体到以下三个问题,这里拿四年级做例子
(1) Could you do school math with reasonable proficiency? (School Math). I
don't care small mistakes, but I do care your speed and power.
(2) Could you explore new/unknown math use any way you could use? (MOEMS) I
don't care your speed or try 100 dumb ways, or even you tired and decide to
try it tomorrow. I only care whether you are willing, and be able, to made
progress.
(3) Could you understand my presentation on MOEMS using my magician table?
which is set-theory-based spatial presentations/solutions. (Magician-Table
on MOEMS). I don't care whether you can do it yourself, or even how much you
can remember it tomorrow. As long as you can understand my presentation
more or less, and find it interesting, that is my call.

【在 s**********y 的大作中提到】
: 太认真了, 不过闲来聊聊。对人有点小益, 也就罢了。
:
: math

t******l
发帖数: 10908
48
在这个规模化立体战系统里,你所说的 “Guess, Check, and Revise”,在四年级是
,相当于我说的(2),也就是 “装甲矛头”,我觉得确实是课后数学加料最最重要的
部分。
但另一方面,(1)这种 “机械化步兵炮灰”,以及(3)这种 “101st 空中骑兵轻装
师敌后海空直降”,可能也要有机地结合起来。。。个人看法,不一定对。。。

progress.

【在 t******l 的大作中提到】
: 其实别人觉得俺认真的时候,俺是在开玩笑。。。俺现在认真一下。。。
: 我是一个强调规模的人,所以我对娃教育的看法,也是基于海空立体战的看法。具体而
: 言,三支力量立体作战,具体到以下三个问题,这里拿四年级做例子
: (1) Could you do school math with reasonable proficiency? (School Math). I
: don't care small mistakes, but I do care your speed and power.
: (2) Could you explore new/unknown math use any way you could use? (MOEMS) I
: don't care your speed or try 100 dumb ways, or even you tired and decide to
: try it tomorrow. I only care whether you are willing, and be able, to made
: progress.
: (3) Could you understand my presentation on MOEMS using my magician table?

s**********y
发帖数: 509
49
你这个1, 2, 3, 自然是好的。 斯大林也说过, 速度也是一种质量。3对一般小蛙
实行起来有困难, 要是环境许可, 也不失为一种方法。

【在 t******l 的大作中提到】
: 在这个规模化立体战系统里,你所说的 “Guess, Check, and Revise”,在四年级是
: ,相当于我说的(2),也就是 “装甲矛头”,我觉得确实是课后数学加料最最重要的
: 部分。
: 但另一方面,(1)这种 “机械化步兵炮灰”,以及(3)这种 “101st 空中骑兵轻装
: 师敌后海空直降”,可能也要有机地结合起来。。。个人看法,不一定对。。。
:
: progress.

c***x
发帖数: 1826
50

你既是老师又是家长,不可以糊涂。我的理解是按层次教:
1. teaching kids facts, concepts, processes, procedures and principles;
2. developing kids’ higher-order thinking skills in the cognitive domain--
applying, analyzing, evaluating, and creating;
3. promoting kids’ interest, motivation and aptitudes in the affective
domain.
教到最后不是和我们差不多,而是实现他们每个人在合理范围内可以实现的美好生活。
“The good life is one inspired by love and guided by knowledge.”
--Bertrand Russell
一个最有效的初始猜测似乎是假设鸡兔一样多,check总脚数如果太多,往鸡多兔少方
向revise,反之则反是。对于小学低年级而言,guess, check and revise其实是一个
很好的思维方式培养。将来解决很多更复杂的问题,不少也会用到类似的思路。

【在 w**d 的大作中提到】
: 我现在是糊涂了,到底我们要教孩子什么?知识点,还是学习探索的方法?教到最后和
: 我们差不多?

相关主题
鸡兔同笼其实非常简单老警察问个新问题
BSO: 儿子解射雕瑛姑九宫格如何帮助有一定数学天赋的孩子
混BBS要抱着贡献和学习的心Q. & A. With Liping Ma
进入Parenting版参与讨论
c***x
发帖数: 1826
51

昨天本来想和你打个赌:
如果潮水三天内跳这个楼,你送我一个包子;
如果潮水三天内不跳这个楼,我送你一个包子。
后来想,这种讹人包子的事情就不要干了 :-)

【在 s**********y 的大作中提到】
: 怪不得刚才手机一亮, 果然是潮水兄发帖了。
: 急学急用, 活学活用, 立竿见影 。 不见太阳怎么办?

c***x
发帖数: 1826
52

),
一直很赞你对"数学抽象" 的理解,是 space-time-pattern。
窃以为鸡兔同笼问题是space-time-pattern在小学低年级数学的极佳体现。

【在 t******l 的大作中提到】
: 我个人对 "数学抽象" 的理解,是 space-time-pattern。。。之所以说是 “抽象”,
: 是因为其不依赖于特定的事物,而是反过来,某一类的事物都依赖于其背后的 “抽象
: ”。。。或者说,the fabric of cosmos 就是 “抽象” 在物理(mathematical
: science)上的具体例子,在这个例子里,cosmos 是实物(虽然对很多人也算抽象了),
: 而 fabric 则是 cosmos 这个实物的 “数学抽象”。
: 所以从这个角度说,“数学抽象” 是能够用笔和纸在空间上大略的画出来的。(“大
: 略” 指通常可能有一定二义性,可能需要解释,当然猴子可能解释了也不明白)。。
: 。我个人认为,任何无法用笔和纸在空间上大略画出来的,都不是 “数学抽象”。。
: 。当然 “哲学抽象” 可能画不出大略,话说猴子文科里基本没啥东西能画出大略。。
: 。比如文科生的 “面朝大海,春暖花开”,画不出来的不一定懂海子,但画出来的一

z*****a
发帖数: 1214
53
太瞧不起我们推妈了,哼

【在 s**********y 的大作中提到】
: 潮水兄是一朝被蛇咬, 十年有余悸。
: 我觉得退妈能把AMC10的题看懂就不错了。 断出不出来36招, 一般也就是3岁能弄个加
: 法, 5岁背99表, 出来显摆显摆, 再往上, 就心有余, 力不足了。
:
: AMC

c***x
发帖数: 1826
54

我还在犹豫,我要不要带上推妈的帽子,来second this?
又被你一语说破了:)

【在 z*****a 的大作中提到】
: 太瞧不起我们推妈了,哼
t******l
发帖数: 10908
55
我个人觉得,幼儿园到小学数学里 space-time-pattern:
第一个里程碑是 understanding (naive) sets(比如 counting。。。)
第二个里程碑是 place value。实际上 The Hindu–Arabic numeral system --
positional notation system 是人类数学史上的一个里程碑。
第三个里程碑是 arithmetic properties,这个里程碑是从算术到代数的桥梁。。。因
为从算术到代数的跨越式的发展/外延这一跃,所有其它算术里的结论和 routine,在
代数里都有可能不适用。。。But arithmetic properties always stands. Because
if arithmetic properties fall, the whole algebra system fall. That is simply
because algebra system SHALL stand no matter what numbers (within the range
) are used to substitute the letters ...
后面里程碑的等你家娃大一点再说。。。// super fast run

【在 c***x 的大作中提到】
:
: 我还在犹豫,我要不要带上推妈的帽子,来second this?
: 又被你一语说破了:)

t******l
发帖数: 10908
56
在 arithmetic properties 之前,everything is intuitive,比如 commutative /
associative。
在 arithmetic properties 之后,e.g. the order of operation stands for
causality, which is timeline. And think of that, you can never / ever
violate the timeline, which is causality. Instead, you use commutative /
associative property to get an alternative yet equivalent new timeline, then
you walk on the new timeline and never look back the old timeline (don't
mix two different timeline / order-of-operation).

simply
range

【在 t******l 的大作中提到】
: 我个人觉得,幼儿园到小学数学里 space-time-pattern:
: 第一个里程碑是 understanding (naive) sets(比如 counting。。。)
: 第二个里程碑是 place value。实际上 The Hindu–Arabic numeral system --
: positional notation system 是人类数学史上的一个里程碑。
: 第三个里程碑是 arithmetic properties,这个里程碑是从算术到代数的桥梁。。。因
: 为从算术到代数的跨越式的发展/外延这一跃,所有其它算术里的结论和 routine,在
: 代数里都有可能不适用。。。But arithmetic properties always stands. Because
: if arithmetic properties fall, the whole algebra system fall. That is simply
: because algebra system SHALL stand no matter what numbers (within the range
: ) are used to substitute the letters ...

w**d
发帖数: 2334
57
罗素的话说的不错。以前在houston待过,每天听收音机里头walmart的广告,讲的是一
个道理:find something you love, and make a living out of it.
觉得糊涂是因为发现很多东西没法教,只能nurture。比如说
higher-order thinking skills 和 interest。这个前者也只能教一些,具体的举一反
三,推广应用就只能看孩子的造化。
具体到日常,我一般开始让孩子自己发挥做题,除非他问。完了再问他的做法 - 只对
那种有点小意思的题而言 - 然后再给他说我的做法。按道理这种方式应该不错,可能
是我儿子自尊心太强的缘故,发现这样居然对他有些打击。

【在 c***x 的大作中提到】
:
: 我还在犹豫,我要不要带上推妈的帽子,来second this?
: 又被你一语说破了:)

s**********y
发帖数: 509
58
这个要赞。
本来还有一段, 关于袋鼠算数衔接, 以后的空再写。

simply
range

【在 t******l 的大作中提到】
: 我个人觉得,幼儿园到小学数学里 space-time-pattern:
: 第一个里程碑是 understanding (naive) sets(比如 counting。。。)
: 第二个里程碑是 place value。实际上 The Hindu–Arabic numeral system --
: positional notation system 是人类数学史上的一个里程碑。
: 第三个里程碑是 arithmetic properties,这个里程碑是从算术到代数的桥梁。。。因
: 为从算术到代数的跨越式的发展/外延这一跃,所有其它算术里的结论和 routine,在
: 代数里都有可能不适用。。。But arithmetic properties always stands. Because
: if arithmetic properties fall, the whole algebra system fall. That is simply
: because algebra system SHALL stand no matter what numbers (within the range
: ) are used to substitute the letters ...

s**********y
发帖数: 509
59
退妈/爸总是想控制孩子。 支妈/爸(supporting mom/dad)支持孩子的成长。 中国
做过竞赛的就知道, 这玩意靠拼体力/刷题是不行的。中国人叫真明白, 美国人说
connection,更gifted是一个意思。 各位自然是真明白的支妈/爸。

【在 z*****a 的大作中提到】
: 太瞧不起我们推妈了,哼
d**********h
发帖数: 2795
60
袋鼠,还以为是鸡兔同笼的澳大利亚版本呢,现在明白了 :)

【在 s**********y 的大作中提到】
: 这个要赞。
: 本来还有一段, 关于袋鼠算数衔接, 以后的空再写。
:
: simply
: range

相关主题
什么时候给孩子引入方程概念求科普: 奥数 vs. Math Olympiad vs. Math Count vs. Math circle vs. 超前学数学
别鸡兔同笼了,来喝啤酒吧分数应用题
9岁, 如何报名考amc10代数小朋友不理解怎么办?
进入Parenting版参与讨论
s**********y
发帖数: 509
61
老师应该是 (1),(2)
家长应该是 (3), (2)
有是老师又是家长, 自然糊涂了。

【在 c***x 的大作中提到】
:
: 我还在犹豫,我要不要带上推妈的帽子,来second this?
: 又被你一语说破了:)

t******l
发帖数: 10908
62
属实


:老师应该是 (1),(2)
:家长应该是 (3),(2)
i**e
发帖数: 19242
63
哦,潮水太座原来属蛇的啊,哈哈
没关系,大胆承认,退妈我连AMC10的题目确实都看不懂
不过,这个并不妨碍我找能看得懂会教数学的人帮助娃提高数学能力,呵呵

【在 s**********y 的大作中提到】
: 潮水兄是一朝被蛇咬, 十年有余悸。
: 我觉得退妈能把AMC10的题看懂就不错了。 断出不出来36招, 一般也就是3岁能弄个加
: 法, 5岁背99表, 出来显摆显摆, 再往上, 就心有余, 力不足了。
:
: AMC

w*********y
发帖数: 7895
64
这个概括挺好的。。。有些词换换就更精确了。

【在 c***x 的大作中提到】
:
: 我还在犹豫,我要不要带上推妈的帽子,来second this?
: 又被你一语说破了:)

t******l
发帖数: 10908
65
属实。
很多时候军事学和军医学其实是等价的表述。
但另一方面,单单对于 “(3)101st 敌后海空直降” 而言,我个人更喜欢军事学的
表述,胜于军医学的表述。。。原因首先是军医学对(3)的研究本身很不够,导致军
医学以理论纲领为主,在实践上对敌后海空直降的具体 时间/空间/规模/距离/孤岛岛
链网络 的探讨非常不够,外加又被 8 billion 猴子 contaminate,导致根本不重视战
场时空,以至于弄出 spiral 这种名为海空直降,实际上基本是往敌后扔散兵游勇还不
带通讯器材的,活下来基本要看造化的,这种搞法。。。

【在 w*********y 的大作中提到】
: 这个概括挺好的。。。有些词换换就更精确了。
t******l
发帖数: 10908
66
:一个最有效的初始猜测似乎是假设鸡兔一样多,check总脚数如果太多,往鸡多兔少方
:向revise,反之则反是。对于小学低年级而言,guess, check and revise其实是一个
:很好的思维方式培养。将来解决很多更复杂的问题,不少也会用到类似的思路。
这个例子其实可以看到从 arithmetic 到 algebra 的 leap,如果用俺的在
arithmetic 和 algebra 当中的 set-theory-based deduction (Magician Table for
MOEMS),那最初假设常常是假设全部都是鸡更方便,因为这样把一只鸡 flip 成兔子,
多 2 条腿。于是一个除法解决问题(外加一个加法)。(当然选中点也不是不可以,
但是自找麻烦)。
这个跟 sum of arithmetic sequence,在 arithmetic 阶段的娃最初都是找中间点,
找左右 pair-wise 的 pattern。而到 algebra 阶段的娃就变成 make another copy,
reverse order,这样避免了 algebra 中找中点的不方便。
而发展到 sum of geometric sequence,基本上已经不会有人去用 arithmetic 的想法
去求解了。(geometric sequence 还有 sum to infinite 的,arithmetic 这下子
不能回家吃晚饭了。。。)。
这个问题是在于,algebra 里面开始用一个符号代表整个 set,而对于 analytic
geometric 里的 直线的 point-slope 概念上,初始 point 取在哪里问题都不大。在
这种情况下,algebra 里如果是取 homogeneous 比 heterogeneous 更方便,一般就取
homogeneous 的那些点,常常就是最左边或者最右边的点,而不是当中的那个点。
当然,我娃四年级的时候我不会跟娃啰嗦那么多,也就是 magician table 给她看看,
有个直观的感受。只 present,不解释,更不要求会玩。具体她到 algebra 之后她
自己会有自己的理解的,也不一定就跟俺的理解一样,毕竟她不是俺的 copy cat。

【在 c***x 的大作中提到】
:
: 我还在犹豫,我要不要带上推妈的帽子,来second this?
: 又被你一语说破了:)

t******l
发帖数: 10908
67
不仅仅是娃版连续数学,在娃版离散数学里的比如 combination w/ repetition 的
求解模型里面的那只著名的 ice-cream robot,也是从最最左边的那只 ice-cream jar
开始,蹦蹦跳跳地按照手里纸带上的大头符号 hop or scoope 的(这样多少种方式的
scoope ice-cream 的问题,转换成存在多少种不同的纸带的问题),而不会从当中那只
jar 那个开始蹦。。。话说从当中那个 jar 开始蹦也太坑爹了,搞不好没想明白给想
崩溃了。。。

少方
一个
for

【在 t******l 的大作中提到】
: :一个最有效的初始猜测似乎是假设鸡兔一样多,check总脚数如果太多,往鸡多兔少方
: :向revise,反之则反是。对于小学低年级而言,guess, check and revise其实是一个
: :很好的思维方式培养。将来解决很多更复杂的问题,不少也会用到类似的思路。
: 这个例子其实可以看到从 arithmetic 到 algebra 的 leap,如果用俺的在
: arithmetic 和 algebra 当中的 set-theory-based deduction (Magician Table for
: MOEMS),那最初假设常常是假设全部都是鸡更方便,因为这样把一只鸡 flip 成兔子,
: 多 2 条腿。于是一个除法解决问题(外加一个加法)。(当然选中点也不是不可以,
: 但是自找麻烦)。
: 这个跟 sum of arithmetic sequence,在 arithmetic 阶段的娃最初都是找中间点,
: 找左右 pair-wise 的 pattern。而到 algebra 阶段的娃就变成 make another copy,

x***1
发帖数: 999
68
说的入木三分。
即使天才的父母,也是请人教孩子,比如高斯,父母一看,这孩子还了得,赶紧请人吧
,别耽误了孩子。几年前看到本版syuan的孩子,我的第一反应就是赶紧请能人。
普通娃,如果想提高数学,更应该请人教。隔壁高楼的例子,还有月光微积分的发言。
琴棋书画,都在请人教,数学为啥不可以?学校里面没学好嘛,学校里的老师不会教嘛。
这些资本主义国家,其核心就是私,当然包括私教。这些国家的公,就是扯淡。私表现
在方方面面,如果不会私,就郁郁寡欢。私就是争取,呐喊,独立独行,毛遂自荐,自
己的事情自己考虑。
所以给孩子请个私教,一了百了。数学没什么了不起嘛。什么鸡啊兔啊的,别烦我,交
给那些会的人,不就几毛钱的事。

【在 i**e 的大作中提到】
: 哦,潮水太座原来属蛇的啊,哈哈
: 没关系,大胆承认,退妈我连AMC10的题目确实都看不懂
: 不过,这个并不妨碍我找能看得懂会教数学的人帮助娃提高数学能力,呵呵

t******l
发帖数: 10908
69
属实

嘛。

【在 x***1 的大作中提到】
: 说的入木三分。
: 即使天才的父母,也是请人教孩子,比如高斯,父母一看,这孩子还了得,赶紧请人吧
: ,别耽误了孩子。几年前看到本版syuan的孩子,我的第一反应就是赶紧请能人。
: 普通娃,如果想提高数学,更应该请人教。隔壁高楼的例子,还有月光微积分的发言。
: 琴棋书画,都在请人教,数学为啥不可以?学校里面没学好嘛,学校里的老师不会教嘛。
: 这些资本主义国家,其核心就是私,当然包括私教。这些国家的公,就是扯淡。私表现
: 在方方面面,如果不会私,就郁郁寡欢。私就是争取,呐喊,独立独行,毛遂自荐,自
: 己的事情自己考虑。
: 所以给孩子请个私教,一了百了。数学没什么了不起嘛。什么鸡啊兔啊的,别烦我,交
: 给那些会的人,不就几毛钱的事。

i**e
发帖数: 19242
70
呵呵
只是有点看不过,闲来调侃了几句,就入木了:)
你跟着闹啊:)

嘛。

【在 x***1 的大作中提到】
: 说的入木三分。
: 即使天才的父母,也是请人教孩子,比如高斯,父母一看,这孩子还了得,赶紧请人吧
: ,别耽误了孩子。几年前看到本版syuan的孩子,我的第一反应就是赶紧请能人。
: 普通娃,如果想提高数学,更应该请人教。隔壁高楼的例子,还有月光微积分的发言。
: 琴棋书画,都在请人教,数学为啥不可以?学校里面没学好嘛,学校里的老师不会教嘛。
: 这些资本主义国家,其核心就是私,当然包括私教。这些国家的公,就是扯淡。私表现
: 在方方面面,如果不会私,就郁郁寡欢。私就是争取,呐喊,独立独行,毛遂自荐,自
: 己的事情自己考虑。
: 所以给孩子请个私教,一了百了。数学没什么了不起嘛。什么鸡啊兔啊的,别烦我,交
: 给那些会的人,不就几毛钱的事。

相关主题
代数小朋友不理解怎么办?有些小学四年级的数学题不太会做
教绝对值方程:Khan Academy vs AMC10 答案 vs 分段函数替换?AMC 8 成绩 2016
系统学习的反面是什么?singapore math 到底怎么样啊?
进入Parenting版参与讨论
s***n
发帖数: 1280
71
(3)(2)(1)(2)(3)(2)(1)(2)(3)(2)(1)....

【在 s**********y 的大作中提到】
: 老师应该是 (1),(2)
: 家长应该是 (3), (2)
: 有是老师又是家长, 自然糊涂了。

d**********h
发帖数: 2795
72
私教可以教技术,教思路,教道理,但是教不了热情,教不了勇气,教不了爱
如果私教这些都交了,那么父母就可以退出孩子的生活了
这不是说梦,过去老农民把自己娃送到老师家里,读书的是学生,做手艺的是学徒,由
此有了师徒胜父子的关系,血缘父子还有啥剩下了?
烦恼就在于,一生下娃来就别想省事,这是前辈子欠的 :)
想在任何地方偷懒,都会在不久的将来发现这个地方是个窟窿 :)

嘛。

【在 x***1 的大作中提到】
: 说的入木三分。
: 即使天才的父母,也是请人教孩子,比如高斯,父母一看,这孩子还了得,赶紧请人吧
: ,别耽误了孩子。几年前看到本版syuan的孩子,我的第一反应就是赶紧请能人。
: 普通娃,如果想提高数学,更应该请人教。隔壁高楼的例子,还有月光微积分的发言。
: 琴棋书画,都在请人教,数学为啥不可以?学校里面没学好嘛,学校里的老师不会教嘛。
: 这些资本主义国家,其核心就是私,当然包括私教。这些国家的公,就是扯淡。私表现
: 在方方面面,如果不会私,就郁郁寡欢。私就是争取,呐喊,独立独行,毛遂自荐,自
: 己的事情自己考虑。
: 所以给孩子请个私教,一了百了。数学没什么了不起嘛。什么鸡啊兔啊的,别烦我,交
: 给那些会的人,不就几毛钱的事。

d****g
发帖数: 7460
73
话说我娃是这么做鸡兔同笼的:
鸡兔 20只, 脚 64。 应该用假设法,假设什么呢?假设---------- 所有的脚都是鸡
,应有32个头。.....
d****g
发帖数: 7460
74
如果同样编一个象鸡屁股坐地的典故,就是假设脚都是鸡,然后把两个鸡头使劲砸在一
起,砸出来一个兔头,鸡就变兔子了。

【在 d****g 的大作中提到】
: 话说我娃是这么做鸡兔同笼的:
: 鸡兔 20只, 脚 64。 应该用假设法,假设什么呢?假设---------- 所有的脚都是鸡
: ,应有32个头。.....

t******l
发帖数: 10908
75
属实

【在 d**********h 的大作中提到】
: 私教可以教技术,教思路,教道理,但是教不了热情,教不了勇气,教不了爱
: 如果私教这些都交了,那么父母就可以退出孩子的生活了
: 这不是说梦,过去老农民把自己娃送到老师家里,读书的是学生,做手艺的是学徒,由
: 此有了师徒胜父子的关系,血缘父子还有啥剩下了?
: 烦恼就在于,一生下娃来就别想省事,这是前辈子欠的 :)
: 想在任何地方偷懒,都会在不久的将来发现这个地方是个窟窿 :)
:
: 嘛。

h*****m
发帖数: 1034
76
你们知识都忒丰富,理论都忒高深。我还是聊聊题目本身吧。
话说,某人去见大师说:“我教我孩子鸡兔同笼问题,他总是喜欢猜出来答案,我想教
他先假设都是鸡,然后替换一只成兔子增加两支脚的办法来算,他总是不听,怎么办?“
大师从身旁拿起一串念珠,说:“老衲这串珠子,是两位施主所赠的168颗珠子串成,第
一位施主所赠珠子每颗重10克,第二位施主所赠珠子每颗重14克,整串珠子一共2000克
重。你且猜猜看每种珠子多少个?“。某人呆立半晌,说:“我知道了,这岂不就是潮
水智人所言,用具体的题目来penalize某种解法,和鼓励另一种解法吗?“。大师笑曰
:“数目太大,他猜不出来,自然就要按你的办法来算了。再不行,你给他整几万的数
字来出题“
某人大喜,回家依计而行。唤儿过来,出一巨大数目问题,料儿难以猜出。儿呆立半晌
,怒曰:“你莫不是在消遣洒家?“。一把把题目扔在地上,跑走玩儿去了。某人吐血
,倒地之前喃喃道:“你不按常理出牌!”
t******l
发帖数: 10908
77
大师从袖子里拿出这张泛黄的古旧地图:

?“
,第

【在 h*****m 的大作中提到】
: 你们知识都忒丰富,理论都忒高深。我还是聊聊题目本身吧。
: 话说,某人去见大师说:“我教我孩子鸡兔同笼问题,他总是喜欢猜出来答案,我想教
: 他先假设都是鸡,然后替换一只成兔子增加两支脚的办法来算,他总是不听,怎么办?“
: 大师从身旁拿起一串念珠,说:“老衲这串珠子,是两位施主所赠的168颗珠子串成,第
: 一位施主所赠珠子每颗重10克,第二位施主所赠珠子每颗重14克,整串珠子一共2000克
: 重。你且猜猜看每种珠子多少个?“。某人呆立半晌,说:“我知道了,这岂不就是潮
: 水智人所言,用具体的题目来penalize某种解法,和鼓励另一种解法吗?“。大师笑曰
: :“数目太大,他猜不出来,自然就要按你的办法来算了。再不行,你给他整几万的数
: 字来出题“
: 某人大喜,回家依计而行。唤儿过来,出一巨大数目问题,料儿难以猜出。儿呆立半晌

t******l
发帖数: 10908
78
大师另附一 Amazon 网购蓝光 HDDVD 的 web 链接:

?“
,第

【在 h*****m 的大作中提到】
: 你们知识都忒丰富,理论都忒高深。我还是聊聊题目本身吧。
: 话说,某人去见大师说:“我教我孩子鸡兔同笼问题,他总是喜欢猜出来答案,我想教
: 他先假设都是鸡,然后替换一只成兔子增加两支脚的办法来算,他总是不听,怎么办?“
: 大师从身旁拿起一串念珠,说:“老衲这串珠子,是两位施主所赠的168颗珠子串成,第
: 一位施主所赠珠子每颗重10克,第二位施主所赠珠子每颗重14克,整串珠子一共2000克
: 重。你且猜猜看每种珠子多少个?“。某人呆立半晌,说:“我知道了,这岂不就是潮
: 水智人所言,用具体的题目来penalize某种解法,和鼓励另一种解法吗?“。大师笑曰
: :“数目太大,他猜不出来,自然就要按你的办法来算了。再不行,你给他整几万的数
: 字来出题“
: 某人大喜,回家依计而行。唤儿过来,出一巨大数目问题,料儿难以猜出。儿呆立半晌

d**********h
发帖数: 2795
79
这就是穷则图变
力穷之时,就是智力出场之日
没有计算机之前大家还不活了,数学家不算题了,工程师不建房子,统帅不打仗,老财
不收租了 :)

?“
,第

【在 h*****m 的大作中提到】
: 你们知识都忒丰富,理论都忒高深。我还是聊聊题目本身吧。
: 话说,某人去见大师说:“我教我孩子鸡兔同笼问题,他总是喜欢猜出来答案,我想教
: 他先假设都是鸡,然后替换一只成兔子增加两支脚的办法来算,他总是不听,怎么办?“
: 大师从身旁拿起一串念珠,说:“老衲这串珠子,是两位施主所赠的168颗珠子串成,第
: 一位施主所赠珠子每颗重10克,第二位施主所赠珠子每颗重14克,整串珠子一共2000克
: 重。你且猜猜看每种珠子多少个?“。某人呆立半晌,说:“我知道了,这岂不就是潮
: 水智人所言,用具体的题目来penalize某种解法,和鼓励另一种解法吗?“。大师笑曰
: :“数目太大,他猜不出来,自然就要按你的办法来算了。再不行,你给他整几万的数
: 字来出题“
: 某人大喜,回家依计而行。唤儿过来,出一巨大数目问题,料儿难以猜出。儿呆立半晌

t******l
发帖数: 10908
80
属实。。。另外也可以是 “懒则图变”。。。当然硬币的另一面嘛。。。古人有问:
“勤奋和愚蠢是不是一对天生的孪生兄弟?” // super fast run

【在 d**********h 的大作中提到】
: 这就是穷则图变
: 力穷之时,就是智力出场之日
: 没有计算机之前大家还不活了,数学家不算题了,工程师不建房子,统帅不打仗,老财
: 不收租了 :)
:
: ?“
: ,第

相关主题
给初中学生家长的一些建议 (转载)BSO: 儿子解射雕瑛姑九宫格
7,8岁的女孩子暑假在家都干什么呢?混BBS要抱着贡献和学习的心
鸡兔同笼其实非常简单老警察问个新问题
进入Parenting版参与讨论
i**e
发帖数: 19242
81
因为父母最懂自己的孩子,所以父母可以是最好的老师
又因为父母对孩子有情感上的投入,所以父母也可能是最情绪化也就是最糟糕的老师
放心
你说的这些东西,tutor教不来的,底子是在家里父母给打的:)

私教可以教技术,教思路,教道理,但是教不了热情,教不了勇气,教不了爱

【在 d**********h 的大作中提到】
: 私教可以教技术,教思路,教道理,但是教不了热情,教不了勇气,教不了爱
: 如果私教这些都交了,那么父母就可以退出孩子的生活了
: 这不是说梦,过去老农民把自己娃送到老师家里,读书的是学生,做手艺的是学徒,由
: 此有了师徒胜父子的关系,血缘父子还有啥剩下了?
: 烦恼就在于,一生下娃来就别想省事,这是前辈子欠的 :)
: 想在任何地方偷懒,都会在不久的将来发现这个地方是个窟窿 :)
:
: 嘛。

i**e
发帖数: 19242
82
实在是受不了了:)
孩子第一次接触鸡兔同笼的时候
猜之,然后修正,最后因为计算不过关,等于我帮着算出来的
第二次再见 //第一次的基本忘光了
又猜之,运气太好,直接猜准
我只好腹议之后,把一肚子的算术之话全部憋回去了:)
第三次再见
依旧猜之,修正之,终于算正确了
看她算得那么辛苦,我就说了一句
你猜几只,换来换去还要加加减减,如果直接猜全部/零只
换过去几只就是几只,还不容易出错,是不是?
现在娃会咋做,我也吃不准,呵呵
别人兜售过吹哨法
我没用,万一不是鸡兔,来三头六臂的妖魔鬼怪,吹哨人家不听呢?呵呵
潮大师讲的东西,对我来说基本上是天书
我的rule of thumb
是观察自己的孩子是如何解决问题的
跟着孩子的思维走,guide,不要硬性灌输
top down的东西不牢靠
自己想出来走出来的才牢固
有的时候,老师父母指点一下,总结一下规律优化一下算法就可以了
这些是算术
我个人觉着是beauty of problem solving, 当然是初级的
所以我个人不主张过早让孩子接触代数
多玩玩这种problem solving多好

?“
,第

【在 h*****m 的大作中提到】
: 你们知识都忒丰富,理论都忒高深。我还是聊聊题目本身吧。
: 话说,某人去见大师说:“我教我孩子鸡兔同笼问题,他总是喜欢猜出来答案,我想教
: 他先假设都是鸡,然后替换一只成兔子增加两支脚的办法来算,他总是不听,怎么办?“
: 大师从身旁拿起一串念珠,说:“老衲这串珠子,是两位施主所赠的168颗珠子串成,第
: 一位施主所赠珠子每颗重10克,第二位施主所赠珠子每颗重14克,整串珠子一共2000克
: 重。你且猜猜看每种珠子多少个?“。某人呆立半晌,说:“我知道了,这岂不就是潮
: 水智人所言,用具体的题目来penalize某种解法,和鼓励另一种解法吗?“。大师笑曰
: :“数目太大,他猜不出来,自然就要按你的办法来算了。再不行,你给他整几万的数
: 字来出题“
: 某人大喜,回家依计而行。唤儿过来,出一巨大数目问题,料儿难以猜出。儿呆立半晌

t******l
发帖数: 10908
83
说实话我从来没有给娃去整过鸡兔同笼问题,都是版上讨论很热烈所以我参合一脚。
原因是我从来没有认为鸡兔同笼是 K-12 数学教育任何一阶段的重点。。。古人云,海
空直降抢滩得抢战略要地不是?。。。当然土豪派 USMC 去津巴布韦抢个厕所也不是不
行。。。

:实在是受不了了:)
i**e
发帖数: 19242
84
呵呵
对,你都去整amc8 AMC10了:)
新加坡数学里的题
我娃的3次遭遇,也是跨过了3,4年的光阴
我个人认为见识见识挺好的,没啥坏处
当然,我从来不强求孩子在什么年龄一定要会什么罢了
所以,如果问我现在孩子会怎么解这种题,我还真不知道
数学的本质是reasoning/thinking,要用心精心思考要悟
不是每个孩子很快能悟出来的
可惜米国这边练习太少,少了很多悟的机会

【在 t******l 的大作中提到】
: 说实话我从来没有给娃去整过鸡兔同笼问题,都是版上讨论很热烈所以我参合一脚。
: 原因是我从来没有认为鸡兔同笼是 K-12 数学教育任何一阶段的重点。。。古人云,海
: 空直降抢滩得抢战略要地不是?。。。当然土豪派 USMC 去津巴布韦抢个厕所也不是不
: 行。。。
:
: :实在是受不了了:)
: :

d**********h
发帖数: 2795
85
明白了,
纸上谈兵算鸡兔
云遮雾罩抢滩涂
潮水旧浪换新浪
不喜呆仗怨推娘

【在 t******l 的大作中提到】
: 说实话我从来没有给娃去整过鸡兔同笼问题,都是版上讨论很热烈所以我参合一脚。
: 原因是我从来没有认为鸡兔同笼是 K-12 数学教育任何一阶段的重点。。。古人云,海
: 空直降抢滩得抢战略要地不是?。。。当然土豪派 USMC 去津巴布韦抢个厕所也不是不
: 行。。。
:
: :实在是受不了了:)
: :

r*g
发帖数: 3159
86
Re. 就一普通三年级题目。因为中国老祖宗碰巧会做,才出了名。

【在 t******l 的大作中提到】
: 说实话我从来没有给娃去整过鸡兔同笼问题,都是版上讨论很热烈所以我参合一脚。
: 原因是我从来没有认为鸡兔同笼是 K-12 数学教育任何一阶段的重点。。。古人云,海
: 空直降抢滩得抢战略要地不是?。。。当然土豪派 USMC 去津巴布韦抢个厕所也不是不
: 行。。。
:
: :实在是受不了了:)
: :

t******l
发帖数: 10908
87
我不是说鸡兔同笼代数解法对小学生简单,而是说鸡兔同笼/追击问题的代数解法本来
不属于小学问题,没有必要硬塞在小学阶段拔苗助长。
这种本来就是学会 二元一次方程 / 运动学 以后自动就会了的东西,如果有放在小学
阶段,那就是玩玩兴趣的意思。。。战术上就算搞得再成功,把有趣变成无趣就是战略
上的失败。。。就好比为了砍一棵树,把林子给烧了,这咋看都不值。。。话糙理不糙
。。。
或者再退一步说,就算鸡兔同笼 / 追击问题 统统不会,那 AMC10 拿个平均分丝毫不
会受影响(AMC 10 前几题太简单)。。。所以等到 AMC 10 能拿到平均分以后,再搞
鸡兔同笼,那也不至于输在起跑线上不是?。。。

:Re. 就一普通三年级题目。因为中国老祖宗碰巧会做,才出了名。
C*****d
发帖数: 2253
88
AMC10几分算是高分?

【在 t******l 的大作中提到】
: 我不是说鸡兔同笼代数解法对小学生简单,而是说鸡兔同笼/追击问题的代数解法本来
: 不属于小学问题,没有必要硬塞在小学阶段拔苗助长。
: 这种本来就是学会 二元一次方程 / 运动学 以后自动就会了的东西,如果有放在小学
: 阶段,那就是玩玩兴趣的意思。。。战术上就算搞得再成功,把有趣变成无趣就是战略
: 上的失败。。。就好比为了砍一棵树,把林子给烧了,这咋看都不值。。。话糙理不糙
: 。。。
: 或者再退一步说,就算鸡兔同笼 / 追击问题 统统不会,那 AMC10 拿个平均分丝毫不
: 会受影响(AMC 10 前几题太简单)。。。所以等到 AMC 10 能拿到平均分以后,再搞
: 鸡兔同笼,那也不至于输在起跑线上不是?。。。
:

t******l
发帖数: 10908
89
我觉得 AMC 10 拿 100 分算入门砖,可以去竞赛组当炮灰。90 分以下我觉得还不算竞
赛入门。
当然我娃离开入门还很远,走一步看一步,无所谓,只要不是投机取巧搞过度烤肉拉天
窗水果刀,就算竞赛不入门,学到的数学知识能力也赚翻了。

:AMC10几分算是高分?
:【 在 timefall (时光崩塌) 的大作中提到: 】
s**********y
发帖数: 509
90
承认自己不足, 替孩子找老师, 是好事哈。

【在 i**e 的大作中提到】
: 哦,潮水太座原来属蛇的啊,哈哈
: 没关系,大胆承认,退妈我连AMC10的题目确实都看不懂
: 不过,这个并不妨碍我找能看得懂会教数学的人帮助娃提高数学能力,呵呵

相关主题
如何帮助有一定数学天赋的孩子别鸡兔同笼了,来喝啤酒吧
Q. & A. With Liping Ma9岁, 如何报名考amc10
什么时候给孩子引入方程概念求科普: 奥数 vs. Math Olympiad vs. Math Count vs. Math circle vs. 超前学数学
进入Parenting版参与讨论
s**********y
发帖数: 509
91
你这明显小瞧了娃凑的能力。
168 颗珠子, 假设都是10克, 总重1680克。 2000-1680=320, 320/4=80
答案: 80 14克珠子, 88颗10克珠子。 一般蛙新蒜即可。

?“
,第

【在 h*****m 的大作中提到】
: 你们知识都忒丰富,理论都忒高深。我还是聊聊题目本身吧。
: 话说,某人去见大师说:“我教我孩子鸡兔同笼问题,他总是喜欢猜出来答案,我想教
: 他先假设都是鸡,然后替换一只成兔子增加两支脚的办法来算,他总是不听,怎么办?“
: 大师从身旁拿起一串念珠,说:“老衲这串珠子,是两位施主所赠的168颗珠子串成,第
: 一位施主所赠珠子每颗重10克,第二位施主所赠珠子每颗重14克,整串珠子一共2000克
: 重。你且猜猜看每种珠子多少个?“。某人呆立半晌,说:“我知道了,这岂不就是潮
: 水智人所言,用具体的题目来penalize某种解法,和鼓励另一种解法吗?“。大师笑曰
: :“数目太大,他猜不出来,自然就要按你的办法来算了。再不行,你给他整几万的数
: 字来出题“
: 某人大喜,回家依计而行。唤儿过来,出一巨大数目问题,料儿难以猜出。儿呆立半晌

t******l
发帖数: 10908
92
属实


:你这明显小瞧了娃凑的能力。
c***x
发帖数: 1826
93

simply
range
替我家小娃谢谢潮水叔叔,她显然还在第一个里程碑里折腾。
希望这个网站不要太快倒闭 :-)

【在 t******l 的大作中提到】
: 我个人觉得,幼儿园到小学数学里 space-time-pattern:
: 第一个里程碑是 understanding (naive) sets(比如 counting。。。)
: 第二个里程碑是 place value。实际上 The Hindu–Arabic numeral system --
: positional notation system 是人类数学史上的一个里程碑。
: 第三个里程碑是 arithmetic properties,这个里程碑是从算术到代数的桥梁。。。因
: 为从算术到代数的跨越式的发展/外延这一跃,所有其它算术里的结论和 routine,在
: 代数里都有可能不适用。。。But arithmetic properties always stands. Because
: if arithmetic properties fall, the whole algebra system fall. That is simply
: because algebra system SHALL stand no matter what numbers (within the range
: ) are used to substitute the letters ...

c***x
发帖数: 1826
94

自尊心强是个很好的属性,长久看,比会用高明的方法解题更重要吧。
话说回来,我们自己做或者教某一个方向的,必须要警惕试图把这种专业偏好和要求影
射在孩子身上。
一个是因为mean-reverting,毕竟我们已经靠某个专业拿支票了。所以,我们认为司空
见惯理所当然的东西,也许在平均意义上而言,并不那么显然。
另一个是很可能孩子们的兴趣和擅长都与我们不同。他们完全可能在他们喜欢和擅长的
领域做得比我们好的多,开心得多。
不是针对你说的,是我自己的自省,和你共勉吧。

【在 w**d 的大作中提到】
: 罗素的话说的不错。以前在houston待过,每天听收音机里头walmart的广告,讲的是一
: 个道理:find something you love, and make a living out of it.
: 觉得糊涂是因为发现很多东西没法教,只能nurture。比如说
: higher-order thinking skills 和 interest。这个前者也只能教一些,具体的举一反
: 三,推广应用就只能看孩子的造化。
: 具体到日常,我一般开始让孩子自己发挥做题,除非他问。完了再问他的做法 - 只对
: 那种有点小意思的题而言 - 然后再给他说我的做法。按道理这种方式应该不错,可能
: 是我儿子自尊心太强的缘故,发现这样居然对他有些打击。

c***x
发帖数: 1826
95

这个,有启发,谢谢。
虽然好的老师也可以做到(3),但是可遇不可求。
家长是否要涉及(1),我还需要深入想想。

【在 s**********y 的大作中提到】
: 老师应该是 (1),(2)
: 家长应该是 (3), (2)
: 有是老师又是家长, 自然糊涂了。

s**********y
发帖数: 509
96
总算的点空,开始写 数学教育 一家之言 番外篇 II 袋鼠算数衔接。
当年其实是问过的, 老师也讲了一大段, 没记全, 有些忘了。 正好最近版上讨论了
这样一道题, 我就拿它来入手。
I 班上男孩女孩共10, 男孩比女孩多2, 男孩几, 女孩几。
这个题会做10以内的加法 就应该会做了。
那么大一点就该问的难一点了。 班上男孩女孩共20, 男孩比女孩多2, 男孩几, 女
孩几。
II 这个题会做20以内的加法 就应该会做了。
那么大一点就该问的难一点了。
III
班上男孩女孩共20, 男孩比女孩多4, 男孩几, 女孩几。
班上男孩女孩共20, 男孩比女孩多6, 男孩几, 女孩几。
班上男孩女孩共20, 男孩比女孩多8, 男孩几, 女孩几。
班上男孩女孩共20, 男孩比女孩多10, 男孩几, 女孩几。
小蛙应该总结规律 不用猜了。
如果你娃到了这一步, 你可以往下看, 如果没有, 重复 I- III。
虾面是袋鼠算数的衔接。 开始引入富豪了。
IV 班上男孩女孩共20, 男孩比女孩多N, 男孩几, 女孩几。
如果小蛙自己倒出公式, 恭喜, 你娃迈出了 袋鼠第一步, 请往下。 如果没有, 还
要耐心等待。
V
班上男孩女孩共M, 男孩比女孩多N, 男孩几, 女孩几。
如果小蛙自己倒出公式, 恭喜, 你娃自己解除二元一次方程。 如果没有, 还要耐心
等待。
我们当年老师倒是名震一方。 下次回去, 该问问袋鼠如何进阶了。

【在 s**********y 的大作中提到】
: 这个要赞。
: 本来还有一段, 关于袋鼠算数衔接, 以后的空再写。
:
: simply
: range

s**********y
发帖数: 509
97
老师反正要叫, 何苦浪费宝贵的青紫时间。
反过来说, 万一更老师说的不同, 蛙听谁的?

【在 c***x 的大作中提到】
:
: 这个,有启发,谢谢。
: 虽然好的老师也可以做到(3),但是可遇不可求。
: 家长是否要涉及(1),我还需要深入想想。

c***x
发帖数: 1826
98

你也灌了十多年水了,怎么连观点和事实都混淆?
对于debatable的现象,回复属实或不属实;
对于debatable的观点,回复同意或不同意。

【在 t******l 的大作中提到】
: 属实
:
: :
: :你这明显小瞧了娃凑的能力。

c***x
发帖数: 1826
99

求建议,记得你是专业做这个的,我也希望写的更精确些。

【在 w*********y 的大作中提到】
: 这个概括挺好的。。。有些词换换就更精确了。
s**********y
发帖数: 509
100
赞蛙有造反精神。

?“
,第

【在 h*****m 的大作中提到】
: 你们知识都忒丰富,理论都忒高深。我还是聊聊题目本身吧。
: 话说,某人去见大师说:“我教我孩子鸡兔同笼问题,他总是喜欢猜出来答案,我想教
: 他先假设都是鸡,然后替换一只成兔子增加两支脚的办法来算,他总是不听,怎么办?“
: 大师从身旁拿起一串念珠,说:“老衲这串珠子,是两位施主所赠的168颗珠子串成,第
: 一位施主所赠珠子每颗重10克,第二位施主所赠珠子每颗重14克,整串珠子一共2000克
: 重。你且猜猜看每种珠子多少个?“。某人呆立半晌,说:“我知道了,这岂不就是潮
: 水智人所言,用具体的题目来penalize某种解法,和鼓励另一种解法吗?“。大师笑曰
: :“数目太大,他猜不出来,自然就要按你的办法来算了。再不行,你给他整几万的数
: 字来出题“
: 某人大喜,回家依计而行。唤儿过来,出一巨大数目问题,料儿难以猜出。儿呆立半晌

相关主题
分数应用题系统学习的反面是什么?
代数小朋友不理解怎么办?有些小学四年级的数学题不太会做
教绝对值方程:Khan Academy vs AMC10 答案 vs 分段函数替换?AMC 8 成绩 2016
进入Parenting版参与讨论
c***x
发帖数: 1826
101

少方
一个
for
谢谢潮水老师启发。这个主要和我自己常做的问题都是内点解而不是角点解有关,所以
一般猜中间值然后反复迭代的收敛速度最快,属于杀鸡用牛刀式的思维定式。
你这个例子让我想了想,是不是将来要请你给我家娃做数学私教呢?

【在 t******l 的大作中提到】
: :一个最有效的初始猜测似乎是假设鸡兔一样多,check总脚数如果太多,往鸡多兔少方
: :向revise,反之则反是。对于小学低年级而言,guess, check and revise其实是一个
: :很好的思维方式培养。将来解决很多更复杂的问题,不少也会用到类似的思路。
: 这个例子其实可以看到从 arithmetic 到 algebra 的 leap,如果用俺的在
: arithmetic 和 algebra 当中的 set-theory-based deduction (Magician Table for
: MOEMS),那最初假设常常是假设全部都是鸡更方便,因为这样把一只鸡 flip 成兔子,
: 多 2 条腿。于是一个除法解决问题(外加一个加法)。(当然选中点也不是不可以,
: 但是自找麻烦)。
: 这个跟 sum of arithmetic sequence,在 arithmetic 阶段的娃最初都是找中间点,
: 找左右 pair-wise 的 pattern。而到 algebra 阶段的娃就变成 make another copy,

c***x
发帖数: 1826
102

你这是认真的还是搞笑的?倒是有种序列上的优美。。。

【在 s***n 的大作中提到】
: (3)(2)(1)(2)(3)(2)(1)(2)(3)(2)(1)....
c***x
发帖数: 1826
103

嘛。
果然我刚想到啥,大家就讨论啥。知我心者,此版也。
都很有道理。。。让我再好好想想。

【在 d**********h 的大作中提到】
: 私教可以教技术,教思路,教道理,但是教不了热情,教不了勇气,教不了爱
: 如果私教这些都交了,那么父母就可以退出孩子的生活了
: 这不是说梦,过去老农民把自己娃送到老师家里,读书的是学生,做手艺的是学徒,由
: 此有了师徒胜父子的关系,血缘父子还有啥剩下了?
: 烦恼就在于,一生下娃来就别想省事,这是前辈子欠的 :)
: 想在任何地方偷懒,都会在不久的将来发现这个地方是个窟窿 :)
:
: 嘛。

c***x
发帖数: 1826
104

今天的捧腹大笑配额用掉一次:-)

【在 d**********h 的大作中提到】
: 明白了,
: 纸上谈兵算鸡兔
: 云遮雾罩抢滩涂
: 潮水旧浪换新浪
: 不喜呆仗怨推娘

d**********h
发帖数: 2795
105
下一步可以问:
分析M和N的定义域

【在 s**********y 的大作中提到】
: 总算的点空,开始写 数学教育 一家之言 番外篇 II 袋鼠算数衔接。
: 当年其实是问过的, 老师也讲了一大段, 没记全, 有些忘了。 正好最近版上讨论了
: 这样一道题, 我就拿它来入手。
: I 班上男孩女孩共10, 男孩比女孩多2, 男孩几, 女孩几。
: 这个题会做10以内的加法 就应该会做了。
: 那么大一点就该问的难一点了。 班上男孩女孩共20, 男孩比女孩多2, 男孩几, 女
: 孩几。
: II 这个题会做20以内的加法 就应该会做了。
: 那么大一点就该问的难一点了。
: III

y****i
发帖数: 1504
106
magician table是什么?

少方
一个
for

【在 t******l 的大作中提到】
: :一个最有效的初始猜测似乎是假设鸡兔一样多,check总脚数如果太多,往鸡多兔少方
: :向revise,反之则反是。对于小学低年级而言,guess, check and revise其实是一个
: :很好的思维方式培养。将来解决很多更复杂的问题,不少也会用到类似的思路。
: 这个例子其实可以看到从 arithmetic 到 algebra 的 leap,如果用俺的在
: arithmetic 和 algebra 当中的 set-theory-based deduction (Magician Table for
: MOEMS),那最初假设常常是假设全部都是鸡更方便,因为这样把一只鸡 flip 成兔子,
: 多 2 条腿。于是一个除法解决问题(外加一个加法)。(当然选中点也不是不可以,
: 但是自找麻烦)。
: 这个跟 sum of arithmetic sequence,在 arithmetic 阶段的娃最初都是找中间点,
: 找左右 pair-wise 的 pattern。而到 algebra 阶段的娃就变成 make another copy,

t******l
发帖数: 10908
107
就是一张马粪纸,骗娃说是 magician's table。。。

:magician table是什么?
:【 在 timefall (时光崩塌) 的大作中提到: 】
h*****m
发帖数: 1034
108
你这已经是某人想教给孩子的高级解法了。: )
某人孩子如果就是这么算的,某人就不用去求教大师了。
某人孩子一向是随便猜一个数,不行就继续猜,直到猜对为止。
数目小了,没一会儿也就猜出来了。
所以得上很大的数目让他不能很容易猜出来,才能迫使他接受你说的这种高级算法。

【在 s**********y 的大作中提到】
: 你这明显小瞧了娃凑的能力。
: 168 颗珠子, 假设都是10克, 总重1680克。 2000-1680=320, 320/4=80
: 答案: 80 14克珠子, 88颗10克珠子。 一般蛙新蒜即可。
:
: ?“
: ,第

s**********y
发帖数: 509
109
you got it.

【在 d**********h 的大作中提到】
: 下一步可以问:
: 分析M和N的定义域

t******l
发帖数: 10908
110
我再声明一遍,我没有要求娃这么算。。。我就是在马粪纸的 magician's table
上变个帽子戏法给娃看看,属于娱乐。。。没要求娃这么算。。。

:你这已经是某人想教给孩子的高级解法了。: )
相关主题
AMC 8 成绩 20167,8岁的女孩子暑假在家都干什么呢?
singapore math 到底怎么样啊?鸡兔同笼其实非常简单
给初中学生家长的一些建议 (转载)BSO: 儿子解射雕瑛姑九宫格
进入Parenting版参与讨论
h*****m
发帖数: 1034
111
我没说你要求娃这么算。别紧张。: )
引用你的话只是受你的话启发,想到通过题目本身的变化,来促使娃往某些方向去想。
其实我也没“要求”,只是“希望”而已。

【在 t******l 的大作中提到】
: 我再声明一遍,我没有要求娃这么算。。。我就是在马粪纸的 magician's table
: 上变个帽子戏法给娃看看,属于娱乐。。。没要求娃这么算。。。
:
: :你这已经是某人想教给孩子的高级解法了。: )
: :

c****i
发帖数: 506
112
mark

【在 s**********y 的大作中提到】
: Guess, Check, and Revise 是学好数学的核心
: 去年回国, 找了本地最富盛名的中学老师, 请教代数算数如何衔接。他的回答让我大
: 跌眼睛。猜想, 检验, 和修正。 他说他从不叫公式, 放手让小蛙自己试, 试到那
: 一步算哪一步。 我说这不漫无边际了? 他说自然要加以引导。 秉承一贯风格, 举例
: 如下:
: 鸡兔同笼问题。 鸡兔共20只, 脚64只, 鸡若干/兔若干?
: 鼓励小蛙先猜: 鸡10, 兔10. 脚60. 问题: 先加鸡, 还是先加兔? 答案: 加兔。
: 加几? 不知。 试试? 鸡:9, 兔:11. 脚62. 嘿:再加一就的。
: 鸡兔同笼问题。 鸡兔共20只, 脚70只, 鸡若干/兔若干?
: 还是鼓励小蛙先猜: 鸡10, 兔10. 脚60. 问题: 先加鸡, 还是先加兔? 答案: 加

h*****m
发帖数: 1034
113
文中所说的“大师”也不是你。
“潮水智人”是一传说中的人物,不会亲自下凡来讲小学题目的。: )

【在 h*****m 的大作中提到】
: 我没说你要求娃这么算。别紧张。: )
: 引用你的话只是受你的话启发,想到通过题目本身的变化,来促使娃往某些方向去想。
: 其实我也没“要求”,只是“希望”而已。

h*****m
发帖数: 1034
114
倒也不是限于中国,美国书上也有一种植物上结几个果,另一种植物上结几个果,一共
多少植物多少果这种类似题目的。

【在 r*g 的大作中提到】
: Re. 就一普通三年级题目。因为中国老祖宗碰巧会做,才出了名。
t******l
发帖数: 10908
115
当然话也要说回来,我虽然放浪形骸,但我还是中肯的说,如果期望数学竞赛出名次,
那最好的办法还是送外面专门的训练班、训练队。打个比方就好比想 GRE 高分就要去
新东方一样。
但话再说回来,我自己的小算盘是,GRE 新东方也就拼个半年一年,而且那时都已经成
人了。。。数学竞赛要拼的话,K-11 整整 12 年,外加 99% 的概率是炮灰。。。我看
看 health insurance deductible 就不多想了,我们就当娱乐一下。。。
当然我小时候我爸妈狠多了。。。我就记得吃饭的时候经常听到说:看看,AAA 去当和
尚了,BBB 又去精神病院了。。。

【在 h*****m 的大作中提到】
: 我没说你要求娃这么算。别紧张。: )
: 引用你的话只是受你的话启发,想到通过题目本身的变化,来促使娃往某些方向去想。
: 其实我也没“要求”,只是“希望”而已。

t******l
发帖数: 10908
116
当然,“娱乐” 这词也是有点自欺欺人。。。中肯的说,确保兴趣,这样确保进州立
不错的专业。。。而从保持兴趣角度,竞赛数学题比学校瞎几把傻鼻人肉计算器题,还
是要有趣多了。。。不拼的话,竞赛还是中小学数学里最最有趣的部分,实话实说。。。

【在 t******l 的大作中提到】
: 当然话也要说回来,我虽然放浪形骸,但我还是中肯的说,如果期望数学竞赛出名次,
: 那最好的办法还是送外面专门的训练班、训练队。打个比方就好比想 GRE 高分就要去
: 新东方一样。
: 但话再说回来,我自己的小算盘是,GRE 新东方也就拼个半年一年,而且那时都已经成
: 人了。。。数学竞赛要拼的话,K-11 整整 12 年,外加 99% 的概率是炮灰。。。我看
: 看 health insurance deductible 就不多想了,我们就当娱乐一下。。。
: 当然我小时候我爸妈狠多了。。。我就记得吃饭的时候经常听到说:看看,AAA 去当和
: 尚了,BBB 又去精神病院了。。。

h*****m
发帖数: 1034
117
非常中肯。
对于大多数娃来说,“训练“和“兴趣“之间好像是有天然不可调和的矛盾。
至于养出一个竞赛能出名次的娃,真是可遇而不可求的事情。
还是本着“娱乐”的精神,保持兴趣吧。

。。

【在 t******l 的大作中提到】
: 当然,“娱乐” 这词也是有点自欺欺人。。。中肯的说,确保兴趣,这样确保进州立
: 不错的专业。。。而从保持兴趣角度,竞赛数学题比学校瞎几把傻鼻人肉计算器题,还
: 是要有趣多了。。。不拼的话,竞赛还是中小学数学里最最有趣的部分,实话实说。。。

B********e
发帖数: 10014
118
有意思,尤其是‘放浪形骸’四个字
能不能多点细节?

【在 t******l 的大作中提到】
: 当然话也要说回来,我虽然放浪形骸,但我还是中肯的说,如果期望数学竞赛出名次,
: 那最好的办法还是送外面专门的训练班、训练队。打个比方就好比想 GRE 高分就要去
: 新东方一样。
: 但话再说回来,我自己的小算盘是,GRE 新东方也就拼个半年一年,而且那时都已经成
: 人了。。。数学竞赛要拼的话,K-11 整整 12 年,外加 99% 的概率是炮灰。。。我看
: 看 health insurance deductible 就不多想了,我们就当娱乐一下。。。
: 当然我小时候我爸妈狠多了。。。我就记得吃饭的时候经常听到说:看看,AAA 去当和
: 尚了,BBB 又去精神病院了。。。

f******e
发帖数: 423
119
鸡兔同笼问题。 鸡兔共20只, 脚65只, 鸡若干/兔若干?
are you kidding me?
odd number of feet?

【在 s**********y 的大作中提到】
: Guess, Check, and Revise 是学好数学的核心
: 去年回国, 找了本地最富盛名的中学老师, 请教代数算数如何衔接。他的回答让我大
: 跌眼睛。猜想, 检验, 和修正。 他说他从不叫公式, 放手让小蛙自己试, 试到那
: 一步算哪一步。 我说这不漫无边际了? 他说自然要加以引导。 秉承一贯风格, 举例
: 如下:
: 鸡兔同笼问题。 鸡兔共20只, 脚64只, 鸡若干/兔若干?
: 鼓励小蛙先猜: 鸡10, 兔10. 脚60. 问题: 先加鸡, 还是先加兔? 答案: 加兔。
: 加几? 不知。 试试? 鸡:9, 兔:11. 脚62. 嘿:再加一就的。
: 鸡兔同笼问题。 鸡兔共20只, 脚70只, 鸡若干/兔若干?
: 还是鼓励小蛙先猜: 鸡10, 兔10. 脚60. 问题: 先加鸡, 还是先加兔? 答案: 加

t******l
发帖数: 10908
120
既然首贴是讨论从算术到代数的跳跃,我觉得首先一个问题系列是:
(1)如何才算 “懂” 了代数?
(2)对于只会死套公式的 “人肉代数计算器”,到底算不算 “懂” 代数?。。。或
者推而广之,TI-86 的人肉版实现,算不算有 intelligence in algebra,还是
本质上跟从 Walmart 里买的 TI-86 差别不大?。。。甚至更进一步,对于拿个
TI-86 解 SAT 题考了满分的,从 intelligence 角度,其本质上是不是就是充当了
“TI-86 <=> SAT” 的 “人肉 I/O 接口”。。。也就是说,这个满分的意思是:该
“人肉 I/O 接口” 的实现,得了满分。。。至于是不是有 intelligence in algebra,
天知道?。。。
(3)“懂” 代数到底有啥价值和意义?。。。在当今电算横行的年代,绝大多数猴子
做个不错的 “人肉 I/O 接口” 就已经 far beyond expectation 了。。。那为啥需
要 “懂” 代数?。。。
(4)如何知道是不是 “懂” 了代数?。。。根据著名的 “图灵测试”,死背公式但
不能 draw connection between them 的,显然通不过图灵测试。。。从这个角度说,
SAT / AMC 10 / AMC 12 / AIME / USAMO 测试的基本假设,是 intelligence 总是有
一定程度的 “懒惰”。而题目的变化越多,则基于 “懒惰” 的假设,就越 penalize
死套公式题型,同时 promote draw connection,这样能一定程度的测试真正的
intelligence,同时避免出现让 “弼马温主持图灵测试” 这种国际玩笑。。。但另一
方面,这么看的话,“愚蠢而勤奋” 永远是这类测试(相对而言的)false positive
不是?。。。
先想到这里,在线程里慢慢聊。。。
相关主题
混BBS要抱着贡献和学习的心Q. & A. With Liping Ma
老警察问个新问题什么时候给孩子引入方程概念
如何帮助有一定数学天赋的孩子别鸡兔同笼了,来喝啤酒吧
进入Parenting版参与讨论
i**e
发帖数: 19242
121
实体到抽象的第一次飞跃,质变
代数itself,可能没有那么重要价值也不算大
但是,这个质变很重要
my one cent

algebra,

【在 t******l 的大作中提到】
: 既然首贴是讨论从算术到代数的跳跃,我觉得首先一个问题系列是:
: (1)如何才算 “懂” 了代数?
: (2)对于只会死套公式的 “人肉代数计算器”,到底算不算 “懂” 代数?。。。或
: 者推而广之,TI-86 的人肉版实现,算不算有 intelligence in algebra,还是
: 本质上跟从 Walmart 里买的 TI-86 差别不大?。。。甚至更进一步,对于拿个
: TI-86 解 SAT 题考了满分的,从 intelligence 角度,其本质上是不是就是充当了
: “TI-86 <=> SAT” 的 “人肉 I/O 接口”。。。也就是说,这个满分的意思是:该
: “人肉 I/O 接口” 的实现,得了满分。。。至于是不是有 intelligence in algebra,
: 天知道?。。。
: (3)“懂” 代数到底有啥价值和意义?。。。在当今电算横行的年代,绝大多数猴子

t******l
发帖数: 10908
122
另外 common core math explanation 的问题,是不是理论上试图更接近 “图灵
测试”,但实践上因为智人测试官不太容易找,而演变成 “弼马温主持图灵测试”
的坑爹场景。。。后果大概是猴子统统都满分,而智人统统都瞠目结舌。。。

algebra,

【在 t******l 的大作中提到】
: 既然首贴是讨论从算术到代数的跳跃,我觉得首先一个问题系列是:
: (1)如何才算 “懂” 了代数?
: (2)对于只会死套公式的 “人肉代数计算器”,到底算不算 “懂” 代数?。。。或
: 者推而广之,TI-86 的人肉版实现,算不算有 intelligence in algebra,还是
: 本质上跟从 Walmart 里买的 TI-86 差别不大?。。。甚至更进一步,对于拿个
: TI-86 解 SAT 题考了满分的,从 intelligence 角度,其本质上是不是就是充当了
: “TI-86 <=> SAT” 的 “人肉 I/O 接口”。。。也就是说,这个满分的意思是:该
: “人肉 I/O 接口” 的实现,得了满分。。。至于是不是有 intelligence in algebra,
: 天知道?。。。
: (3)“懂” 代数到底有啥价值和意义?。。。在当今电算横行的年代,绝大多数猴子

t******l
发帖数: 10908
123
死套公式不需要 “抽象” 吧。。。马工说,一个 LUT(Look-Up-Table)足够了。。。

【在 i**e 的大作中提到】
: 实体到抽象的第一次飞跃,质变
: 代数itself,可能没有那么重要价值也不算大
: 但是,这个质变很重要
: my one cent
:
: algebra,

i**e
发帖数: 19242
124
看了aops pre algebra
我说 now we are talking about math using the right language
如果孩子不ready,就读不懂
t******l
发帖数: 10908
125
实话实说,这些数学书我都没买,因为买书容易不落灰难。。。这都是大伙儿讨论热烈
,我就跟着在一起起哄。。。
这么说不 ready 的读不懂,ready 的不会去读。。。难道是竞赛想拿名次的才会去读?

【在 i**e 的大作中提到】
: 看了aops pre algebra
: 我说 now we are talking about math using the right language
: 如果孩子不ready,就读不懂

i**e
发帖数: 19242
126
粗分
一种人,必须懂了才能掌握
一种人,无须懂但是可以按部就班去做,做多了其义自现
学霸,一部分是真懂
一部分是记忆好执行力强先练习再理解掌握,或者边做边悟,由于悟性不错,所以成绩
很好
米国的CC就是希望让娃理解了懂了
可惜,每个概念计算的练习都是蜻蜓点水
孩子的“懂”只不过是 heard about it,能说出来不会运用

。。

【在 t******l 的大作中提到】
: 死套公式不需要 “抽象” 吧。。。马工说,一个 LUT(Look-Up-Table)足够了。。。
t******l
发帖数: 10908
127
:只不过是 heard about it,能说出来不会运用
这本质上属于 “人肉计算器不如”。
话说也不能太嘲笑 “人肉计算器” / Look-Up-Table。。。虽然 LUT 一般发不了
paper,但从马工的 code 到 INTEL 的数学协处理器,没 LUT 的话连跑都没法跑,直
接就地残废。。。

【在 i**e 的大作中提到】
: 粗分
: 一种人,必须懂了才能掌握
: 一种人,无须懂但是可以按部就班去做,做多了其义自现
: 学霸,一部分是真懂
: 一部分是记忆好执行力强先练习再理解掌握,或者边做边悟,由于悟性不错,所以成绩
: 很好
: 米国的CC就是希望让娃理解了懂了
: 可惜,每个概念计算的练习都是蜻蜓点水
: 孩子的“懂”只不过是 heard about it,能说出来不会运用
:

i**e
发帖数: 19242
128
算了,我不多说了
你慢慢忽悠吧:)

读?

【在 t******l 的大作中提到】
: 实话实说,这些数学书我都没买,因为买书容易不落灰难。。。这都是大伙儿讨论热烈
: ,我就跟着在一起起哄。。。
: 这么说不 ready 的读不懂,ready 的不会去读。。。难道是竞赛想拿名次的才会去读?

t******l
发帖数: 10908
129
我确实没买那些书。。。唯一买的是 MOEMS 三大本里面搭卖了一本。那本原来的打算
是给娃当数学辞典用,但实际的后果是落灰。。。我是实话实说。。。你说没人给我开
支票我为啥要忽悠?。。。

【在 i**e 的大作中提到】
: 算了,我不多说了
: 你慢慢忽悠吧:)
:
: 读?

i**e
发帖数: 19242
130
数学普通娃,要是文科发散性思维,如果拒绝练习,就是这水平啊
I know, I know
本质就是
I heard about it and I can talk about it, but pls don't ask me to use it:)
人肉计算器,至少拿个好成绩
对数学的领悟肯定比“I know I know"只会说不练的来的要多的多
生来不是精英,能做计算器也很不错了,至少踏踏实实地长本领有恒心有毅力

【在 t******l 的大作中提到】
: :只不过是 heard about it,能说出来不会运用
: 这本质上属于 “人肉计算器不如”。
: 话说也不能太嘲笑 “人肉计算器” / Look-Up-Table。。。虽然 LUT 一般发不了
: paper,但从马工的 code 到 INTEL 的数学协处理器,没 LUT 的话连跑都没法跑,直
: 接就地残废。。。

相关主题
9岁, 如何报名考amc10代数小朋友不理解怎么办?
求科普: 奥数 vs. Math Olympiad vs. Math Count vs. Math circle vs. 超前学数学教绝对值方程:Khan Academy vs AMC10 答案 vs 分段函数替换?
分数应用题系统学习的反面是什么?
进入Parenting版参与讨论
t******l
发帖数: 10908
131
:I heard about it and I can talk about it。。。
对于这种,我觉得我的反应会是:Do it now, or Tell me when you are going to
try it. That is your choice... Other than that, I don't wanna hear anything!

【在 i**e 的大作中提到】
: 数学普通娃,要是文科发散性思维,如果拒绝练习,就是这水平啊
: I know, I know
: 本质就是
: I heard about it and I can talk about it, but pls don't ask me to use it:)
: 人肉计算器,至少拿个好成绩
: 对数学的领悟肯定比“I know I know"只会说不练的来的要多的多
: 生来不是精英,能做计算器也很不错了,至少踏踏实实地长本领有恒心有毅力

i**e
发帖数: 19242
132
哈哈
然后你娃就去do it了
有的娃就不去,看着你,眼睛里流露的是:u can't make me!!!

anything!

【在 t******l 的大作中提到】
: :I heard about it and I can talk about it。。。
: 对于这种,我觉得我的反应会是:Do it now, or Tell me when you are going to
: try it. That is your choice... Other than that, I don't wanna hear anything!

s***n
发帖数: 1280
133
1. teaching kids facts, concepts, processes, procedures and principles;
2. developing kids’ higher-order thinking skills in the cognitive domain--
applying, analyzing, evaluating, and creating
3. promoting kids’ interest, motivation and aptitudes in the affective
domain.
我是半开玩笑。

【在 c***x 的大作中提到】
:
: 今天的捧腹大笑配额用掉一次:-)

t******l
发帖数: 10908
134
我觉得做父母要有 guts。。。昨晚我娃 complain 找不到 protractor (话说量角器这
种破玩意儿我们家一般都找不到,很正常)。。。但我觉得娃她是又想找借口,于是问
她说,我从没听说过 AMC 需要量角器,啥题?。。。娃说那是学校数学。。。于是我
头也不回的说,你就不会用计算器算个 tangent 再用直尺量一下?。。。娃怒了,说
不能这样。。。我也怒了,说我给你买的 Casio FX-115 ES+ 是吃素的吗?。。。但问
题还是要解决,于是我跑过去问娃到底是多少度。。。娃这才真正看了看题(我擦),
说是 45 度。。。尼玛我立马啪地拉出一张纸对折一下,拍在桌子上说 45度 也要
protractor?Are you nuts?。。。娃理亏,于是开始耍赖,说她前面没看题不知道是
45 度。。。我说,少废话,有 45 度了还不快做题去。。。说完我立马拔腿溜号了,
免得跟叛逆期的娃又一言不合火花四溅。。。

【在 i**e 的大作中提到】
: 哈哈
: 然后你娃就去do it了
: 有的娃就不去,看着你,眼睛里流露的是:u can't make me!!!
:
: anything!

l***y
发帖数: 1166
135
说明你还没理解这题的含义。
1.鸡2腿,兔4腿,兔比鸡多2腿,
2.20只假如全部是鸡,40腿,多出来的64-40=24腿必然是兔子的。24/2=12只兔子。
3.20-12=8 只鸡。
列方程解也是一样,只不过没有去想解方程过程的含义而已,仔细想想和前面推理是一
致的:
x+y=20
2x+4y=64
2x+2y=40(相当于假设全部是鸡)
2x+4y=64
2y=64-40 (多出来的腿必然是兔子的)

【在 r*g 的大作中提到】
: 这是exactly我当时教娃的步骤。随便猜一个初始解,然后一对一对换,改进答案就行
: 。不然你要解释为何一开始假设全是鸡或者兔子,其实一开始假设什么都可以。

t******l
发帖数: 10908
136
当然实际上我对折了两次,因为不能 presume 猴子工厂的 quality control。。。当
然火冒三丈时就不多啰嗦了,省得火花四溅。。。当然如果对直边还不信任,那就对折
三次,不过直边看上去也足够呱嚓了。。。



【在 t******l 的大作中提到】
: 我觉得做父母要有 guts。。。昨晚我娃 complain 找不到 protractor (话说量角器这
: 种破玩意儿我们家一般都找不到,很正常)。。。但我觉得娃她是又想找借口,于是问
: 她说,我从没听说过 AMC 需要量角器,啥题?。。。娃说那是学校数学。。。于是我
: 头也不回的说,你就不会用计算器算个 tangent 再用直尺量一下?。。。娃怒了,说
: 不能这样。。。我也怒了,说我给你买的 Casio FX-115 ES+ 是吃素的吗?。。。但问
: 题还是要解决,于是我跑过去问娃到底是多少度。。。娃这才真正看了看题(我擦),
: 说是 45 度。。。尼玛我立马啪地拉出一张纸对折一下,拍在桌子上说 45度 也要
: protractor?Are you nuts?。。。娃理亏,于是开始耍赖,说她前面没看题不知道是
: 45 度。。。我说,少废话,有 45 度了还不快做题去。。。说完我立马拔腿溜号了,
: 免得跟叛逆期的娃又一言不合火花四溅。。。

i**e
发帖数: 19242
137
高!



【在 t******l 的大作中提到】
: 我觉得做父母要有 guts。。。昨晚我娃 complain 找不到 protractor (话说量角器这
: 种破玩意儿我们家一般都找不到,很正常)。。。但我觉得娃她是又想找借口,于是问
: 她说,我从没听说过 AMC 需要量角器,啥题?。。。娃说那是学校数学。。。于是我
: 头也不回的说,你就不会用计算器算个 tangent 再用直尺量一下?。。。娃怒了,说
: 不能这样。。。我也怒了,说我给你买的 Casio FX-115 ES+ 是吃素的吗?。。。但问
: 题还是要解决,于是我跑过去问娃到底是多少度。。。娃这才真正看了看题(我擦),
: 说是 45 度。。。尼玛我立马啪地拉出一张纸对折一下,拍在桌子上说 45度 也要
: protractor?Are you nuts?。。。娃理亏,于是开始耍赖,说她前面没看题不知道是
: 45 度。。。我说,少废话,有 45 度了还不快做题去。。。说完我立马拔腿溜号了,
: 免得跟叛逆期的娃又一言不合火花四溅。。。

l***y
发帖数: 1166
138
应该是先想出算术解法,后有代数,然后人脑就彻底解放了,以后碰到类似情况都可以
套代数方程。
小孩能自己想出算术解法确实非常牛逼。



【在 s**********y 的大作中提到】
: 一般来说, 诸如此类的技巧, 都是成人用代数的方法解出来后, 再用算数的方法来
: 加以解释。
: 如果小蛙自己能找到如此算法, 又能用如此语言表述, 应该是天赋异禀, 才华横溢
: (或称之为天才)。
: 总的来说, 诸如此类的技巧既不能帮助学习下一阶段(袋鼠), 有无助于上一阶段
: (算数)。 可以称之为鸡肋。

d**********h
发帖数: 2795
139
哈哈,
这就是我所说的袋鼠进阶时期的问题:定义域分析

【在 f******e 的大作中提到】
: 鸡兔同笼问题。 鸡兔共20只, 脚65只, 鸡若干/兔若干?
: are you kidding me?
: odd number of feet?

t*g
发帖数: 1758
140
真的么?计算机解方程就是这么解的。随便给个初值,矫正,再逼近。把人训练成计算
机了
[在 sunflowerboy (lonestar) 的大作中提到:]
:Guess, Check, and Revise 是学好数学的核心
:去年回国, 找了本地最富盛名的中学老师, 请教代数算数如何衔接。他的回答让我
大跌眼睛。猜想, 检验, 和修正。 他说他从不叫公式, 放手让小蛙自己试, 试到那
:一步算哪一步。 我说这不漫无边际了? 他说自然要加以引导。 秉承一贯风格, 举
例如下:
:鸡兔同笼问题。 鸡兔共20只, 脚64只, 鸡若干/兔若干?
:鼓励小蛙先猜: 鸡10, 兔10. 脚60. 问题: 先加鸡, 还是先加兔? 答案: 加兔
。加几? 不知。 试试? 鸡:9, 兔:11. 脚62. 嘿:再加一就的。
:鸡兔同笼问题。 鸡兔共20只, 脚70只, 鸡若干/兔若干?
:还是鼓励小蛙先猜: 鸡10, 兔10. 脚60. 问题: 先加鸡, 还是先加兔? 答案:
加兔。加几? 不知。 从上一个点试起。 鸡:8, 兔:12. 脚64. 再加一, 再加一
。 慢慢也算到15.
:鸡兔同笼问题。 鸡兔共20只, 脚46只, 鸡若干/兔若干?
:还是重复。 慢慢小蛙发现:鸡加一, 脚减二, 兔加一, 脚加二。 这样只要猜一个
:点, 算一下差异, 在除一下就好。
:..........
相关主题
系统学习的反面是什么?singapore math 到底怎么样啊?
有些小学四年级的数学题不太会做给初中学生家长的一些建议 (转载)
AMC 8 成绩 20167,8岁的女孩子暑假在家都干什么呢?
进入Parenting版参与讨论
t******l
发帖数: 10908
141
但我觉得大多数人对这个也就是学个人肉计算器 LUT,否则下面这道题也不至于在 AMC
里放成第 13 题而不是第 1 题送分题。。。2010 AMC 10B problem 13。。。不过话
说我娃已经被这题卡住两次了,虽然第二次比第一次好那么一点。。。
http://www.artofproblemsolving.com/wiki/index.php?title=2010_AM

【在 d**********h 的大作中提到】
: 哈哈,
: 这就是我所说的袋鼠进阶时期的问题:定义域分析

C********e
发帖数: 2327
142
这是歧视

【在 s**********y 的大作中提到】
: 潮水兄是一朝被蛇咬, 十年有余悸。
: 我觉得退妈能把AMC10的题看懂就不错了。 断出不出来36招, 一般也就是3岁能弄个加
: 法, 5岁背99表, 出来显摆显摆, 再往上, 就心有余, 力不足了。
:
: AMC

C********e
发帖数: 2327
143
这个AAA和BBB是数学好的还是不好的?

【在 t******l 的大作中提到】
: 当然话也要说回来,我虽然放浪形骸,但我还是中肯的说,如果期望数学竞赛出名次,
: 那最好的办法还是送外面专门的训练班、训练队。打个比方就好比想 GRE 高分就要去
: 新东方一样。
: 但话再说回来,我自己的小算盘是,GRE 新东方也就拼个半年一年,而且那时都已经成
: 人了。。。数学竞赛要拼的话,K-11 整整 12 年,外加 99% 的概率是炮灰。。。我看
: 看 health insurance deductible 就不多想了,我们就当娱乐一下。。。
: 当然我小时候我爸妈狠多了。。。我就记得吃饭的时候经常听到说:看看,AAA 去当和
: 尚了,BBB 又去精神病院了。。。

t******l
发帖数: 10908
144
不知道。。。但我目测没进冬令营的,去精神病院五千次也不会有人知道名字。。。

:这个AAA和BBB是数学好的还是不好的?
d**********h
发帖数: 2795
145
这种题真的就是要熟练,要心细
你就是数学教授,如果马虎一点也可能会多个曾根或者漏个解之类的

AMC

【在 t******l 的大作中提到】
: 但我觉得大多数人对这个也就是学个人肉计算器 LUT,否则下面这道题也不至于在 AMC
: 里放成第 13 题而不是第 1 题送分题。。。2010 AMC 10B problem 13。。。不过话
: 说我娃已经被这题卡住两次了,虽然第二次比第一次好那么一点。。。
: http://www.artofproblemsolving.com/wiki/index.php?title=2010_AM

d**********h
发帖数: 2795
146
这个计算机只能求数值解,如果是代数,怎么迭代收敛
再复杂一些,如果有多个解,计算机如何保证找全,到了这一步就是咱前面说的那个了
“需要一些智力,而不仅仅是力气了”
数值迭代的局限性已经有了充分研究,比如初值依赖性,混沌分叉。。。
以力压人终究不是万能解

到那

【在 t*g 的大作中提到】
: 真的么?计算机解方程就是这么解的。随便给个初值,矫正,再逼近。把人训练成计算
: 机了
: [在 sunflowerboy (lonestar) 的大作中提到:]
: :Guess, Check, and Revise 是学好数学的核心
: :去年回国, 找了本地最富盛名的中学老师, 请教代数算数如何衔接。他的回答让我
: 大跌眼睛。猜想, 检验, 和修正。 他说他从不叫公式, 放手让小蛙自己试, 试到那
: :一步算哪一步。 我说这不漫无边际了? 他说自然要加以引导。 秉承一贯风格, 举
: 例如下:
: :鸡兔同笼问题。 鸡兔共20只, 脚64只, 鸡若干/兔若干?
: :鼓励小蛙先猜: 鸡10, 兔10. 脚60. 问题: 先加鸡, 还是先加兔? 答案: 加兔

B********e
发帖数: 10014
147
只看到了nuts,guts在哪里?呵呵



【在 t******l 的大作中提到】
: 我觉得做父母要有 guts。。。昨晚我娃 complain 找不到 protractor (话说量角器这
: 种破玩意儿我们家一般都找不到,很正常)。。。但我觉得娃她是又想找借口,于是问
: 她说,我从没听说过 AMC 需要量角器,啥题?。。。娃说那是学校数学。。。于是我
: 头也不回的说,你就不会用计算器算个 tangent 再用直尺量一下?。。。娃怒了,说
: 不能这样。。。我也怒了,说我给你买的 Casio FX-115 ES+ 是吃素的吗?。。。但问
: 题还是要解决,于是我跑过去问娃到底是多少度。。。娃这才真正看了看题(我擦),
: 说是 45 度。。。尼玛我立马啪地拉出一张纸对折一下,拍在桌子上说 45度 也要
: protractor?Are you nuts?。。。娃理亏,于是开始耍赖,说她前面没看题不知道是
: 45 度。。。我说,少废话,有 45 度了还不快做题去。。。说完我立马拔腿溜号了,
: 免得跟叛逆期的娃又一言不合火花四溅。。。

t******l
发帖数: 10908
148
这题不需要太细心,更不需要熟练。。。两个绝对值符号嵌套,decision-tree branch
2 次,最多四个方程带定义域,当中发生退化现象就少一些。。。至少我娃不是不细心,
而是 totally lost。。。我觉得对大多数 10 年级娃,应该不是栽在细心上。。。另外
标准答案写太多字,太 literature 化,当然是写成网页给大家看没办法,题外话。。。
这种题目都还要反复熟练以后才能做,那多半还没理解这题的本质是 follow order-of
-operation、用 binary decision-tree、展开成 piece-wise-linear。。。我现在看
到 “熟练” 两字就不由得想起 “人肉计算器”,看到 “心细” 两字就不由得想起
“人肉计算器v2.0”。。。

【在 d**********h 的大作中提到】
: 这种题真的就是要熟练,要心细
: 你就是数学教授,如果马虎一点也可能会多个曾根或者漏个解之类的
:
: AMC

t******l
发帖数: 10908
149
另外不要拿数学教授说事儿,人是早就不玩这个,外加年龄也大了。。。否则初中生弄个
老态龙钟的样子也能体育及格了。。。话糙理不糙!!!

branch
心,
另外
。。
of


【在 t******l 的大作中提到】
: 这题不需要太细心,更不需要熟练。。。两个绝对值符号嵌套,decision-tree branch
: 2 次,最多四个方程带定义域,当中发生退化现象就少一些。。。至少我娃不是不细心,
: 而是 totally lost。。。我觉得对大多数 10 年级娃,应该不是栽在细心上。。。另外
: 标准答案写太多字,太 literature 化,当然是写成网页给大家看没办法,题外话。。。
: 这种题目都还要反复熟练以后才能做,那多半还没理解这题的本质是 follow order-of
: -operation、用 binary decision-tree、展开成 piece-wise-linear。。。我现在看
: 到 “熟练” 两字就不由得想起 “人肉计算器”,看到 “心细” 两字就不由得想起
: “人肉计算器v2.0”。。。

i**e
发帖数: 19242
150
呵呵
来来来
show show ur push-mama's 神彩
闪瞎TA那双歧视的眼
哈哈哈

【在 C********e 的大作中提到】
: 这是歧视
相关主题
鸡兔同笼其实非常简单老警察问个新问题
BSO: 儿子解射雕瑛姑九宫格如何帮助有一定数学天赋的孩子
混BBS要抱着贡献和学习的心Q. & A. With Liping Ma
进入Parenting版参与讨论
i**e
发帖数: 19242
151
有nuts了
guts还会远吗?:)

【在 B********e 的大作中提到】
: 只看到了nuts,guts在哪里?呵呵
:
: 是

t******l
发帖数: 10908
152
我觉得跟推爸推妈数学水准关系不大。。。主要原因我猜要么是爱幻想,所以视自己的
过去而不见;要么是干脆也没记住自己的过去。。。否则如果真的小学四年级就能用代
数方法解鸡兔同笼不在话下的话,那高一至少给进个冬令营吧,如果不是集训队的话。
。。否则的话,这把学校数学教研组的脸都给丢尽了。。。

【在 i**e 的大作中提到】
: 呵呵
: 来来来
: show show ur push-mama's 神彩
: 闪瞎TA那双歧视的眼
: 哈哈哈

t******l
发帖数: 10908
153
再不信的话,看个统计数据好了:
今年 AMC 10A,全美 6 年级及以下统计:
>= 60.0 :总共 626 个人头(6 年级及以下)
>= 90.0 :总共 180 个人头(6 年级及以下)
AMC 10 拿个 60 分根本不需要会代数法解鸡兔同笼(simultaneous equation),
拿 90 分可能需要会。。。这还是 6 年级及以下,而不是 4 年级及以下。。。
(人 AoPS 都不单独统计 4 年级及以下的人头数)。。。再不信的话,counting
towards 500 总不可能不会吧。。。

【在 t******l 的大作中提到】
: 我觉得跟推爸推妈数学水准关系不大。。。主要原因我猜要么是爱幻想,所以视自己的
: 过去而不见;要么是干脆也没记住自己的过去。。。否则如果真的小学四年级就能用代
: 数方法解鸡兔同笼不在话下的话,那高一至少给进个冬令营吧,如果不是集训队的话。
: 。。否则的话,这把学校数学教研组的脸都给丢尽了。。。

B********e
发帖数: 10014
154
你是在开黄腔么

【在 i**e 的大作中提到】
: 有nuts了
: guts还会远吗?:)

B********e
发帖数: 10014
155
时光兄弟,在哪里搞得这些好数据?

【在 t******l 的大作中提到】
: 再不信的话,看个统计数据好了:
: 今年 AMC 10A,全美 6 年级及以下统计:
: >= 60.0 :总共 626 个人头(6 年级及以下)
: >= 90.0 :总共 180 个人头(6 年级及以下)
: AMC 10 拿个 60 分根本不需要会代数法解鸡兔同笼(simultaneous equation),
: 拿 90 分可能需要会。。。这还是 6 年级及以下,而不是 4 年级及以下。。。
: (人 AoPS 都不单独统计 4 年级及以下的人头数)。。。再不信的话,counting
: towards 500 总不可能不会吧。。。

B********e
发帖数: 10014
156
以前我觉得教孩子是个简单事,至少目前2年级是。
看你们的讨论越来越觉得好复杂好头大啊,
到6年级感觉比带研究生还复杂

【在 t******l 的大作中提到】
: 再不信的话,看个统计数据好了:
: 今年 AMC 10A,全美 6 年级及以下统计:
: >= 60.0 :总共 626 个人头(6 年级及以下)
: >= 90.0 :总共 180 个人头(6 年级及以下)
: AMC 10 拿个 60 分根本不需要会代数法解鸡兔同笼(simultaneous equation),
: 拿 90 分可能需要会。。。这还是 6 年级及以下,而不是 4 年级及以下。。。
: (人 AoPS 都不单独统计 4 年级及以下的人头数)。。。再不信的话,counting
: towards 500 总不可能不会吧。。。

t******l
发帖数: 10908
157
Google search "AMC 10 statistics"

:时光兄弟,在哪里搞得这些好数据?
:【 在 timefall (时光崩塌) 的大作中提到: 】
B********e
发帖数: 10014
158
兄弟你的帖子猛一看乱糟糟一片片
但是就像一个挖不完的宝藏啊
可以允许我给你发个包子吗?

【在 t******l 的大作中提到】
: Google search "AMC 10 statistics"
:
: :时光兄弟,在哪里搞得这些好数据?
: :【 在 timefall (时光崩塌) 的大作中提到: 】

t******l
发帖数: 10908
159
只要是包子,俺就笑纳先。。。

:兄弟你的帖子猛一看乱糟糟一片片
:但是就像一个挖不完的宝藏啊
t******l
发帖数: 10908
160
本版沉底线是集训队,贫困线是陶天才,及格线是高斯。。。你带的研究生里有人姓高
名斯?

:以前我觉得教孩子是个简单事,至少目前2年级是。
:看你们的讨论越来越觉得好复杂好头大啊,
相关主题
什么时候给孩子引入方程概念求科普: 奥数 vs. Math Olympiad vs. Math Count vs. Math circle vs. 超前学数学
别鸡兔同笼了,来喝啤酒吧分数应用题
9岁, 如何报名考amc10代数小朋友不理解怎么办?
进入Parenting版参与讨论
B********e
发帖数: 10014
161
高斯,字圆圆。

【在 t******l 的大作中提到】
: 本版沉底线是集训队,贫困线是陶天才,及格线是高斯。。。你带的研究生里有人姓高
: 名斯?
:
: :以前我觉得教孩子是个简单事,至少目前2年级是。
: :看你们的讨论越来越觉得好复杂好头大啊,

i**e
发帖数: 19242
162
果然学好E文很重要啊
还真就是想到了,冬天来了春天还会远吗,开了个玩笑
没想到居然可以双关到有色笑话啊,醉了

【在 B********e 的大作中提到】
: 你是在开黄腔么
i**e
发帖数: 19242
163
没啥不信的
我对奥竞不感兴趣,can't afford it,出门吃豆腐脑
几年前争论fiction,info text的时候就想得很明白的事了
不可望又不可及的东西,有空拿来玩玩锻炼一下大脑罢了
玩趴下了是正常,玩转是幸运

【在 t******l 的大作中提到】
: 再不信的话,看个统计数据好了:
: 今年 AMC 10A,全美 6 年级及以下统计:
: >= 60.0 :总共 626 个人头(6 年级及以下)
: >= 90.0 :总共 180 个人头(6 年级及以下)
: AMC 10 拿个 60 分根本不需要会代数法解鸡兔同笼(simultaneous equation),
: 拿 90 分可能需要会。。。这还是 6 年级及以下,而不是 4 年级及以下。。。
: (人 AoPS 都不单独统计 4 年级及以下的人头数)。。。再不信的话,counting
: towards 500 总不可能不会吧。。。

t******l
发帖数: 10908
164
对数学竞赛不感兴趣很好理解,毕竟人各有所好,不必强求一致。。。
我只是看不懂那些纠结三年级会不会鸡兔同笼代数解法,但目标却不是 USAMO
qualifiers 那种,也不知道这么急吼吼是图个啥。。。真要目标是 USAMO qualifiers
or above,那倒是容易理解。。。当然话说回来,比起那种跳级但不参加任何竞赛,
纯属为纳税人省钱的假私济公的雷锋精神,还是要好理解多了。。。

:没啥不信的
:我对奥竞不感兴趣,can't afford it,出门吃豆腐脑
l****i
发帖数: 406
165
昨天看了这个贴,晚上就和preK的娃有了第一次关于鸡兔同笼的对话:
我: 如果你去动物园,发现一个笼子里又有兔子又有鸡。。。
娃哈哈大笑: 这动物园太搞笑了,怎么把鸡和兔子关在一个笼子里?
我: 嗯,好玩吧?现在告诉你笼子里一共有两个头,6只脚,笼子里有几只兔子几只鸡。
娃不假思索立刻说:一只兔子一只鸡,一共就两只,又有兔子又有鸡,还管脚干嘛?
我:哦,这道题出失败了。那么要是有三个头,8只脚呢?
娃开始看着我的表情猜:一只兔子两只鸡?还是两只兔子一只鸡?
然后又扳着手指头算了半天,也没算清楚,扔下我跑去玩了。

【在 s**********y 的大作中提到】
: Guess, Check, and Revise 是学好数学的核心
: 去年回国, 找了本地最富盛名的中学老师, 请教代数算数如何衔接。他的回答让我大
: 跌眼睛。猜想, 检验, 和修正。 他说他从不叫公式, 放手让小蛙自己试, 试到那
: 一步算哪一步。 我说这不漫无边际了? 他说自然要加以引导。 秉承一贯风格, 举例
: 如下:
: 鸡兔同笼问题。 鸡兔共20只, 脚64只, 鸡若干/兔若干?
: 鼓励小蛙先猜: 鸡10, 兔10. 脚60. 问题: 先加鸡, 还是先加兔? 答案: 加兔。
: 加几? 不知。 试试? 鸡:9, 兔:11. 脚62. 嘿:再加一就的。
: 鸡兔同笼问题。 鸡兔共20只, 脚70只, 鸡若干/兔若干?
: 还是鼓励小蛙先猜: 鸡10, 兔10. 脚60. 问题: 先加鸡, 还是先加兔? 答案: 加

s**********y
发帖数: 509
166
至少练了加法, 练着练着, 小蛙自己就琢磨出来了。

鸡。

【在 l****i 的大作中提到】
: 昨天看了这个贴,晚上就和preK的娃有了第一次关于鸡兔同笼的对话:
: 我: 如果你去动物园,发现一个笼子里又有兔子又有鸡。。。
: 娃哈哈大笑: 这动物园太搞笑了,怎么把鸡和兔子关在一个笼子里?
: 我: 嗯,好玩吧?现在告诉你笼子里一共有两个头,6只脚,笼子里有几只兔子几只鸡。
: 娃不假思索立刻说:一只兔子一只鸡,一共就两只,又有兔子又有鸡,还管脚干嘛?
: 我:哦,这道题出失败了。那么要是有三个头,8只脚呢?
: 娃开始看着我的表情猜:一只兔子两只鸡?还是两只兔子一只鸡?
: 然后又扳着手指头算了半天,也没算清楚,扔下我跑去玩了。

l****i
发帖数: 406
167
刚才早饭时聊天又说起来。
我:昨天的3个头8只脚的问题想出来了没有?
娃:想出来了,两只兔子,没有鸡。
我:为什么?
娃:两只兔子,刚好八只脚啊。
我:可是两只兔子只有两个头,笼子里有三个头啊?
娃:有一个兔子想这个问题想糊涂了,一个头就变成了两个,笼子里就变成三个头了。

【在 s**********y 的大作中提到】
: 至少练了加法, 练着练着, 小蛙自己就琢磨出来了。
:
: 鸡。

d**********h
发帖数: 2795
168
细心和马虎谁对问题复杂度而言的
你给娃只套一层绝对值符号试试,百分之九十九没问题
你给她套十层试试,百分之五十会出错
不用那么难,你自己找笔和纸写阿拉伯数字,你看你自己可以集中精神写到多少不出错
(写错再改不算数的)
这个是大脑的神经系统的生理极限,没人可以无限期的集中精神。依赖于大脑的发育程
度,所受训练
这个和人肉计算器没有关系,你就是抽象思维也同样受此约束

branch
心,
另外
。。
of


【在 t******l 的大作中提到】
: 这题不需要太细心,更不需要熟练。。。两个绝对值符号嵌套,decision-tree branch
: 2 次,最多四个方程带定义域,当中发生退化现象就少一些。。。至少我娃不是不细心,
: 而是 totally lost。。。我觉得对大多数 10 年级娃,应该不是栽在细心上。。。另外
: 标准答案写太多字,太 literature 化,当然是写成网页给大家看没办法,题外话。。。
: 这种题目都还要反复熟练以后才能做,那多半还没理解这题的本质是 follow order-of
: -operation、用 binary decision-tree、展开成 piece-wise-linear。。。我现在看
: 到 “熟练” 两字就不由得想起 “人肉计算器”,看到 “心细” 两字就不由得想起
: “人肉计算器v2.0”。。。

t******l
发帖数: 10908
169
一层绝对值符号用 naive intuition 目测(相当于 guess & check 的另一种形式)都
不会出错。。。如果两层绝对值符号就需要非常细心认真,或者要练上十几遍手熟,基
本说明还在 guess & check,代数基本不会。。。


细心和马虎谁对问题复杂度而言的
x***1
发帖数: 999
170
其实这些题比鸡兔更reality 一些,考试常考。不知大家咋教小孩。
我娃三年级,不用凑,应直接列式计算。我这么教,总数加上一个比另一个多的数,就
等于两倍的这个比较大的数,除以2,就是这个比较大的数,孩子似懂非懂。
另一种解释:
男孩加女孩等于10,男孩等于女孩加2,等于说女孩加女孩加2等于10,也就是说2times
女孩加2等于10,。。。等于在教代数,娃明白。
这些题的升级版是:
一个年级有三个班,总人数100,二班比一班多10人,三班比二班多14人,问三个班各
多少人?
不用代数,列式计算。

【在 s**********y 的大作中提到】
: 总算的点空,开始写 数学教育 一家之言 番外篇 II 袋鼠算数衔接。
: 当年其实是问过的, 老师也讲了一大段, 没记全, 有些忘了。 正好最近版上讨论了
: 这样一道题, 我就拿它来入手。
: I 班上男孩女孩共10, 男孩比女孩多2, 男孩几, 女孩几。
: 这个题会做10以内的加法 就应该会做了。
: 那么大一点就该问的难一点了。 班上男孩女孩共20, 男孩比女孩多2, 男孩几, 女
: 孩几。
: II 这个题会做20以内的加法 就应该会做了。
: 那么大一点就该问的难一点了。
: III

相关主题
代数小朋友不理解怎么办?有些小学四年级的数学题不太会做
教绝对值方程:Khan Academy vs AMC10 答案 vs 分段函数替换?AMC 8 成绩 2016
系统学习的反面是什么?singapore math 到底怎么样啊?
进入Parenting版参与讨论
t******l
发帖数: 10908
171
这取决于这个考试是普通班考试,还是天才班考试,还是数学竞赛。
如果是四年级及以下的普通班考试,我目测 problem size 应该 guess & check 就行
了。guess & check 普通班的老师会教,这种普通班的 guess & check,家长一般不需
要重复基建。

:其实这些题比鸡兔更reality 一些,考试常考。不知大家咋教小孩。
t******l
发帖数: 10908
172
其实说 “代数基本不会” 还是有点过。。。认真确切的说,应该归类于 “人肉代数
计算器”,被编程过的就按只会葫芦话葫芦,而没被编程过的就直接不会,照着葫芦也
画不出瓢的那型。。。

:一层绝对值符号用 naive intuition 目测(相当于 guess & check 的另一种形式)
都不会出错。。。如果两层绝对值符号就需要非常细心认真,或者要练上十几遍手熟,
基本说明还在 guess & check,代数基本不会。。。
i**e
发帖数: 19242
173
你是认真的吗?
我在现实生活在网上,都没有遇见过你说的这种人
“纠结三年级会不会鸡兔同笼代数解法,但目标却不是 USAMO qualifiers”
不管孩子刚生下来的时候有多少期望幻想理想
磨过几年就会越来越现实了
所以呢,人在网上,往往是跟自己的假想敌在较劲

qualifiers

【在 t******l 的大作中提到】
: 对数学竞赛不感兴趣很好理解,毕竟人各有所好,不必强求一致。。。
: 我只是看不懂那些纠结三年级会不会鸡兔同笼代数解法,但目标却不是 USAMO
: qualifiers 那种,也不知道这么急吼吼是图个啥。。。真要目标是 USAMO qualifiers
: or above,那倒是容易理解。。。当然话说回来,比起那种跳级但不参加任何竞赛,
: 纯属为纳税人省钱的假私济公的雷锋精神,还是要好理解多了。。。
:
: :没啥不信的
: :我对奥竞不感兴趣,can't afford it,出门吃豆腐脑

t******l
发帖数: 10908
174
当然话再说回来,“人肉代数计算器” 也还可以了,好歹是个计算器不是?比起 SAT
数学那种 “TI-82 人肉 I/O 转换头” 还是强太多了。。。神马 SAT 数学满分,也就
是 4 pin 到 6 pin 的转换头接头质量不错,pin 都是镀金的保证接点不断线!!

:其实说 “代数基本不会” 还是有点过。。。认真确切的说,应该归类于 “人肉代数
:计算器”,被编程过的就按只会葫芦话葫芦,而没被编程过的就直接不会,照着葫芦
也画不出瓢的那型。。。
t******l
发帖数: 10908
175
老实说,现实生活中这种人基本没有。就好比现实生活中就基本没有狠推才艺钢琴走火
入魔的。。。狠推钢琴的基本娃一听就是货真价实 gifted 的那种。。。
但网上不一样啊,网上从前这里钢琴智商、钢琴数学、钢琴万金油大力丸的帖子还少么
?。。。所以上网就快意恩仇一下也是灌水的一种方式不是?

:你是认真的吗?
:我在现实生活在网上,都没有遇见过你说的这种人
t******l
发帖数: 10908
176
当然这里比 NG 版还是好很多。。。NG 版那地,快意恩仇的兴致都不会有。。。

i**e
发帖数: 19242
177
这种帖子和讨论,我都不看的
浪费我的时间的事选择不去做罢了
本来就是啊
打听奥赛的,多半孩子是有那个潜力的
没那个潜力的,打听这些事给为的又是哪几毛呢?

【在 t******l 的大作中提到】
: 老实说,现实生活中这种人基本没有。就好比现实生活中就基本没有狠推才艺钢琴走火
: 入魔的。。。狠推钢琴的基本娃一听就是货真价实 gifted 的那种。。。
: 但网上不一样啊,网上从前这里钢琴智商、钢琴数学、钢琴万金油大力丸的帖子还少么
: ?。。。所以上网就快意恩仇一下也是灌水的一种方式不是?
:
: :你是认真的吗?
: :我在现实生活在网上,都没有遇见过你说的这种人

i**e
发帖数: 19242
178
呵呵
你还逛NG啊?
那个荷尔蒙level,我match不了:)

【在 t******l 的大作中提到】
: 当然这里比 NG 版还是好很多。。。NG 版那地,快意恩仇的兴致都不会有。。。
:
: :

t******l
发帖数: 10908
179
但是我看到大力丸贴就快意恩仇一下,这不违法吧。。。

:这种帖子和讨论,我都不看的
:浪费我的时间的事选择不去做罢了
i**e
发帖数: 19242
180
绝对不违法
对你来说多多益善 good luck!

【在 t******l 的大作中提到】
: 但是我看到大力丸贴就快意恩仇一下,这不违法吧。。。
:
: :这种帖子和讨论,我都不看的
: :浪费我的时间的事选择不去做罢了

相关主题
给初中学生家长的一些建议 (转载)BSO: 儿子解射雕瑛姑九宫格
7,8岁的女孩子暑假在家都干什么呢?混BBS要抱着贡献和学习的心
鸡兔同笼其实非常简单老警察问个新问题
进入Parenting版参与讨论
d****g
发帖数: 7460
181
绝对值那题。霸王开,然后在验证,就容易了。先讨论在开很容易疯掉。
B********e
发帖数: 10014
182
是不是人生处处有惊喜的感觉?

【在 i**e 的大作中提到】
: 果然学好E文很重要啊
: 还真就是想到了,冬天来了春天还会远吗,开了个玩笑
: 没想到居然可以双关到有色笑话啊,醉了

i**e
发帖数: 19242
183
处处是地雷 智商欠费的在下 炸飞了都不知道是怎么挂的,呵呵
最近有空上来耍啊?

【在 B********e 的大作中提到】
: 是不是人生处处有惊喜的感觉?
t******l
发帖数: 10908
184
那题目本质就是把绝对值那个 operator 替换成 2 段的 piece-wise linear (
segmented expression),两个绝对值符号就 follow order-of-operation 走
decision-tree(decision-tree 简而言之也就是 if-then-else 的 spatial 表述)。
这基本上就是代数里最 “正合” 的做法,既不需要事先把所有的定义域先讨论,也不
需要事先不管三七二十一霸王开,搞成这两种极端的搞法(那些极端搞法,多几层迟早
自己把自己搞糊涂)。。。分两步 deduction,每层绝对值替换算一步,老老实实一步
一步的 deduction 的过程中,decision-tree 自然 splice 就行了。。。根本不麻烦
,也不会错。。。

【在 d****g 的大作中提到】
: 绝对值那题。霸王开,然后在验证,就容易了。先讨论在开很容易疯掉。
B********e
发帖数: 10014
185
对不起刚才说了一句话,然后摸了摸肚皮,默默删了
暑假计划是,健身和来本版学习!

【在 i**e 的大作中提到】
: 处处是地雷 智商欠费的在下 炸飞了都不知道是怎么挂的,呵呵
: 最近有空上来耍啊?

C********e
发帖数: 2327
186


【在 l****i 的大作中提到】
: 刚才早饭时聊天又说起来。
: 我:昨天的3个头8只脚的问题想出来了没有?
: 娃:想出来了,两只兔子,没有鸡。
: 我:为什么?
: 娃:两只兔子,刚好八只脚啊。
: 我:可是两只兔子只有两个头,笼子里有三个头啊?
: 娃:有一个兔子想这个问题想糊涂了,一个头就变成了两个,笼子里就变成三个头了。

t******l
发帖数: 10908
187
仿佛俺没看见那贴似的。。。

【在 B********e 的大作中提到】
: 对不起刚才说了一句话,然后摸了摸肚皮,默默删了
: 暑假计划是,健身和来本版学习!

B********e
发帖数: 10014
188
太熟了,夸你不能太直接嘛

【在 t******l 的大作中提到】
: 仿佛俺没看见那贴似的。。。
t******l
发帖数: 10908
189
另外我拿这题说事的主要原因是这题是 “正合”。。。如果 “正合” 教不会或者教
歪了,那是俺的问题。。。而如果 “奇胜” (比如 “乱序计算”) 教不会的话,至
少可以归于娃太笨,上帝的事,父母别费心了,顺其自然。。。

【在 t******l 的大作中提到】
: 那题目本质就是把绝对值那个 operator 替换成 2 段的 piece-wise linear (
: segmented expression),两个绝对值符号就 follow order-of-operation 走
: decision-tree(decision-tree 简而言之也就是 if-then-else 的 spatial 表述)。
: 这基本上就是代数里最 “正合” 的做法,既不需要事先把所有的定义域先讨论,也不
: 需要事先不管三七二十一霸王开,搞成这两种极端的搞法(那些极端搞法,多几层迟早
: 自己把自己搞糊涂)。。。分两步 deduction,每层绝对值替换算一步,老老实实一步
: 一步的 deduction 的过程中,decision-tree 自然 splice 就行了。。。根本不麻烦
: ,也不会错。。。

i**e
发帖数: 19242
190
奥?错过了...

【在 B********e 的大作中提到】
: 对不起刚才说了一句话,然后摸了摸肚皮,默默删了
: 暑假计划是,健身和来本版学习!

相关主题
如何帮助有一定数学天赋的孩子别鸡兔同笼了,来喝啤酒吧
Q. & A. With Liping Ma9岁, 如何报名考amc10
什么时候给孩子引入方程概念求科普: 奥数 vs. Math Olympiad vs. Math Count vs. Math circle vs. 超前学数学
进入Parenting版参与讨论
i**e
发帖数: 19242
191
那么,我应该说
错过是一种幸福
还是说
错过是一种幸运呢?

【在 t******l 的大作中提到】
: 仿佛俺没看见那贴似的。。。
i**e
发帖数: 19242
192
教授啊?
不出去玩?
旅途上遇到各种神人是学习的更好的机会呢:)

【在 B********e 的大作中提到】
: 对不起刚才说了一句话,然后摸了摸肚皮,默默删了
: 暑假计划是,健身和来本版学习!

t******l
发帖数: 10908
193
包打听还是要打听的,未来其实不可知,古人云:“有备无患 / 备而不用,没大错”
。。。赵云刚才发电报说了,这急中生智也还是要有锦囊在手的。。。这样在刘邦和项
羽为了 Kumon 而在鸿门宴上大打出手的时候,赵云从锦囊里掏出一张 AMC,跟项羽说
C stands for Computation,跟刘邦说 C stands for Competition,这样鸿门宴上就
一拍即合了,多省事啊。。。所以从此古人云:“Computation 也是 C,Competition
也是 C,Computation 放在 Competition 里,俩 C 并一 C!!!”。。。// run

【在 i**e 的大作中提到】
: 这种帖子和讨论,我都不看的
: 浪费我的时间的事选择不去做罢了
: 本来就是啊
: 打听奥赛的,多半孩子是有那个潜力的
: 没那个潜力的,打听这些事给为的又是哪几毛呢?

B********e
发帖数: 10014
194
。。。mm你不是开黄腔,就是在开黄腔的路上么?

【在 i**e 的大作中提到】
: 教授啊?
: 不出去玩?
: 旅途上遇到各种神人是学习的更好的机会呢:)

i**e
发帖数: 19242
195

什么叫有口难辩
什么叫六月飞雪
什么叫智者见智
什么叫浮想联翩
假作真时真亦假,无为有处有还无
那,嫩就慢慢观赏透过镜片的黄段子吧:)

【在 B********e 的大作中提到】
: 。。。mm你不是开黄腔,就是在开黄腔的路上么?
B********e
发帖数: 10014
196
那你认真开一个,我看看有什么不一样

【在 i**e 的大作中提到】
: 晕
: 什么叫有口难辩
: 什么叫六月飞雪
: 什么叫智者见智
: 什么叫浮想联翩
: 假作真时真亦假,无为有处有还无
: 那,嫩就慢慢观赏透过镜片的黄段子吧:)

i**e
发帖数: 19242
197
这个万万不能啊
岂不屈辱和白白浪费了您的高智商啊
请保持贴贴看出彩儿的节奏啊,必须的,加油!

【在 B********e 的大作中提到】
: 那你认真开一个,我看看有什么不一样
B********e
发帖数: 10014
198
对我这种以低智商为荣的家伙,你这贴的确是侮辱啊
非开一个不能抚平我的创伤哈
[在 izze (若水) 的大作中提到:]
:这个万万不能啊
:岂不屈辱和白白浪费了您的高智商啊
:请保持贴贴看出彩儿的节奏啊,必须的,加油!
s**********y
发帖数: 509
199
我怎么看着 怎么也叫不过来啊。
男孩女孩一共十人, 男孩比女孩的两倍还多1, 男孩几, 女孩几
男孩女孩一共十人, 男孩比女孩的一半还多1, 男孩几, 女孩几
男孩女孩一共十人, 男孩比女孩的三倍还多2, 男孩几, 女孩几
男孩女孩一共十人, 男孩比女孩的4倍还多5, 男孩几, 女孩几
男孩女孩一共十人, 男孩比女孩的5倍还多4, 男孩几, 女孩几
难不成 每道题您都总结一个累死 “总数加上一个比另一个多的数初二”的办法让蛙明
白?让我我也不明白啊。

2times

【在 x***1 的大作中提到】
: 其实这些题比鸡兔更reality 一些,考试常考。不知大家咋教小孩。
: 我娃三年级,不用凑,应直接列式计算。我这么教,总数加上一个比另一个多的数,就
: 等于两倍的这个比较大的数,除以2,就是这个比较大的数,孩子似懂非懂。
: 另一种解释:
: 男孩加女孩等于10,男孩等于女孩加2,等于说女孩加女孩加2等于10,也就是说2times
: 女孩加2等于10,。。。等于在教代数,娃明白。
: 这些题的升级版是:
: 一个年级有三个班,总人数100,二班比一班多10人,三班比二班多14人,问三个班各
: 多少人?
: 不用代数,列式计算。

i**e
发帖数: 19242
200
得,打住了,越说越说不到一起了
虽说a picture worth a thousand words
可小学毕业的我实在是看不懂啊,辜负了

【在 B********e 的大作中提到】
: 对我这种以低智商为荣的家伙,你这贴的确是侮辱啊
: 非开一个不能抚平我的创伤哈
: [在 izze (若水) 的大作中提到:]
: :这个万万不能啊
: :岂不屈辱和白白浪费了您的高智商啊
: :请保持贴贴看出彩儿的节奏啊,必须的,加油!

相关主题
分数应用题系统学习的反面是什么?
代数小朋友不理解怎么办?有些小学四年级的数学题不太会做
教绝对值方程:Khan Academy vs AMC10 答案 vs 分段函数替换?AMC 8 成绩 2016
进入Parenting版参与讨论
x***1
发帖数: 999
201
你如何给小孩子讲这些题,我也觉得我讲的别扭。

【在 s**********y 的大作中提到】
: 我怎么看着 怎么也叫不过来啊。
: 男孩女孩一共十人, 男孩比女孩的两倍还多1, 男孩几, 女孩几
: 男孩女孩一共十人, 男孩比女孩的一半还多1, 男孩几, 女孩几
: 男孩女孩一共十人, 男孩比女孩的三倍还多2, 男孩几, 女孩几
: 男孩女孩一共十人, 男孩比女孩的4倍还多5, 男孩几, 女孩几
: 男孩女孩一共十人, 男孩比女孩的5倍还多4, 男孩几, 女孩几
: 难不成 每道题您都总结一个累死 “总数加上一个比另一个多的数初二”的办法让蛙明
: 白?让我我也不明白啊。
:
: 2times

s**********y
发帖数: 509
202
我老师的办法就是让蛙凑。 慢慢她比你还强。

【在 x***1 的大作中提到】
: 你如何给小孩子讲这些题,我也觉得我讲的别扭。
x***1
发帖数: 999
203
我想孩子大点,就该列算式,不用凑了,但又不用方程,比较难。

【在 s**********y 的大作中提到】
: 我老师的办法就是让蛙凑。 慢慢她比你还强。
B********e
发帖数: 10014
204
刚开始还是让孩子自己凑好
不要讲
讲的都是灌,感觉不好
我们家孩子记忆力好,讲的很多都能记住,
后来我发现这阻碍了孩子的思考力,不好

【在 x***1 的大作中提到】
: 你如何给小孩子讲这些题,我也觉得我讲的别扭。
B********e
发帖数: 10014
205
我也觉得自己有点云里雾里了
是看见人照的照片觉得好看给你看看
虽然有点像带着有色眼睛看世界
咱们小学毕业的都知道,唯有青春和有色段子不可辜负呀

【在 i**e 的大作中提到】
: 得,打住了,越说越说不到一起了
: 虽说a picture worth a thousand words
: 可小学毕业的我实在是看不懂啊,辜负了

i**e
发帖数: 19242
206
如果把多出的一个男生放在一边,那么总数是9,这个时候男生数目刚刚好是女生数目
的两倍
(总数是9,男生是女生数目的俩倍,这个会算吧?)
算好了,再把一边的男生加回去就好啦

【在 x***1 的大作中提到】
: 你如何给小孩子讲这些题,我也觉得我讲的别扭。
i**e
发帖数: 19242
207
这样啊,谢谢你!
还真是我喜欢的暖色调
而且是一天里我最喜欢的夕阳西下的时刻
这个时刻对我来说,象征着回家,温暖和劳作之后的安逸:)
以前看过一个渔舟唱晚的照片
很相似的感觉

【在 B********e 的大作中提到】
: 我也觉得自己有点云里雾里了
: 是看见人照的照片觉得好看给你看看
: 虽然有点像带着有色眼睛看世界
: 咱们小学毕业的都知道,唯有青春和有色段子不可辜负呀

d****g
发帖数: 7460
208
凑是综合。算式是分析。
娃凑出来是第一步,但着实不难。尤其是小数。
男加女100,还差不多。
会列算式要花些功夫。

【在 B********e 的大作中提到】
: 刚开始还是让孩子自己凑好
: 不要讲
: 讲的都是灌,感觉不好
: 我们家孩子记忆力好,讲的很多都能记住,
: 后来我发现这阻碍了孩子的思考力,不好

t******l
发帖数: 10908
209
我的办法是把 pre-algebra (包括鸡兔同笼、追击问题等等的 pre-algebra 解法)统
统扔给学校,我自己要么教 arithmetic(包括 arithmetic properties),要么直接
教 algebra 1。
原因很简单,因为 pre-algebra 这玩意儿我自己好像从来就不会,也不曾会过。。。
自己平时动扳手、开焊枪、大卸汽车点火系统的后院 handyman 都知道修玩意儿最基本
原则就是:自己不懂的玩意儿就根本不要去碰,否则一定漫天都是谁的幺蛾子在飞。。。

【在 s**********y 的大作中提到】
: 我老师的办法就是让蛙凑。 慢慢她比你还强。
t******l
发帖数: 10908
210
所以大伙儿看到俺从来不谈论四年级以下的娃版数学,真实的原因是我自己不会!

。。

【在 t******l 的大作中提到】
: 我的办法是把 pre-algebra (包括鸡兔同笼、追击问题等等的 pre-algebra 解法)统
: 统扔给学校,我自己要么教 arithmetic(包括 arithmetic properties),要么直接
: 教 algebra 1。
: 原因很简单,因为 pre-algebra 这玩意儿我自己好像从来就不会,也不曾会过。。。
: 自己平时动扳手、开焊枪、大卸汽车点火系统的后院 handyman 都知道修玩意儿最基本
: 原则就是:自己不懂的玩意儿就根本不要去碰,否则一定漫天都是谁的幺蛾子在飞。。。

相关主题
AMC 8 成绩 20167,8岁的女孩子暑假在家都干什么呢?
singapore math 到底怎么样啊?鸡兔同笼其实非常简单
给初中学生家长的一些建议 (转载)BSO: 儿子解射雕瑛姑九宫格
进入Parenting版参与讨论
d****g
发帖数: 7460
211
美国学校也是这么想的吧。大纲不会教吧?
等着直接上方程也是个思路。我LD就只列方程解。:)

。。

【在 t******l 的大作中提到】
: 我的办法是把 pre-algebra (包括鸡兔同笼、追击问题等等的 pre-algebra 解法)统
: 统扔给学校,我自己要么教 arithmetic(包括 arithmetic properties),要么直接
: 教 algebra 1。
: 原因很简单,因为 pre-algebra 这玩意儿我自己好像从来就不会,也不曾会过。。。
: 自己平时动扳手、开焊枪、大卸汽车点火系统的后院 handyman 都知道修玩意儿最基本
: 原则就是:自己不懂的玩意儿就根本不要去碰,否则一定漫天都是谁的幺蛾子在飞。。。

B********e
发帖数: 10014
212
我再给mm找几张

【在 i**e 的大作中提到】
: 这样啊,谢谢你!
: 还真是我喜欢的暖色调
: 而且是一天里我最喜欢的夕阳西下的时刻
: 这个时刻对我来说,象征着回家,温暖和劳作之后的安逸:)
: 以前看过一个渔舟唱晚的照片
: 很相似的感觉

t******l
发帖数: 10908
213
美国学校里 pre-algebra 从 4 年级开始教到 8 年级结束,如果是 normal track。。。

:美国学校也是这么想的吧。大纲不会教吧?
B********e
发帖数: 10014
214
大家适当休息一下哈
[在 BlackKnife (刀) 的大作中提到:]
:我再给mm找几张
B********e
发帖数: 10014
215
再来一组
注明:这是微博上叫 视界旅行家 的id发的
人引标题的确被你说中了一一浮云游子意 落日故人情
d****g
发帖数: 7460
216
男加女100,男比女的2倍还多1。要凑怎么个凑法?
如果女30,男61。还不够。
如果女31, 32,33。..... 对了!
这个如果做上100道,娃可能就自发上算式,不用教。那的确挺酷。但这小时数可得不
少。
如果一天一两道。娃很容易沾沾自喜,满足于凑,不思进取。家长憋不住就连启发带硬
罐,直接告诉题型答案了。
象高斯同学,会算1加到100,保不齐就是一天100道逼出来的。:)

【在 d****g 的大作中提到】
: 凑是综合。算式是分析。
: 娃凑出来是第一步,但着实不难。尤其是小数。
: 男加女100,还差不多。
: 会列算式要花些功夫。

d****g
发帖数: 7460
217
所以练凑就要狠。狠凑。比苦闷还要狠。我就是不够狠。又是钢琴又是出去玩儿,没空
。所以直接捷径
。奔着最中级去了。不高级。

【在 d****g 的大作中提到】
: 男加女100,男比女的2倍还多1。要凑怎么个凑法?
: 如果女30,男61。还不够。
: 如果女31, 32,33。..... 对了!
: 这个如果做上100道,娃可能就自发上算式,不用教。那的确挺酷。但这小时数可得不
: 少。
: 如果一天一两道。娃很容易沾沾自喜,满足于凑,不思进取。家长憋不住就连启发带硬
: 罐,直接告诉题型答案了。
: 象高斯同学,会算1加到100,保不齐就是一天100道逼出来的。:)

i**e
发帖数: 19242
218
谢谢,美不胜收啊!
这样好吗?在数学楼里大放彩照:)

【在 B********e 的大作中提到】
: 再来一组
: 注明:这是微博上叫 视界旅行家 的id发的
: 人引标题的确被你说中了一一浮云游子意 落日故人情

x***1
发帖数: 999
219
这个还真是不错。

【在 i**e 的大作中提到】
: 如果把多出的一个男生放在一边,那么总数是9,这个时候男生数目刚刚好是女生数目
: 的两倍
: (总数是9,男生是女生数目的俩倍,这个会算吧?)
: 算好了,再把一边的男生加回去就好啦

x***1
发帖数: 999
220
有道理。

【在 B********e 的大作中提到】
: 刚开始还是让孩子自己凑好
: 不要讲
: 讲的都是灌,感觉不好
: 我们家孩子记忆力好,讲的很多都能记住,
: 后来我发现这阻碍了孩子的思考力,不好

相关主题
混BBS要抱着贡献和学习的心Q. & A. With Liping Ma
老警察问个新问题什么时候给孩子引入方程概念
如何帮助有一定数学天赋的孩子别鸡兔同笼了,来喝啤酒吧
进入Parenting版参与讨论
x***1
发帖数: 999
221
你也太过分,四年级以下不会,四年级以上会。

【在 t******l 的大作中提到】
: 所以大伙儿看到俺从来不谈论四年级以下的娃版数学,真实的原因是我自己不会!
:
: 。。

B********e
发帖数: 10014
222
这么臭长的帖子,彩照是多好的放松机会啊
如果有人有意见再删呗

【在 i**e 的大作中提到】
: 谢谢,美不胜收啊!
: 这样好吗?在数学楼里大放彩照:)

x***1
发帖数: 999
223
这个算式比列方程要难,孩子到底要不要学习列这个算式?小时候,牛娃会列算式,但
到了列方程阶段,有些普通孩子冒上来了,有些牛娃就普通了。

【在 d****g 的大作中提到】
: 凑是综合。算式是分析。
: 娃凑出来是第一步,但着实不难。尤其是小数。
: 男加女100,还差不多。
: 会列算式要花些功夫。

h*****m
发帖数: 1034
224
汇报一下刚刚跟国内我爸爸通电话听来的:一亲戚孩子,一年级,留的数学作业里有这
么一道:
小明和小红去书店买书,看好了一本,小明发现自己还差2块钱够买这本书的,小红发
现自己差3块钱,然后他们发现把两人的钱合在一起,刚刚好可以买下这本书,问这本
书多少钱?
问了一下我家三年级的老大,他倒是猜出答案来了,但是不能给出一个正规的解法。大
家以为对一年级孩子来说,这题难度如何?
t******l
发帖数: 10908
225
那。。。哥们这个要每天多少遍才能逼出来?

:男加女100,男比女的2倍还多1。要凑怎么个凑法?
d****g
发帖数: 7460
226
不可能的了。会列算式的娃才会画辅助性。
所以小学算术和平几才天下最难。
当然了,这两项都没多大用是真的。
但总不至于会算式的就学不会方程了。
牛娃变普通的可能是超细心超死记的普通娃。也是异数。

【在 x***1 的大作中提到】
: 这个算式比列方程要难,孩子到底要不要学习列这个算式?小时候,牛娃会列算式,但
: 到了列方程阶段,有些普通孩子冒上来了,有些牛娃就普通了。

t******l
发帖数: 10908
227
天下最没用的当然是天下最难的。。。比如一个例子就是自宫技术,估计是天下最最最
难,比平几难多了。。。

【在 d****g 的大作中提到】
: 不可能的了。会列算式的娃才会画辅助性。
: 所以小学算术和平几才天下最难。
: 当然了,这两项都没多大用是真的。
: 但总不至于会算式的就学不会方程了。
: 牛娃变普通的可能是超细心超死记的普通娃。也是异数。

h*****m
发帖数: 1034
228
到底高斯花了多长时间搞出这个超级牛叉的玩意的?
是先找到一般规律然后应用到17边形,还是先是硬搞定17边形,然后发现一般规律的?
如果是前者,至少知道能画出来,还有个确定的目标。

【在 t******l 的大作中提到】
: 那。。。哥们这个要每天多少遍才能逼出来?
:
: :男加女100,男比女的2倍还多1。要凑怎么个凑法?
: :

d****g
发帖数: 7460
229
收藏!

【在 t******l 的大作中提到】
: 那。。。哥们这个要每天多少遍才能逼出来?
:
: :男加女100,男比女的2倍还多1。要凑怎么个凑法?
: :

t******l
发帖数: 10908
230
我不是说了多少年了,我小学数学被红叉追屁股。。。你说我要是会四年级以下小学数
学,我为啥要主动乐意让红叉追屁股啊?。。。我再傻也不至于傻到主动乐意让红叉追
屁股。。。

【在 x***1 的大作中提到】
: 你也太过分,四年级以下不会,四年级以上会。
相关主题
9岁, 如何报名考amc10代数小朋友不理解怎么办?
求科普: 奥数 vs. Math Olympiad vs. Math Count vs. Math circle vs. 超前学数学教绝对值方程:Khan Academy vs AMC10 答案 vs 分段函数替换?
分数应用题系统学习的反面是什么?
进入Parenting版参与讨论
t******l
发帖数: 10908
231
我带的研究生里面没有姓高名斯的,有一个姓高的但名叫圆圆。 -- 黑刀

【在 h*****m 的大作中提到】
: 到底高斯花了多长时间搞出这个超级牛叉的玩意的?
: 是先找到一般规律然后应用到17边形,还是先是硬搞定17边形,然后发现一般规律的?
: 如果是前者,至少知道能画出来,还有个确定的目标。

d****g
发帖数: 7460
232
的确,就是些雕虫小技。练练脑子。就当是另版智力题。的确不宜多做。也不宜认为是
多高的境界。
高境界的还真是美国这种教法。可以不会鸡兔同笼算数解,不耽误高中就学微积分。
对大概念的掌握的确是高考教育的一个弊端。

【在 t******l 的大作中提到】
: 天下最没用的当然是天下最难的。。。比如一个例子就是自宫技术,估计是天下最最最
: 难,比平几难多了。。。

d****g
发帖数: 7460
233
所以啥是更牛X的数学呢?怎么培养娃更牛X的素质呢?
真得从小就群环域?或者数论?或者微积分?我这只会四年级以下的,不会四年级以上
的,更可肿么办呢?

【在 d****g 的大作中提到】
: 的确,就是些雕虫小技。练练脑子。就当是另版智力题。的确不宜多做。也不宜认为是
: 多高的境界。
: 高境界的还真是美国这种教法。可以不会鸡兔同笼算数解,不耽误高中就学微积分。
: 对大概念的掌握的确是高考教育的一个弊端。

h*****m
发帖数: 1034
234
唉,求人不如求己,自己动手搜了搜中文网站,找到以下结果:
整理以上内容,我认为结论是这样的:(1)“导师不小心给了高斯学术难题,结果被
不明就里的高斯一晚上刷掉……”这个故事是假的。(2)高斯于1796年19岁时证明怎
样的正多边形可以用尺规作出来,并发表研究成果,轰动学术界。1796年3月30日,他
在笔记中写下正十七边形作法,但没有发表(的确,相较证明怎样的正多边形可以用尺
规作出来,这一成果微不足道)。1825年约翰尼斯·厄钦格第一次公开发表正十七边形
作法。1898年整理高斯遗物时发现笔记中高斯的正十七边形作法。(3)所以说,高斯
会尺规作正十七边形,而约翰尼斯·厄钦格最早给出(即发表)正十七边形作法。
http://www.guokr.com/post/620415/
看来,是先搞定了更更更高级牛叉的规律,然后顺手把17边形给做了,甚至都不稀得发
表。我们草民还在这里膜拜,唉!。。。

【在 t******l 的大作中提到】
: 我带的研究生里面没有姓高名斯的,有一个姓高的但名叫圆圆。 -- 黑刀
x***1
发帖数: 999
235
一天100道逼出来的不是高斯,另有其人。
高斯这个典故家喻户晓,还有另外一个大名人欧啦的典故,好像没人提:
说欧拉小时候老师给出了相同的题目,从1加到100,欧拉也是第一个举手,老师问他咋
算的,欧拉说我就这么1+2+3一直加到100加起来的,老师又问,同学都这么做,为什么
你最快,欧拉说天天在做加法,所以比他们快。后来老师把高斯的方法讲了一遍,同学
称赞不已,欧拉觉得不可思议,认为这么简单的加法,没必要用高深的方法去做。
有好事者做个对比,说从1加到10^6,欧拉比高斯快。10^6以上,高斯稍微快点。
真正的事实是,欧拉晚年双目失明,心算四位数开放,三角函数不在话下。

【在 d****g 的大作中提到】
: 男加女100,男比女的2倍还多1。要凑怎么个凑法?
: 如果女30,男61。还不够。
: 如果女31, 32,33。..... 对了!
: 这个如果做上100道,娃可能就自发上算式,不用教。那的确挺酷。但这小时数可得不
: 少。
: 如果一天一两道。娃很容易沾沾自喜,满足于凑,不思进取。家长憋不住就连启发带硬
: 罐,直接告诉题型答案了。
: 象高斯同学,会算1加到100,保不齐就是一天100道逼出来的。:)

t******l
发帖数: 10908
236
中小学数学首先是实用,我觉得 arithmetic properties 是一个比较好的折中。


:所以啥是更牛X的数学呢?怎么培养娃更牛X的素质呢?
h*****m
发帖数: 1034
237
欧拉应该去参加最强大脑!

【在 x***1 的大作中提到】
: 一天100道逼出来的不是高斯,另有其人。
: 高斯这个典故家喻户晓,还有另外一个大名人欧啦的典故,好像没人提:
: 说欧拉小时候老师给出了相同的题目,从1加到100,欧拉也是第一个举手,老师问他咋
: 算的,欧拉说我就这么1+2+3一直加到100加起来的,老师又问,同学都这么做,为什么
: 你最快,欧拉说天天在做加法,所以比他们快。后来老师把高斯的方法讲了一遍,同学
: 称赞不已,欧拉觉得不可思议,认为这么简单的加法,没必要用高深的方法去做。
: 有好事者做个对比,说从1加到10^6,欧拉比高斯快。10^6以上,高斯稍微快点。
: 真正的事实是,欧拉晚年双目失明,心算四位数开放,三角函数不在话下。

t******l
发帖数: 10908
238
属实。。。其实很多证明主要是一个思路草稿,剩下的都是体力活。。。如果不是因为
世人看不懂高斯思路草稿,高斯估计证明过程都懒得写。。。

:唉,求人不如求己,自己动手搜了搜中文网站,找到以下结果:
d****g
发帖数: 7460
239
看来得人脑加电脑,算法加处理器速度。
我娃就是处理器速度不灵。算个78+89都慢条斯理的。

【在 x***1 的大作中提到】
: 一天100道逼出来的不是高斯,另有其人。
: 高斯这个典故家喻户晓,还有另外一个大名人欧啦的典故,好像没人提:
: 说欧拉小时候老师给出了相同的题目,从1加到100,欧拉也是第一个举手,老师问他咋
: 算的,欧拉说我就这么1+2+3一直加到100加起来的,老师又问,同学都这么做,为什么
: 你最快,欧拉说天天在做加法,所以比他们快。后来老师把高斯的方法讲了一遍,同学
: 称赞不已,欧拉觉得不可思议,认为这么简单的加法,没必要用高深的方法去做。
: 有好事者做个对比,说从1加到10^6,欧拉比高斯快。10^6以上,高斯稍微快点。
: 真正的事实是,欧拉晚年双目失明,心算四位数开放,三角函数不在话下。

t******l
发帖数: 10908
240
很多初中参加 AMC 10 的,state test 也就是好学区中等偏上,所以看从哪个角度说。

:不可能的了。会列算式的娃才会画辅助性。
:所以小学算术和平几才天下最难。
相关主题
系统学习的反面是什么?singapore math 到底怎么样啊?
有些小学四年级的数学题不太会做给初中学生家长的一些建议 (转载)
AMC 8 成绩 20167,8岁的女孩子暑假在家都干什么呢?
进入Parenting版参与讨论
x***1
发帖数: 999
241
这样难度很高。
等于说,小明加小红的钱,买一本正好,买2本差5块,为啥差钱,多买了一本,也就是
说差的钱正好买一本。

【在 h*****m 的大作中提到】
: 汇报一下刚刚跟国内我爸爸通电话听来的:一亲戚孩子,一年级,留的数学作业里有这
: 么一道:
: 小明和小红去书店买书,看好了一本,小明发现自己还差2块钱够买这本书的,小红发
: 现自己差3块钱,然后他们发现把两人的钱合在一起,刚刚好可以买下这本书,问这本
: 书多少钱?
: 问了一下我家三年级的老大,他倒是猜出答案来了,但是不能给出一个正规的解法。大
: 家以为对一年级孩子来说,这题难度如何?

i**e
发帖数: 19242
242
小明的钱,补上了小红差的,3块
小红的钱,补上了小明差的,2块
所以一共5块
r*g
发帖数: 3159
243
应用题不该多练。
凡是专门教鸡兔问题的,追及问题的,甚至前面还有个教两个数求和的,都是教太多了。
说是锻炼素质,其实都成记题型了。
t******l
发帖数: 10908
244
老农的说法,总体原则是不该用勺喂 finger food。。。当然具体情况具体分析。。。

:应用题不该多练。
:凡是专门教鸡兔问题的,追及问题的,甚至前面还有个教两个数求和的,都是教太多
了。
s**********y
发帖数: 509
245
其实国内人也烦这个。 关键是没法教。 鸡兔背了, 鸡鸭同笼, 鸡蜘蛛同笼, 鸡蛇
同笼, 鸡蜈蚣同笼, 叫不叫?
兔鸭同笼, 兔蜘蛛同笼, 兔蛇同笼, 兔蜈蚣同笼, 叫不叫? (七七八八同笼好像
会有问题哈?)
蜘蛛蛇同笼,蜘蛛蜈蚣同笼, 叫不叫?
一般来说, 凡事热衷于此的, 老师属于勤奋但水平差的, 是老师中最差的一类。

了。

【在 r*g 的大作中提到】
: 应用题不该多练。
: 凡是专门教鸡兔问题的,追及问题的,甚至前面还有个教两个数求和的,都是教太多了。
: 说是锻炼素质,其实都成记题型了。

s**********y
发帖数: 509
246
最近好像是少了。

【在 t******l 的大作中提到】
: 老实说,现实生活中这种人基本没有。就好比现实生活中就基本没有狠推才艺钢琴走火
: 入魔的。。。狠推钢琴的基本娃一听就是货真价实 gifted 的那种。。。
: 但网上不一样啊,网上从前这里钢琴智商、钢琴数学、钢琴万金油大力丸的帖子还少么
: ?。。。所以上网就快意恩仇一下也是灌水的一种方式不是?
:
: :你是认真的吗?
: :我在现实生活在网上,都没有遇见过你说的这种人

t******l
发帖数: 10908
247
我刚才想了想你首贴说的那个教师的例子。。。我现在觉得他可能是赢在数学教育方法
学层面,而不是具体数学教育内容技巧层面。具体这么说:
我觉得你首贴那位教师:在数学教育方法战略上,可能是赢在 “不用勺子喂娃 finger
food”;而在数学教育方法战术方面,可能是赢在 “在教师概略引导下让娃乱中取胜、
激发兴趣”。
话说回来,在具体数学教育内容技巧方面,在小学四年级阶段,我个人觉得是应该侧重
于 “arithmetic properties” 以及 “娃版 foundation of mathematics -- half-
naive half-formal set theory concept for elementary school kids”。。。但这
边有个问题是,这个是四年级以上的数学内容。。。而我只教自家两娃,而且是一对一
的教育。而你首贴那位教师啥娃进班级都得教。。。既然世上有我这种四年级以下数学
根本不会,四年级以下解题基本靠猜的这种。那搞不好也有四年级以上数学基本不会的
那型。。。从这个角度看,教师比家长更侧重数学教育方法学,而不是具体数学教育内
容,确实有其切实的现实依据。。。

了。

【在 s**********y 的大作中提到】
: 其实国内人也烦这个。 关键是没法教。 鸡兔背了, 鸡鸭同笼, 鸡蜘蛛同笼, 鸡蛇
: 同笼, 鸡蜈蚣同笼, 叫不叫?
: 兔鸭同笼, 兔蜘蛛同笼, 兔蛇同笼, 兔蜈蚣同笼, 叫不叫? (七七八八同笼好像
: 会有问题哈?)
: 蜘蛛蛇同笼,蜘蛛蜈蚣同笼, 叫不叫?
: 一般来说, 凡事热衷于此的, 老师属于勤奋但水平差的, 是老师中最差的一类。
:
: 了。

t******l
发帖数: 10908
248
另外从这个角度看,common core 的 explanation 对于 8 年级以下,还是可能有些
问题的。如果遇到四年级以下数学不会、解题基本靠猜的那型,那还要 common core
解释就不是更郁闷了?

finger
胜、
了。

【在 t******l 的大作中提到】
: 我刚才想了想你首贴说的那个教师的例子。。。我现在觉得他可能是赢在数学教育方法
: 学层面,而不是具体数学教育内容技巧层面。具体这么说:
: 我觉得你首贴那位教师:在数学教育方法战略上,可能是赢在 “不用勺子喂娃 finger
: food”;而在数学教育方法战术方面,可能是赢在 “在教师概略引导下让娃乱中取胜、
: 激发兴趣”。
: 话说回来,在具体数学教育内容技巧方面,在小学四年级阶段,我个人觉得是应该侧重
: 于 “arithmetic properties” 以及 “娃版 foundation of mathematics -- half-
: naive half-formal set theory concept for elementary school kids”。。。但这
: 边有个问题是,这个是四年级以上的数学内容。。。而我只教自家两娃,而且是一对一
: 的教育。而你首贴那位教师啥娃进班级都得教。。。既然世上有我这种四年级以下数学

B********e
发帖数: 10014
249
这种题真觉得没有必要深入研究到这种成都
太简单的题知道一个基本解法就够了,再多花样都是后边总结的
gut感觉从哪里开始更重要
比较深的题才更能锻炼思维的穿透力
家长们没有必要在这个方面比较
h*****m
发帖数: 1034
250
突然发现这是一钓鱼贴。好吧,我上当了。
莱昂哈德·欧拉(Leonhard Euler,1707年4月15日-1783年9月18日)
约翰·卡尔·弗里德里希·高斯(Carolus Fridericus Gauss;1777年4月30日-1855
年2月23日)
欧拉去世时,高斯才6岁。。。

【在 h*****m 的大作中提到】
: 欧拉应该去参加最强大脑!
相关主题
鸡兔同笼其实非常简单老警察问个新问题
BSO: 儿子解射雕瑛姑九宫格如何帮助有一定数学天赋的孩子
混BBS要抱着贡献和学习的心Q. & A. With Liping Ma
进入Parenting版参与讨论
x***1
发帖数: 999
251
说的是高斯的方法,不是指高斯本人。人家胡诌也不至于这么明显不讲理。

1855

【在 h*****m 的大作中提到】
: 突然发现这是一钓鱼贴。好吧,我上当了。
: 莱昂哈德·欧拉(Leonhard Euler,1707年4月15日-1783年9月18日)
: 约翰·卡尔·弗里德里希·高斯(Carolus Fridericus Gauss;1777年4月30日-1855
: 年2月23日)
: 欧拉去世时,高斯才6岁。。。

h*****m
发帖数: 1034
252
我也不知道这道题的要求是什么。或许很多孩子能猜出答案来,可是如果要求说出来个
道理,有点太弯弯绕了吧,才是一年级的孩子啊。估计没几个能像二位这样简单扼要地
解释清楚。
也许中国的孩子从小就经受这样的训练,稍大点解个鸡兔同笼就不当回事儿了。

【在 x***1 的大作中提到】
: 这样难度很高。
: 等于说,小明加小红的钱,买一本正好,买2本差5块,为啥差钱,多买了一本,也就是
: 说差的钱正好买一本。

h*****m
发帖数: 1034
253
呵呵,我语文就没及格过,您别见怪。

【在 x***1 的大作中提到】
: 说的是高斯的方法,不是指高斯本人。人家胡诌也不至于这么明显不讲理。
:
: 1855

t******l
发帖数: 10908
254
我先问是不是四年级以下数学?

:我也不知道这道题的要求是什么。或许很多孩子能猜出答案来,可是如果要求说出来
个道理,有点太弯弯绕了吧,才是一年级的孩子啊。估计没几个能像二位这样简单扼要
地解释清楚。
h*****m
发帖数: 1034
255
这是四年级以下数学,你不懂的!: )

【在 t******l 的大作中提到】
: 我先问是不是四年级以下数学?
:
: :我也不知道这道题的要求是什么。或许很多孩子能猜出答案来,可是如果要求说出来
: 个道理,有点太弯弯绕了吧,才是一年级的孩子啊。估计没几个能像二位这样简单扼要
: 地解释清楚。
: :

t******l
发帖数: 10908
256
属实

:这是四年级以下数学,你不懂的!: )
x***1
发帖数: 999
257
izze解释的太明白不过,太简单直白,令人恍然大悟,拍案叫绝,一点也不绕弯。我的
解释属于绕弯型的,别扭型的,没意思。
izze的解释等于说:小明手里只有小红差的钱,小红手里只有小明差的钱,两个人的钱
相加,就是书本的钱。

【在 h*****m 的大作中提到】
: 我也不知道这道题的要求是什么。或许很多孩子能猜出答案来,可是如果要求说出来个
: 道理,有点太弯弯绕了吧,才是一年级的孩子啊。估计没几个能像二位这样简单扼要地
: 解释清楚。
: 也许中国的孩子从小就经受这样的训练,稍大点解个鸡兔同笼就不当回事儿了。

h*****m
发帖数: 1034
258
对于我们成人来说,都不算很绕。对于一年级孩子来说,就不容易了。izze的解释可能
小孩子更容易理解一些。

【在 x***1 的大作中提到】
: izze解释的太明白不过,太简单直白,令人恍然大悟,拍案叫绝,一点也不绕弯。我的
: 解释属于绕弯型的,别扭型的,没意思。
: izze的解释等于说:小明手里只有小红差的钱,小红手里只有小明差的钱,两个人的钱
: 相加,就是书本的钱。

i**e
发帖数: 19242
259
要不要拿这些题去给孩子做?
教不教?教什么?怎教?
做题的目的是什么?
自己的孩子不如别人的孩子的时候,会怎么feel怎么想?
这是知己
孩子对这样的要求会有什么反应?
孩子的problem solving 有多少?
孩子能focus的时间有多少?
孩子的接受能力如何?
什么样的方式方法孩子更容易接受?
这是知彼
forget about 祖国的要求,那个不重要

【在 h*****m 的大作中提到】
: 我也不知道这道题的要求是什么。或许很多孩子能猜出答案来,可是如果要求说出来个
: 道理,有点太弯弯绕了吧,才是一年级的孩子啊。估计没几个能像二位这样简单扼要地
: 解释清楚。
: 也许中国的孩子从小就经受这样的训练,稍大点解个鸡兔同笼就不当回事儿了。

t******l
发帖数: 10908
260
你说的 “知己知彼” 确实是娃版数学教育方面很重要的考虑角度。
从你说的这个 “知己知彼” 考虑角度看,对于普通娃而言,鸡兔同笼或者欧拉定理都
不是首要的主攻方向。
原因是看一下 AMC 10 statistics “知己知彼” 一下,就很容易明白。。。我个人认
为普通娃首先是 AMC 10 能够拿到 100 分,因为这对大部分普通娃是个坎。。。而欧
拉定理啥的,普通娃等 AMC 10 拿到 100 分以后在搞,也不至于就会输在欧拉的起跑
线上。。。而对于鸡兔同笼,如果 AMC 10 的目标是拿个 60 分的时候,鸡兔同笼根本
不需要要会就能拿到。。。而如果 AMC 10 能拿到 100 分,那随便算个 simultaneous
equation 的 algebra 1 word problem,鸡兔同笼也就搞定了。。。

【在 i**e 的大作中提到】
: 要不要拿这些题去给孩子做?
: 教不教?教什么?怎教?
: 做题的目的是什么?
: 自己的孩子不如别人的孩子的时候,会怎么feel怎么想?
: 这是知己
: 孩子对这样的要求会有什么反应?
: 孩子的problem solving 有多少?
: 孩子能focus的时间有多少?
: 孩子的接受能力如何?
: 什么样的方式方法孩子更容易接受?

相关主题
什么时候给孩子引入方程概念求科普: 奥数 vs. Math Olympiad vs. Math Count vs. Math circle vs. 超前学数学
别鸡兔同笼了,来喝啤酒吧分数应用题
9岁, 如何报名考amc10代数小朋友不理解怎么办?
进入Parenting版参与讨论
t******l
发帖数: 10908
261
另一方面,现代数学的证明真是基于无数棋谱。。。如果不是理论数学技校毕业的,根
本就不知道那些棋谱。。。但另一方面 3-SAT problem 是 NP problem 没跑,我觉得
基本不可能有办法快速找到 3-SAT 的 proof path。。。于是我猜想未来实际的数学证
明可能要靠阿发猫(AlphaMathOlympiad,简称 AlphaMo),跟阿发狗九段一样,学习
了人类的棋谱以后,在特定棋局的 3-SAT NP problem 里连蒙带猜到处落子找到 proof
path。。。当然阿发猫心中的可能是一大张 3-SAT 表达式型 19k*19k 的棋盘里找大龙
。。。

simultaneous

【在 t******l 的大作中提到】
: 你说的 “知己知彼” 确实是娃版数学教育方面很重要的考虑角度。
: 从你说的这个 “知己知彼” 考虑角度看,对于普通娃而言,鸡兔同笼或者欧拉定理都
: 不是首要的主攻方向。
: 原因是看一下 AMC 10 statistics “知己知彼” 一下,就很容易明白。。。我个人认
: 为普通娃首先是 AMC 10 能够拿到 100 分,因为这对大部分普通娃是个坎。。。而欧
: 拉定理啥的,普通娃等 AMC 10 拿到 100 分以后在搞,也不至于就会输在欧拉的起跑
: 线上。。。而对于鸡兔同笼,如果 AMC 10 的目标是拿个 60 分的时候,鸡兔同笼根本
: 不需要要会就能拿到。。。而如果 AMC 10 能拿到 100 分,那随便算个 simultaneous
: equation 的 algebra 1 word problem,鸡兔同笼也就搞定了。。。

i**e
发帖数: 19242
262
AMC10,几年级拿100分?100分是满分吗?
AMC8呢?
还是你意思,前面的送分题要能掌握?
真正的普通孩子
小学,是正确完整地掌握数学概念,一定程度的熟练运算和基本的运用
初中高中
要完成和完善从实体实物到抽象符号的质跃
学习和习惯用“数学语言”来思考和解决数学或者实际问题
学习和习惯推理论证
这种逻辑思考辩证思维的习惯和能力,不是生而有之的
通过学习数学知识培养锻炼而已
AMC等等都是数学竞赛,普通孩子玩不起的
自己的孩子是啥,自己心里有数就中

simultaneous

【在 t******l 的大作中提到】
: 你说的 “知己知彼” 确实是娃版数学教育方面很重要的考虑角度。
: 从你说的这个 “知己知彼” 考虑角度看,对于普通娃而言,鸡兔同笼或者欧拉定理都
: 不是首要的主攻方向。
: 原因是看一下 AMC 10 statistics “知己知彼” 一下,就很容易明白。。。我个人认
: 为普通娃首先是 AMC 10 能够拿到 100 分,因为这对大部分普通娃是个坎。。。而欧
: 拉定理啥的,普通娃等 AMC 10 拿到 100 分以后在搞,也不至于就会输在欧拉的起跑
: 线上。。。而对于鸡兔同笼,如果 AMC 10 的目标是拿个 60 分的时候,鸡兔同笼根本
: 不需要要会就能拿到。。。而如果 AMC 10 能拿到 100 分,那随便算个 simultaneous
: equation 的 algebra 1 word problem,鸡兔同笼也就搞定了。。。

t******l
发帖数: 10908
263
AMC 10 满分 150 分,总共 25 题。做对一题得 6 分,做错一题的 0 分,留白不做的
1.5 分。
至于大部分普通娃卡在哪个位置,我觉得不是教育学专业出来的父母,不应该靠目测是
不是送分题(祖国高考过关的可能目测 AMC 10 统统都是送分题!!),因为毕竟父母
要回到自己小学时的感觉也不太可能。。。我觉得 statistical data 更有说服力,
AMC statistics 在 AoPS 网站上可以找到。。。

【在 i**e 的大作中提到】
: AMC10,几年级拿100分?100分是满分吗?
: AMC8呢?
: 还是你意思,前面的送分题要能掌握?
: 真正的普通孩子
: 小学,是正确完整地掌握数学概念,一定程度的熟练运算和基本的运用
: 初中高中
: 要完成和完善从实体实物到抽象符号的质跃
: 学习和习惯用“数学语言”来思考和解决数学或者实际问题
: 学习和习惯推理论证
: 这种逻辑思考辩证思维的习惯和能力,不是生而有之的

i**e
发帖数: 19242
264
拿到statistical data又如何?
还不是孩子能考多少就是多少
难道还能拿着data跟孩子过不去么?

【在 t******l 的大作中提到】
: AMC 10 满分 150 分,总共 25 题。做对一题得 6 分,做错一题的 0 分,留白不做的
: 1.5 分。
: 至于大部分普通娃卡在哪个位置,我觉得不是教育学专业出来的父母,不应该靠目测是
: 不是送分题(祖国高考过关的可能目测 AMC 10 统统都是送分题!!),因为毕竟父母
: 要回到自己小学时的感觉也不太可能。。。我觉得 statistical data 更有说服力,
: AMC statistics 在 AoPS 网站上可以找到。。。

t******l
发帖数: 10908
265
statistical data 是用来 look forward,plan forward,make prediction 和
make educated guess 用的。。。具体而言,比如父母自己看第 1 题和第 12 题都是
送分题,这是就要看看 statistical data 有多少娃能做对 3 题,有多少娃能做对 12
题,然后想想可能的原因,在问问自己娃能够做对/能够下笔/能够看懂答案/能够听懂
解释的题目的 distribution 大张概略战略形势图,这样对在哪里选点海空直降更有
感觉不是?。。。
而且更重要的是,这样父母对娃现在/将来的分数也不会有 surprise,也就更不容易受到
sporadic-spontaneous peer-pressure generator(简称:微信)的影响。。。
古人云:心平气和家事兴。。。

【在 i**e 的大作中提到】
: 拿到statistical data又如何?
: 还不是孩子能考多少就是多少
: 难道还能拿着data跟孩子过不去么?

t******l
发帖数: 10908
266
另外看过 AMC 10 statistics 以后,也就能切身体会到网上讨论的那些比如:
“鸡兔同笼/追击问题/欧拉定理/黎曼猜想” 之类的 “拉风题”,其实也没多少
普通娃能做对。。。所以自家普通娃做不出来,也不影响俺们自家普通娃吃晚饭、
上州立不是?。。。这不用鸡汤都有实打实的自信,岂不甚佳?。。。

12
受到

【在 t******l 的大作中提到】
: statistical data 是用来 look forward,plan forward,make prediction 和
: make educated guess 用的。。。具体而言,比如父母自己看第 1 题和第 12 题都是
: 送分题,这是就要看看 statistical data 有多少娃能做对 3 题,有多少娃能做对 12
: 题,然后想想可能的原因,在问问自己娃能够做对/能够下笔/能够看懂答案/能够听懂
: 解释的题目的 distribution 大张概略战略形势图,这样对在哪里选点海空直降更有
: 感觉不是?。。。
: 而且更重要的是,这样父母对娃现在/将来的分数也不会有 surprise,也就更不容易受到
: sporadic-spontaneous peer-pressure generator(简称:微信)的影响。。。
: 古人云:心平气和家事兴。。。

s***n
发帖数: 1280
267
我觉得数学概念的学习是贯穿小初高的。
我觉得对普通娃来说小学初中就要“完成和完善从实体实物到抽象符号的质跃”,“学
习和习惯用数学语言来思考和解决数学或者实际问题”,越早越好。像Common core那
样细水长流,慢条斯理的教,对很多孩子是不太好的。
初中高中阶段普通娃要“学习和习惯推理论证”,培养“逻辑思考辩证思维的习惯和能
力”。这个能力不是生而有之的,但能力区间是生而定之。 有些人逻辑思维发育得早
,有些人发育得晚;成熟后,有些人逻辑思维强,有些人逻辑思维弱。

【在 i**e 的大作中提到】
: AMC10,几年级拿100分?100分是满分吗?
: AMC8呢?
: 还是你意思,前面的送分题要能掌握?
: 真正的普通孩子
: 小学,是正确完整地掌握数学概念,一定程度的熟练运算和基本的运用
: 初中高中
: 要完成和完善从实体实物到抽象符号的质跃
: 学习和习惯用“数学语言”来思考和解决数学或者实际问题
: 学习和习惯推理论证
: 这种逻辑思考辩证思维的习惯和能力,不是生而有之的

s***n
发帖数: 1280
268
我觉得数学概念的学习是贯穿小初高的。
我觉得对普通娃来说小学初中就要“完成和完善从实体实物到抽象符号的质跃”,“学
习和习惯用数学语言来思考和解决数学或者实际问题”,越早越好。像Common core那
样细水长流,慢条斯理的教,对很多孩子是不太好的。
初中高中阶段普通娃要“学习和习惯推理论证”,培养“逻辑思考辩证思维的习惯和能
力”。这个能力不是生而有之的,但能力区间是生而定之。 有些人逻辑思维发育得早
,有些人发育得晚;成熟后,有些人逻辑思维强,有些人逻辑思维弱。

【在 i**e 的大作中提到】
: AMC10,几年级拿100分?100分是满分吗?
: AMC8呢?
: 还是你意思,前面的送分题要能掌握?
: 真正的普通孩子
: 小学,是正确完整地掌握数学概念,一定程度的熟练运算和基本的运用
: 初中高中
: 要完成和完善从实体实物到抽象符号的质跃
: 学习和习惯用“数学语言”来思考和解决数学或者实际问题
: 学习和习惯推理论证
: 这种逻辑思考辩证思维的习惯和能力,不是生而有之的

i**e
发帖数: 19242
269
有这时间还不如想想孩子为什么做不了某道题
问题出在哪里呢

12
受到

【在 t******l 的大作中提到】
: statistical data 是用来 look forward,plan forward,make prediction 和
: make educated guess 用的。。。具体而言,比如父母自己看第 1 题和第 12 题都是
: 送分题,这是就要看看 statistical data 有多少娃能做对 3 题,有多少娃能做对 12
: 题,然后想想可能的原因,在问问自己娃能够做对/能够下笔/能够看懂答案/能够听懂
: 解释的题目的 distribution 大张概略战略形势图,这样对在哪里选点海空直降更有
: 感觉不是?。。。
: 而且更重要的是,这样父母对娃现在/将来的分数也不会有 surprise,也就更不容易受到
: sporadic-spontaneous peer-pressure generator(简称:微信)的影响。。。
: 古人云:心平气和家事兴。。。

t******l
发帖数: 10908
270
属实

【在 s***n 的大作中提到】
: 我觉得数学概念的学习是贯穿小初高的。
: 我觉得对普通娃来说小学初中就要“完成和完善从实体实物到抽象符号的质跃”,“学
: 习和习惯用数学语言来思考和解决数学或者实际问题”,越早越好。像Common core那
: 样细水长流,慢条斯理的教,对很多孩子是不太好的。
: 初中高中阶段普通娃要“学习和习惯推理论证”,培养“逻辑思考辩证思维的习惯和能
: 力”。这个能力不是生而有之的,但能力区间是生而定之。 有些人逻辑思维发育得早
: ,有些人发育得晚;成熟后,有些人逻辑思维强,有些人逻辑思维弱。

相关主题
代数小朋友不理解怎么办?有些小学四年级的数学题不太会做
教绝对值方程:Khan Academy vs AMC10 答案 vs 分段函数替换?AMC 8 成绩 2016
系统学习的反面是什么?singapore math 到底怎么样啊?
进入Parenting版参与讨论
i**e
发帖数: 19242
271
“拉风题”别人都能做出来,或者别人都做不出来
对我要不要让孩子试试这些题
没有影响

【在 t******l 的大作中提到】
: 另外看过 AMC 10 statistics 以后,也就能切身体会到网上讨论的那些比如:
: “鸡兔同笼/追击问题/欧拉定理/黎曼猜想” 之类的 “拉风题”,其实也没多少
: 普通娃能做对。。。所以自家普通娃做不出来,也不影响俺们自家普通娃吃晚饭、
: 上州立不是?。。。这不用鸡汤都有实打实的自信,岂不甚佳?。。。
:
: 12
: 受到

t******l
发帖数: 10908
272
战略和战术都重要吧。。。而且很多时候为啥做不了某题是归娃想而不是归父母想。。
。归父母想的常常是statistically 把难度设在哪里、把要求设多高,而不是拘泥于某
一道题。。。对于单一的某一道题,陶天才说了:“做好题,不求甚解,就完事了。。
。”

【在 i**e 的大作中提到】
: 有这时间还不如想想孩子为什么做不了某道题
: 问题出在哪里呢
:
: 12
: 受到

t******l
发帖数: 10908
273
但微信上刚才疯传说,“拉分题” 做不出来的话,那将来连 community college 都上
不了,只能去麦当劳翻烧饼 。。。那咋办?。。。

【在 i**e 的大作中提到】
: “拉风题”别人都能做出来,或者别人都做不出来
: 对我要不要让孩子试试这些题
: 没有影响

t******l
发帖数: 10908
274
另外不要说微信了,本版以前不是一直疯传,不学钢琴输在起跑线上的话,将来数学一
定不及格。。。您就说咋办吧。。。

【在 t******l 的大作中提到】
: 但微信上刚才疯传说,“拉分题” 做不出来的话,那将来连 community college 都上
: 不了,只能去麦当劳翻烧饼 。。。那咋办?。。。

i**e
发帖数: 19242
275
当然,不同的概念罢了
当然越早越好,可惜不是每个孩子都早早ready的了
CC的问题,是太注重所谓的理解了,练习量不跟上,能悟能真正理解进而应用的是少数

【在 s***n 的大作中提到】
: 我觉得数学概念的学习是贯穿小初高的。
: 我觉得对普通娃来说小学初中就要“完成和完善从实体实物到抽象符号的质跃”,“学
: 习和习惯用数学语言来思考和解决数学或者实际问题”,越早越好。像Common core那
: 样细水长流,慢条斯理的教,对很多孩子是不太好的。
: 初中高中阶段普通娃要“学习和习惯推理论证”,培养“逻辑思考辩证思维的习惯和能
: 力”。这个能力不是生而有之的,但能力区间是生而定之。 有些人逻辑思维发育得早
: ,有些人发育得晚;成熟后,有些人逻辑思维强,有些人逻辑思维弱。

i**e
发帖数: 19242
276
娃要自己能想,我就可以撒手乐自己的去了
无法设立难度,都是看孩子的数学作业考试错误而琢磨娃的漏洞在哪里
说实话,孩子大致能拿多少分,你心里没数吗?

【在 t******l 的大作中提到】
: 战略和战术都重要吧。。。而且很多时候为啥做不了某题是归娃想而不是归父母想。。
: 。归父母想的常常是statistically 把难度设在哪里、把要求设多高,而不是拘泥于某
: 一道题。。。对于单一的某一道题,陶天才说了:“做好题,不求甚解,就完事了。。
: 。”

i**e
发帖数: 19242
277
凉拌呗
真这样就从翻饼子做起呗
所不定做成个商人呢
也或许,你微信群里不是疯子就是自己吓自己的呆子 //run

【在 t******l 的大作中提到】
: 但微信上刚才疯传说,“拉分题” 做不出来的话,那将来连 community college 都上
: 不了,只能去麦当劳翻烧饼 。。。那咋办?。。。

i**e
发帖数: 19242
278
要不,你先用你的脚趾头想想,为啥人云你亦云且深信不疑呢?
这玩意儿,不是比4年级以上的数学简单的太多太多吗?

【在 t******l 的大作中提到】
: 另外不要说微信了,本版以前不是一直疯传,不学钢琴输在起跑线上的话,将来数学一
: 定不及格。。。您就说咋办吧。。。

t******l
发帖数: 10908
279
但世上的现实就是,鸡汤传了三圈就会成为真理。。。如果觉得视觉冲击不够,去麦加
旅游一次好了。。。

【在 i**e 的大作中提到】
: 要不,你先用你的脚趾头想想,为啥人云你亦云且深信不疑呢?
: 这玩意儿,不是比4年级以上的数学简单的太多太多吗?

t******l
发帖数: 10908
280
我确实不知道。。。除非我盯紧娃每一道题目,我确实不知道娃能拿多少分。。。而我
又懒的做 control freak / helicopter-dad 每项事无巨细都查看。。。
而且娃几年后的将来的分数更无法预测。。。但预测是做决定最最重要的一环。。。

【在 i**e 的大作中提到】
: 娃要自己能想,我就可以撒手乐自己的去了
: 无法设立难度,都是看孩子的数学作业考试错误而琢磨娃的漏洞在哪里
: 说实话,孩子大致能拿多少分,你心里没数吗?

相关主题
给初中学生家长的一些建议 (转载)BSO: 儿子解射雕瑛姑九宫格
7,8岁的女孩子暑假在家都干什么呢?混BBS要抱着贡献和学习的心
鸡兔同笼其实非常简单老警察问个新问题
进入Parenting版参与讨论
t******l
发帖数: 10908
281
“真这样就从翻饼子做起呗,所不定做成个商人呢”
这是口是心非吧。。。古人云:不是说没良心就能做强盗的。。。这做强盗也是个技术
活。。。

【在 i**e 的大作中提到】
: 凉拌呗
: 真这样就从翻饼子做起呗
: 所不定做成个商人呢
: 也或许,你微信群里不是疯子就是自己吓自己的呆子 //run

i**e
发帖数: 19242
282
我会有个大概的预估
还挺准的,就从孩子的平时的作业就可以看得差不多准:)

【在 t******l 的大作中提到】
: 我确实不知道。。。除非我盯紧娃每一道题目,我确实不知道娃能拿多少分。。。而我
: 又懒的做 control freak / helicopter-dad 每项事无巨细都查看。。。
: 而且娃几年后的将来的分数更无法预测。。。但预测是做决定最最重要的一环。。。

i**e
发帖数: 19242
283
随便怎么想吧
黑刀还觉着我天天开黄腔呢?传久了,或许这个id就成了黄腔帝呢:)
上帝给每个人都赏的有饭碗滴

【在 t******l 的大作中提到】
: “真这样就从翻饼子做起呗,所不定做成个商人呢”
: 这是口是心非吧。。。古人云:不是说没良心就能做强盗的。。。这做强盗也是个技术
: 活。。。

t******l
发帖数: 10908
284
黄腔帝您好。。。

【在 i**e 的大作中提到】
: 随便怎么想吧
: 黑刀还觉着我天天开黄腔呢?传久了,或许这个id就成了黄腔帝呢:)
: 上帝给每个人都赏的有饭碗滴

B********e
发帖数: 10014
285
mm你怎么当真了啊,我开玩笑的
哎,u made me feel so guilty

【在 i**e 的大作中提到】
: 随便怎么想吧
: 黑刀还觉着我天天开黄腔呢?传久了,或许这个id就成了黄腔帝呢:)
: 上帝给每个人都赏的有饭碗滴

c***x
发帖数: 1826
286

“这个能力不是生而有之的,但能力区间是生而定之。”
让我想起有一次问心理学几个老师对nature vs nurture的看法,他们说现在的观点似
乎是:
nature决定在哪个区间,nurture决定在一个区间里的位置。
听后感觉很复杂,很感慨。

【在 s***n 的大作中提到】
: 我觉得数学概念的学习是贯穿小初高的。
: 我觉得对普通娃来说小学初中就要“完成和完善从实体实物到抽象符号的质跃”,“学
: 习和习惯用数学语言来思考和解决数学或者实际问题”,越早越好。像Common core那
: 样细水长流,慢条斯理的教,对很多孩子是不太好的。
: 初中高中阶段普通娃要“学习和习惯推理论证”,培养“逻辑思考辩证思维的习惯和能
: 力”。这个能力不是生而有之的,但能力区间是生而定之。 有些人逻辑思维发育得早
: ,有些人发育得晚;成熟后,有些人逻辑思维强,有些人逻辑思维弱。

i**e
发帖数: 19242
287
果然会4年级以上的数学有个啥用

【在 t******l 的大作中提到】
: 黄腔帝您好。。。
i**e
发帖数: 19242
288
必须的!
才feel guilty 啊:)
还有,这mm还是不要在我这儿用了吧,鸡皮疙瘩啊有木有

【在 B********e 的大作中提到】
: mm你怎么当真了啊,我开玩笑的
: 哎,u made me feel so guilty

i**e
发帖数: 19242
289
复杂吗?
这个不是显而易见的吗?

【在 c***x 的大作中提到】
:
: “这个能力不是生而有之的,但能力区间是生而定之。”
: 让我想起有一次问心理学几个老师对nature vs nurture的看法,他们说现在的观点似
: 乎是:
: nature决定在哪个区间,nurture决定在一个区间里的位置。
: 听后感觉很复杂,很感慨。

B********e
发帖数: 10014
290
水,听你的!

【在 i**e 的大作中提到】
: 必须的!
: 才feel guilty 啊:)
: 还有,这mm还是不要在我这儿用了吧,鸡皮疙瘩啊有木有

相关主题
如何帮助有一定数学天赋的孩子别鸡兔同笼了,来喝啤酒吧
Q. & A. With Liping Ma9岁, 如何报名考amc10
什么时候给孩子引入方程概念求科普: 奥数 vs. Math Olympiad vs. Math Count vs. Math circle vs. 超前学数学
进入Parenting版参与讨论
B********e
发帖数: 10014
291
怎么样,还是叫回mm好点吧?

【在 B********e 的大作中提到】
: 水,听你的!
i**e
发帖数: 19242
292
我在想,如果改成“大妈”,你咋办涅?

【在 B********e 的大作中提到】
: 怎么样,还是叫回mm好点吧?
c***x
发帖数: 1826
293

原来什么在我眼里的复杂事情,到你这儿都简单,果然是智慧!
区间可以有overlap吗?做凤尾好还是鸡头好?

【在 i**e 的大作中提到】
: 复杂吗?
: 这个不是显而易见的吗?

B********e
发帖数: 10014
294
大伯坚决不同意这个

【在 i**e 的大作中提到】
: 我在想,如果改成“大妈”,你咋办涅?
s***n
发帖数: 1280
295
很多事情是说起来容易做起来难,很多道理是看上去简单,运用起来难。你和若水只是
看事情的文角度不同。都对。

【在 c***x 的大作中提到】
:
: 原来什么在我眼里的复杂事情,到你这儿都简单,果然是智慧!
: 区间可以有overlap吗?做凤尾好还是鸡头好?

i**e
发帖数: 19242
296
我的理解
区间是针对一个人说的,不是跟别人比较而来的区间,所以没有凤尾鸡头一说吧
都是命都是天意
看Boy raised as a dog 那本书里的例子就明白了
要改变一个人智商综合素质的baseline
需要极端的外界环境外加一定的持续性
那么对于baseline 不是 outlier的孩子来说
除非生活中有 dramatic events,环境有巨大的变化
或者说nurture的部分是在outlier range里
那么非outlier的孩子也就是在TA自己的baseline附近的区间浮动罢了
父母尽力平常心悦纳就好了
基础线在那儿呢,跨越阶层的毕竟是少数
这人吧,做了鸡头还是凤尾
还真是靠这个人的悟性和综合能力
做父母的,提供可提供的平台,坐看花开花落,不是挺好的吗?:)

【在 c***x 的大作中提到】
:
: 原来什么在我眼里的复杂事情,到你这儿都简单,果然是智慧!
: 区间可以有overlap吗?做凤尾好还是鸡头好?

w**d
发帖数: 2334
297
这个很正常啊,本来天赋就非常关键。

【在 c***x 的大作中提到】
:
: 原来什么在我眼里的复杂事情,到你这儿都简单,果然是智慧!
: 区间可以有overlap吗?做凤尾好还是鸡头好?

w**d
发帖数: 2334
298
可以有两个区间啊。一个衡量'排名',一个衡量多大程度实现了个体的潜能。鸡头凤尾
当然是
对前一个来说的

【在 i**e 的大作中提到】
: 我的理解
: 区间是针对一个人说的,不是跟别人比较而来的区间,所以没有凤尾鸡头一说吧
: 都是命都是天意
: 看Boy raised as a dog 那本书里的例子就明白了
: 要改变一个人智商综合素质的baseline
: 需要极端的外界环境外加一定的持续性
: 那么对于baseline 不是 outlier的孩子来说
: 除非生活中有 dramatic events,环境有巨大的变化
: 或者说nurture的部分是在outlier range里
: 那么非outlier的孩子也就是在TA自己的baseline附近的区间浮动罢了

c***x
发帖数: 1826
299

我其实是一个很拧巴的人。五年前不敢要孩子,因为觉得自己还没有reconcile myself
with this world,完全是因为我家领导是一个那么爱孩子的人,才让我无奈中做出了
事后看,反而是人生中最正确的决定。然后突然间就发现,那些原来让我纠结不已的
predetermined and random factors,原来绝大部分都可以是causal的。
It's not necessarily the right way or the only way, but it is an easy way
out to reconcile myself with this world.
当然,人最终的任务,还要reconcile oneself with himself/herself.
好在,我还年轻嘛。

【在 i**e 的大作中提到】
: 复杂吗?
: 这个不是显而易见的吗?

c***x
发帖数: 1826
300

今天正好读到这个故事,和姐姐分享一下:-)
《在法的门前》
(作者:弗朗茨·卡夫卡)
在法的门前站着一名卫士。一天来了个乡下人,请求卫士放他进法的门里去。可是卫士
回答说,他现在不能允许他这样做。乡下人考虑了一下又问:他等一等是否可以进去呢?
“有可能,”卫士回答,“但现在不成。”
由于法的大门始终都敞开着,这当儿卫士又退到一边去了,乡下人便弯着腰,往门里瞧
。卫士发现了大笑道:“要是你很想进去,就不妨试试,把我的禁止当耳边风好了。不
过得记住:我可是很厉害的。再说我还仅仅是最低一级的卫士哩。从一座厅堂到另一座
厅堂,每一道门前面都站着一个卫士,而且一个比一个厉害。就说第三座厅堂前的那位
吧,连我都不敢正眼瞧他呐。”
乡下人没料到会碰见这么多困难;人家可是说法律之门人人都可以进,随时都可以进啊
,他想。不过,当他现在仔细打量过那位穿皮大衣的卫士,看了看他那又大又尖的鼻子
,又长又密又黑的鞑靼人似的胡须以后,他觉得还是等一等,到人家允许他进去时再进
去好一些。卫士给他一只小矮凳,让他坐在大门旁边。他于是便坐在那儿,日复一日,
年复一年。其间他做过多次尝试,请求人家放他进去,搞得卫士也厌烦起来。时不时地
,卫士也向他提出些简短的询问,问他的家乡和其他许多情况;不过,这些都是那类大
人物提的不关痛痒的问题,临了卫士还是对他讲,他还不能放他进去。乡下人为旅行到
这儿来原本是准备了许多东西的,如今可全都花光了;为了讨好卫士,花再多也该啊。
那位尽管什么都收了,却对他讲:“我收的目的,仅仅是使你别以为自己有什么礼数不
周到。”
许多年来,乡下人差不多一直不停地在观察着这个卫士。他把其他卫士全给忘了;对于
他来说,这第一个卫士似乎就是进入法律殿堂的惟一障碍。他诅咒自己机会碰得不巧,
头一些年还骂得大声大气,毫无顾忌,到后来人老了,就只能再独自嘟嘟囔囔几句。他
甚至变得孩子气起来;在对卫士的多年观察中,他发现这位老兄的大衣毛领里藏着跳蚤
,于是也请跳蚤帮助他使那位卫士改变主意。终于,他老眼昏花了;但自己却闹不清楚
究竟是周围真的变黑了呢,或者仅仅是眼睛在欺骗他。不过,这当儿在黑暗中,他却清
清楚楚看见一道亮光,一道从法律之门中迸射出来的不灭的亮光。此刻他已经生命垂危
。弥留之际,他在这整个过程中的经验一下子全涌进脑海,凝聚成了一个迄今他还不曾
向卫士提过的问题。他向卫士招了招手;他的身体正在慢慢地僵硬,再也站不起来了。
卫士不得不向他俯下身子,他俩的高矮差距已变得对他大大不利。
“事已至此,你还想知道什么?”卫士问。“你这个人真不知足。”
“不是所有的人都向往法律么,”乡下人说,“可怎么在这许多年间,除去我以外就没
见有任何人来要求进去呢?”
卫士看出乡下人已死到临头,为了让他那听力渐渐消失的耳朵能听清楚,便冲他大声吼
道:“这道门任何别的人都不得进入;因为它是专为你设下的。现在我可得去把它关起
来了。”

【在 i**e 的大作中提到】
: 我的理解
: 区间是针对一个人说的,不是跟别人比较而来的区间,所以没有凤尾鸡头一说吧
: 都是命都是天意
: 看Boy raised as a dog 那本书里的例子就明白了
: 要改变一个人智商综合素质的baseline
: 需要极端的外界环境外加一定的持续性
: 那么对于baseline 不是 outlier的孩子来说
: 除非生活中有 dramatic events,环境有巨大的变化
: 或者说nurture的部分是在outlier range里
: 那么非outlier的孩子也就是在TA自己的baseline附近的区间浮动罢了

相关主题
分数应用题系统学习的反面是什么?
代数小朋友不理解怎么办?有些小学四年级的数学题不太会做
教绝对值方程:Khan Academy vs AMC10 答案 vs 分段函数替换?AMC 8 成绩 2016
进入Parenting版参与讨论
c***x
发帖数: 1826
301

不好意思楼主,瞧我把你这个数学楼给歪的。
要包子赔罪请尽管开口 :-)

【在 s**********y 的大作中提到】
: Guess, Check, and Revise 是学好数学的核心
: 去年回国, 找了本地最富盛名的中学老师, 请教代数算数如何衔接。他的回答让我大
: 跌眼睛。猜想, 检验, 和修正。 他说他从不叫公式, 放手让小蛙自己试, 试到那
: 一步算哪一步。 我说这不漫无边际了? 他说自然要加以引导。 秉承一贯风格, 举例
: 如下:
: 鸡兔同笼问题。 鸡兔共20只, 脚64只, 鸡若干/兔若干?
: 鼓励小蛙先猜: 鸡10, 兔10. 脚60. 问题: 先加鸡, 还是先加兔? 答案: 加兔。
: 加几? 不知。 试试? 鸡:9, 兔:11. 脚62. 嘿:再加一就的。
: 鸡兔同笼问题。 鸡兔共20只, 脚70只, 鸡若干/兔若干?
: 还是鼓励小蛙先猜: 鸡10, 兔10. 脚60. 问题: 先加鸡, 还是先加兔? 答案: 加

d**********h
发帖数: 2795
302
有趣的故事

呢?

【在 c***x 的大作中提到】
:
: 不好意思楼主,瞧我把你这个数学楼给歪的。
: 要包子赔罪请尽管开口 :-)

s**********y
发帖数: 509
303
很好的故事啊。
不过有包子的话, 欧不介意收一两只哈。

【在 c***x 的大作中提到】
:
: 不好意思楼主,瞧我把你这个数学楼给歪的。
: 要包子赔罪请尽管开口 :-)

t******l
发帖数: 10908
304
Fable 是三年级以下阅读。。。

:有趣的故事
h*****m
发帖数: 1034
305
Fable算不算一种“构造法”,用来说明枯燥无味的比较抽象的道理的最简洁最生动的
一种方式。
抢滩,air-lifting,烤肉拉扔飞刀。。。算不算“构造法”?
但是 arguably,“构造法” 或者 “reverse math modelling”,是基础数学里
绝大多数定理的最最简洁的证明方式。。。

【在 t******l 的大作中提到】
: Fable 是三年级以下阅读。。。
:
: :有趣的故事
: :

d**********h
发帖数: 2795
306
很适合我
我喜欢简单而有趣的故事,最重要的是我能读得懂 :)

【在 t******l 的大作中提到】
: Fable 是三年级以下阅读。。。
:
: :有趣的故事
: :

t******l
发帖数: 10908
307
众所周知,Guess & Check 是小学四年级内容。。。那末。。。Guess but don't
bother what-so-called check at all 嘛。。。

【在 h*****m 的大作中提到】
: Fable算不算一种“构造法”,用来说明枯燥无味的比较抽象的道理的最简洁最生动的
: 一种方式。
: 抢滩,air-lifting,烤肉拉扔飞刀。。。算不算“构造法”?
: 但是 arguably,“构造法” 或者 “reverse math modelling”,是基础数学里
: 绝大多数定理的最最简洁的证明方式。。。

i**e
发帖数: 19242
308
说明啥?:)
我就看出来,性格决定命运
不同的人会做出不同的决定和行为
过程和结果会不同
不过都是正常的人生罢了:)
至于作者想阐明的哲学意义,你现在来明说一下吧:)
看我能理解不?

呢?

【在 c***x 的大作中提到】
:
: 不好意思楼主,瞧我把你这个数学楼给歪的。
: 要包子赔罪请尽管开口 :-)

t******l
发帖数: 10908
309
哲学意义是:禽兽不如的永远会被惦记,而禽兽的现在正在强奸案法庭上被电击。。。
//run

呢?

【在 i**e 的大作中提到】
: 说明啥?:)
: 我就看出来,性格决定命运
: 不同的人会做出不同的决定和行为
: 过程和结果会不同
: 不过都是正常的人生罢了:)
: 至于作者想阐明的哲学意义,你现在来明说一下吧:)
: 看我能理解不?
:
: 呢?

d**********h
发帖数: 2795
310
哈哈,我还以为潮水要对若水说:对别人或许有其他意义,但是对你而言,就一个字“
命”

【在 t******l 的大作中提到】
: 哲学意义是:禽兽不如的永远会被惦记,而禽兽的现在正在强奸案法庭上被电击。。。
: //run
:
: 呢?

相关主题
AMC 8 成绩 20167,8岁的女孩子暑假在家都干什么呢?
singapore math 到底怎么样啊?鸡兔同笼其实非常简单
给初中学生家长的一些建议 (转载)BSO: 儿子解射雕瑛姑九宫格
进入Parenting版参与讨论
i**e
发帖数: 19242
311
饿死胆儿小的 撑死胆儿肥?
所以还是人各有命
还是 命
哈哈哈哈

【在 d**********h 的大作中提到】
: 哈哈,我还以为潮水要对若水说:对别人或许有其他意义,但是对你而言,就一个字“
: 命”

t******l
发帖数: 10908
312
其实真正的意思是说,密码学很重要,关键是尼米兹破译山本五十六的密码,剩下的都
是体力活。。。如果没破译密码,炮灰化 is just matter of time。。。

【在 i**e 的大作中提到】
: 饿死胆儿小的 撑死胆儿肥?
: 所以还是人各有命
: 还是 命
: 哈哈哈哈

i**e
发帖数: 19242
313
这个鸡头凤尾
以前也总是有人问,有人纠结
这两天我也在想这个问题:)
我准备跳出来看这个问题,从另一个角度看这个问题
来个答非所问:)
如果做凤尾
是指把孩子放到教师质量更好,同学小伙伴们的素质更成熟,校风更正的环境里
去在学业上做垫底的
那么,我看着,如果有机会有能力在这样的环境里做凤尾,是一种幸运
要处理的是帮助孩子调整自己的心态,deal with 垫底的现实
想明白自己为什么来这里垫底,这底值得垫,而且这个垫底并不能预言未来:)
因为我还是相信
贴赤红 傍墨黑 的道理地:)
发包吧,呵呵

【在 c***x 的大作中提到】
:
: 不好意思楼主,瞧我把你这个数学楼给歪的。
: 要包子赔罪请尽管开口 :-)

t******l
发帖数: 10908
314
我觉得你这个说的是“暂时过渡型凤尾”,其实不算真凤尾。。。 你对“永久死菜型
真凤尾”咋看?

【在 i**e 的大作中提到】
: 这个鸡头凤尾
: 以前也总是有人问,有人纠结
: 这两天我也在想这个问题:)
: 我准备跳出来看这个问题,从另一个角度看这个问题
: 来个答非所问:)
: 如果做凤尾
: 是指把孩子放到教师质量更好,同学小伙伴们的素质更成熟,校风更正的环境里
: 去在学业上做垫底的
: 那么,我看着,如果有机会有能力在这样的环境里做凤尾,是一种幸运
: 要处理的是帮助孩子调整自己的心态,deal with 垫底的现实

i**e
发帖数: 19242
315
凤尾鸡头不过是不同的经历
如果有的挑
我选凤尾的环境和经历

【在 t******l 的大作中提到】
: 我觉得你这个说的是“暂时过渡型凤尾”,其实不算真凤尾。。。 你对“永久死菜型
: 真凤尾”咋看?

i**e
发帖数: 19242
316
是否选择跟优秀的人混的问题
跟优秀的人混并不能证明自己的优秀了

【在 t******l 的大作中提到】
: 我觉得你这个说的是“暂时过渡型凤尾”,其实不算真凤尾。。。 你对“永久死菜型
: 真凤尾”咋看?

i**e
发帖数: 19242
317
荷尔蒙带来的好胜好争性啊:)
我还是认为outliers是生出来的,不是努力出来的
当然,这不表示活着就不努力了

【在 t******l 的大作中提到】
: 我觉得你这个说的是“暂时过渡型凤尾”,其实不算真凤尾。。。 你对“永久死菜型
: 真凤尾”咋看?

t******l
发帖数: 10908
318
在现实社会上我不会选凤尾,因为统计上凤尾在现实社会中会活得太累。。。现实社会
不是青春的藤校,你不可以不管老板的看法,但老板的看法主动会来管你。跟藤校的分
数最多不看就是了完全不同。。。而统计上的收益是 10% 的收入最多了,还抵不上医
疗保险的 deductible。。。

【在 i**e 的大作中提到】
: 荷尔蒙带来的好胜好争性啊:)
: 我还是认为outliers是生出来的,不是努力出来的
: 当然,这不表示活着就不努力了

t******l
发帖数: 10908
319
或者现实中观察一下 fish schooling / bird flocking。。。虽然 better schooling
/ flocking 通常能找到更多的 food,因为整个 schooling / flocking 的速度更快
。。。但是在有 predator 存在的现实世界里,better schooling / flocking 中的凤
尾,还没等到吃到 better food 的时候,很大的可能是已经被 predator 吃掉了。。
。当然,这对整个食物链 collective 的发展的贡献是不可磨灭的 -- 摘自《雷锋日记
》。。。

【在 t******l 的大作中提到】
: 在现实社会上我不会选凤尾,因为统计上凤尾在现实社会中会活得太累。。。现实社会
: 不是青春的藤校,你不可以不管老板的看法,但老板的看法主动会来管你。跟藤校的分
: 数最多不看就是了完全不同。。。而统计上的收益是 10% 的收入最多了,还抵不上医
: 疗保险的 deductible。。。

i**e
发帖数: 19242
320
你先定义一下社会上所谓的凤尾和鸡头吧

【在 t******l 的大作中提到】
: 在现实社会上我不会选凤尾,因为统计上凤尾在现实社会中会活得太累。。。现实社会
: 不是青春的藤校,你不可以不管老板的看法,但老板的看法主动会来管你。跟藤校的分
: 数最多不看就是了完全不同。。。而统计上的收益是 10% 的收入最多了,还抵不上医
: 疗保险的 deductible。。。

相关主题
混BBS要抱着贡献和学习的心Q. & A. With Liping Ma
老警察问个新问题什么时候给孩子引入方程概念
如何帮助有一定数学天赋的孩子别鸡兔同笼了,来喝啤酒吧
进入Parenting版参与讨论
i**e
发帖数: 19242
321
承认答非所问的时候,我的思维是局限在大学之前的小初高的教育上的
大学是不是还应该凤尾鸡头
社会这个江湖怎么混?混成凤尾还是鸡头,我还得想想

schooling

【在 t******l 的大作中提到】
: 或者现实中观察一下 fish schooling / bird flocking。。。虽然 better schooling
: / flocking 通常能找到更多的 food,因为整个 schooling / flocking 的速度更快
: 。。。但是在有 predator 存在的现实世界里,better schooling / flocking 中的凤
: 尾,还没等到吃到 better food 的时候,很大的可能是已经被 predator 吃掉了。。
: 。当然,这对整个食物链 collective 的发展的贡献是不可磨灭的 -- 摘自《雷锋日记
: 》。。。

B********e
发帖数: 10014
322
鸡头?你又开黄腔了

【在 i**e 的大作中提到】
: 承认答非所问的时候,我的思维是局限在大学之前的小初高的教育上的
: 大学是不是还应该凤尾鸡头
: 社会这个江湖怎么混?混成凤尾还是鸡头,我还得想想
:
: schooling

t******l
发帖数: 10908
323
这其实是一个类似 “囚徒困境” 的情况,也就是按照 “理性经济人” 的假设,没有
人愿意做 “凤尾” 而被 predator 吃掉,导致 most advanced flocking /
schooling 在 predator 面前,可能变成 the weakest flocking / schooling
。。。
但现实世界是 “理性经济人” 假设本身不成立,比如 “朗朗” 和 “朗朗爸爸” 的
组合。。。所以该 “囚徒困境” 也不容易出现。。。当然可能的潜移默化的后果,
可能是藤校在不完善的 “弼马温主持的 holistic 入学标准下”,慢慢慢慢被
“少体校化”。。。温水煮青蛙。。。不过还有大批州立,其实无所谓。。。
当然,话说回来,藤校联盟最初本来就是体校联盟。。。
如果是在理工领域,“理性经济人” 假设仅存在于从小就有充分信息获取渠道的一群
,否则很难领悟到北岛的所云:
===============================
无心是无心者的通行证
有情是有情者的墓志铭
看吧 在那发考题的天空中
飘满了千老弯曲的倒影
===============================
但是另一方面,一旦 “理性经济人” 成立,那么在稳定福利社会里,没有 reward 机
制的情况下,那 “军医学” 试图打破这个 “理性经济人” 几乎是不可能的。。。原
因就好比 被判半个月 vs 被判两个月 没有任何实质的差别,noise-level 。。。
这篇其实是回答黄小木同学的意图。。。

schooling

【在 i**e 的大作中提到】
: 承认答非所问的时候,我的思维是局限在大学之前的小初高的教育上的
: 大学是不是还应该凤尾鸡头
: 社会这个江湖怎么混?混成凤尾还是鸡头,我还得想想
:
: schooling

d**********h
发帖数: 2795
324
这个问题我不懂,但是留学生到美国总得还是不错,没到了被吃得渣都不剩的程度

【在 t******l 的大作中提到】
: 这其实是一个类似 “囚徒困境” 的情况,也就是按照 “理性经济人” 的假设,没有
: 人愿意做 “凤尾” 而被 predator 吃掉,导致 most advanced flocking /
: schooling 在 predator 面前,可能变成 the weakest flocking / schooling
: 。。。
: 但现实世界是 “理性经济人” 假设本身不成立,比如 “朗朗” 和 “朗朗爸爸” 的
: 组合。。。所以该 “囚徒困境” 也不容易出现。。。当然可能的潜移默化的后果,
: 可能是藤校在不完善的 “弼马温主持的 holistic 入学标准下”,慢慢慢慢被
: “少体校化”。。。温水煮青蛙。。。不过还有大批州立,其实无所谓。。。
: 当然,话说回来,藤校联盟最初本来就是体校联盟。。。
: 如果是在理工领域,“理性经济人” 假设仅存在于从小就有充分信息获取渠道的一群

t******l
发帖数: 10908
325
大部分留学生都是为出国降了一点学校的标准的。。。君不见,北清的多少不
在哈佛耶鲁里,换个专业也无所谓。。。而死撑学校/专业/教职的,反而被
千老化的概率要高很多不是?。。。所以这个正是 “不做凤尾” 之 “囚徒困境”
模型的真实写照。

【在 d**********h 的大作中提到】
: 这个问题我不懂,但是留学生到美国总得还是不错,没到了被吃得渣都不剩的程度
t******l
发帖数: 10908
326
比如。。。哈佛高尚但坑爹专业无法毕业的博后?。。。// run

【在 i**e 的大作中提到】
: 你先定义一下社会上所谓的凤尾和鸡头吧
i**e
发帖数: 19242
327
等小木看懂之后给我解释吧

【在 t******l 的大作中提到】
: 这其实是一个类似 “囚徒困境” 的情况,也就是按照 “理性经济人” 的假设,没有
: 人愿意做 “凤尾” 而被 predator 吃掉,导致 most advanced flocking /
: schooling 在 predator 面前,可能变成 the weakest flocking / schooling
: 。。。
: 但现实世界是 “理性经济人” 假设本身不成立,比如 “朗朗” 和 “朗朗爸爸” 的
: 组合。。。所以该 “囚徒困境” 也不容易出现。。。当然可能的潜移默化的后果,
: 可能是藤校在不完善的 “弼马温主持的 holistic 入学标准下”,慢慢慢慢被
: “少体校化”。。。温水煮青蛙。。。不过还有大批州立,其实无所谓。。。
: 当然,话说回来,藤校联盟最初本来就是体校联盟。。。
: 如果是在理工领域,“理性经济人” 假设仅存在于从小就有充分信息获取渠道的一群

i**e
发帖数: 19242
328
你呀...

【在 B********e 的大作中提到】
: 鸡头?你又开黄腔了
i**e
发帖数: 19242
329
第一代留学移民,有特殊性,尤其是60,70甚至一些80后
我们的孩子跟这一批人,不好比较的吧?

【在 t******l 的大作中提到】
: 比如。。。哈佛高尚但坑爹专业无法毕业的博后?。。。// run
t******l
发帖数: 10908
330
但是古人云:“英雄莫问出处,千老不看出身”。。。

【在 i**e 的大作中提到】
: 第一代留学移民,有特殊性,尤其是60,70甚至一些80后
: 我们的孩子跟这一批人,不好比较的吧?

相关主题
9岁, 如何报名考amc10代数小朋友不理解怎么办?
求科普: 奥数 vs. Math Olympiad vs. Math Count vs. Math circle vs. 超前学数学教绝对值方程:Khan Academy vs AMC10 答案 vs 分段函数替换?
分数应用题系统学习的反面是什么?
进入Parenting版参与讨论
t******l
发帖数: 10908
331
其实我觉得黄小木还是蛮有理想和热情的。。。我只是觉得伊一个人解决不了 “无心
是无心者的通行证/有情是有情者的墓志铭/看吧,在那发考题的天空中/飘满了千老
弯曲的倒影” 这个 “囚徒困境” 的悖论难题。。。

【在 i**e 的大作中提到】
: 等小木看懂之后给我解释吧
d**********h
发帖数: 2795
332
真心没觉得
降准是,但是没有炮灰

【在 t******l 的大作中提到】
: 大部分留学生都是为出国降了一点学校的标准的。。。君不见,北清的多少不
: 在哈佛耶鲁里,换个专业也无所谓。。。而死撑学校/专业/教职的,反而被
: 千老化的概率要高很多不是?。。。所以这个正是 “不做凤尾” 之 “囚徒困境”
: 模型的真实写照。

t******l
发帖数: 10908
333
或者我这么简单解释一下这个类似 “多人囚徒困境” (multi-player prison's
dilemma)。
首先是定义 cooperate vs defect。。。用藤校入学系统做一个比方,比如理想的藤校
入学系统是择优录取,然后按情况分配专业。。。那如果以理想正确健康的“藤校为本
、以藤校为纲”,这就是 cooperate(cooperate 这个 selection 系统)。。。但如
果不按照藤校希望出牌,那就是 defect(defect 这个 selection 系统)。。。
但 defect 里面还细分为三种:
(1)新东方路子型:合法钻系统漏洞,削尖脑袋进藤校。
(2)聚众支持 race-based AA 游行型:合法通过歧视性法律或者歧视性入学规则。
(3)不去藤校坑爹专业为人类前进做炮灰,去州立大学好专业混个一生开心。这也是
合法钻系统漏洞,削尖脑袋避免藤校坑爹专业、或者避免不幸成为炮灰型凤尾还要乖乖
交高额学费然后去达特茅斯地下室。。。
然后三种 defect 和一种 cooperate 发生了教育军医学里最大的 “多人囚徒困境”
之互殴群殴。。。

【在 i**e 的大作中提到】
: 等小木看懂之后给我解释吧
t******l
发帖数: 10908
334
降准去当鸡头的显然不是炮灰。。。炮灰是不降准的凤尾千老不是?。。。(我本来不
想直说的,您这么热情我也没办法)。。。

【在 d**********h 的大作中提到】
: 真心没觉得
: 降准是,但是没有炮灰

d**********h
发帖数: 2795
335
凤尾钱老显然也不是炮灰,可能降准过猛,过低估计自己价值也是有的。远远不到被捕
食者吞噬吃掉的凄惨境地

【在 t******l 的大作中提到】
: 降准去当鸡头的显然不是炮灰。。。炮灰是不降准的凤尾千老不是?。。。(我本来不
: 想直说的,您这么热情我也没办法)。。。

t******l
发帖数: 10908
336
我就问你 metal health insurance 的 deductible 是多少吧。。。当然 izze 会说不
幸被狼看上了那都是命。。。这个我也没办法。。。

【在 d**********h 的大作中提到】
: 凤尾钱老显然也不是炮灰,可能降准过猛,过低估计自己价值也是有的。远远不到被捕
: 食者吞噬吃掉的凄惨境地

t******l
发帖数: 10908
337
或者更新鲜生猛的例子,University of Missouri 申请人数下降,就是一个现实
的教育军医界的 “多人囚徒困境” 的现实例子。。。有不服的么?。。。
http://m.abc17news.com/news/mizzou-fall-enrollment-down-since-l
而俺跟黄小木同学的区别是,黄小木同学号召大伙儿 cooperate 这个系统。。。而我
认为大伙儿应该更多的从 “理想经济人” 角度做出选择(合法投票之类的都是属于“
理想经济人” 角度,投个票又不费啥事儿)。。。因为我的看法是,真实世界里,你
永远无法避免 8 billion 猴子去 defect / contaminate 这个系统。。。所以只有以
“理想经济人”的角度(当然蝇头小利也不算啥 “理想经济人” 角度),在需要的时
候,合法地去 defect 这个系统,这不仅为着大伙儿各家切身利益,同时也是迫使系统
自我改进(否则系统就等着自我灭亡吧哈哈哈哈哈哈哈)。。。大伙儿不合法合理
defect 系统的话,招生办的弼马温们一高兴就把大伙儿全卖了还等着大伙儿给数钱。
。。话糙理不糙!!。。。

【在 t******l 的大作中提到】
: 或者我这么简单解释一下这个类似 “多人囚徒困境” (multi-player prison's
: dilemma)。
: 首先是定义 cooperate vs defect。。。用藤校入学系统做一个比方,比如理想的藤校
: 入学系统是择优录取,然后按情况分配专业。。。那如果以理想正确健康的“藤校为本
: 、以藤校为纲”,这就是 cooperate(cooperate 这个 selection 系统)。。。但如
: 果不按照藤校希望出牌,那就是 defect(defect 这个 selection 系统)。。。
: 但 defect 里面还细分为三种:
: (1)新东方路子型:合法钻系统漏洞,削尖脑袋进藤校。
: (2)聚众支持 race-based AA 游行型:合法通过歧视性法律或者歧视性入学规则。
: (3)不去藤校坑爹专业为人类前进做炮灰,去州立大学好专业混个一生开心。这也是

i**e
发帖数: 19242
338
1,2,3看懂了
高大上地讨论教育制度及其改革,我不能
爬藤没想过,但对上不上得了藤的大致看法是
真到了那个档次上不了大藤也不是什么大事儿,本事又不会因为没上大藤而消失
这个不是虎落平阳有本事施展不出来了
非藤娃,比较务实的想法是
了解孩子对什么感兴趣在什么方面有优势
培养和发挥这些优势帮助孩子能够自食其力

【在 t******l 的大作中提到】
: 或者我这么简单解释一下这个类似 “多人囚徒困境” (multi-player prison's
: dilemma)。
: 首先是定义 cooperate vs defect。。。用藤校入学系统做一个比方,比如理想的藤校
: 入学系统是择优录取,然后按情况分配专业。。。那如果以理想正确健康的“藤校为本
: 、以藤校为纲”,这就是 cooperate(cooperate 这个 selection 系统)。。。但如
: 果不按照藤校希望出牌,那就是 defect(defect 这个 selection 系统)。。。
: 但 defect 里面还细分为三种:
: (1)新东方路子型:合法钻系统漏洞,削尖脑袋进藤校。
: (2)聚众支持 race-based AA 游行型:合法通过歧视性法律或者歧视性入学规则。
: (3)不去藤校坑爹专业为人类前进做炮灰,去州立大学好专业混个一生开心。这也是

d**********h
发帖数: 2795
339
哪个地方不需要pay deductible?
在国内做鸡头不需要吗?
投名状总是要的,可能付出的多了点,但是得到的绝对收获未必就少
你是选择在大山里做个生产队长,还是北上广做个打工仔,或者美帝刷盘子的?

【在 t******l 的大作中提到】
: 我就问你 metal health insurance 的 deductible 是多少吧。。。当然 izze 会说不
: 幸被狼看上了那都是命。。。这个我也没办法。。。

t******l
发帖数: 10908
340
属实

:1,2,3看懂了
:高大上地讨论教育制度及其改革,我不能
相关主题
系统学习的反面是什么?singapore math 到底怎么样啊?
有些小学四年级的数学题不太会做给初中学生家长的一些建议 (转载)
AMC 8 成绩 20167,8岁的女孩子暑假在家都干什么呢?
进入Parenting版参与讨论
t******l
发帖数: 10908
341
你说的那些都是基于 “理想经济人” 的假设。。。我不知道是不是要感谢你送炮弹。
。。


哪个地方不需要pay deductible?
:在国内做鸡头不需要吗?
d**********h
发帖数: 2795
342
那只能说咱俩划分可能不一致,而且互相交叉
潮水兄的理论老难懂呢

哪个地方不需要pay deductible?

【在 t******l 的大作中提到】
: 你说的那些都是基于 “理想经济人” 的假设。。。我不知道是不是要感谢你送炮弹。
: 。。
:
: :
: 哪个地方不需要pay deductible?
: :在国内做鸡头不需要吗?

t******l
发帖数: 10908
343
无他,我只是觉得网上太不中庸之道了。。。而中庸的观点反而被评论为偏激。。。
当然真实世界里其实不是这样。。。

:那只能说咱俩划分可能不一致,而且互相交叉
t******l
发帖数: 10908
344
我不过是说些中庸的想法
用一点激烈的言辞
和一种拧歪的方式

:那只能说咱俩划分可能不一致,而且互相交叉
d**********h
发帖数: 2795
345
明白明白
以拧巴证正道
以雷霆行慈悲
咱版有这么不上道吗,逼得潮水兄下此猛药
:)

【在 t******l 的大作中提到】
: 我不过是说些中庸的想法
: 用一点激烈的言辞
: 和一种拧歪的方式
:
: :那只能说咱俩划分可能不一致,而且互相交叉
: :

t******l
发帖数: 10908
346
其实也怪不得本版。。。主要是 “军医学” 的模型过于简单而原始,立场过于左派而
激进,导致结论过于激烈而拧歪。。。比如一个例子就是不当凤尾就是鸡头。。。现实
世界里哈佛的凤尾去 UC Berkeley 能当鸡头?我觉得去 UC Davis 都不一定能当鸡头
。。。

【在 d**********h 的大作中提到】
: 明白明白
: 以拧巴证正道
: 以雷霆行慈悲
: 咱版有这么不上道吗,逼得潮水兄下此猛药
: :)

1 (共1页)
进入Parenting版参与讨论
相关主题
什么时候给孩子引入方程概念有些小学四年级的数学题不太会做
别鸡兔同笼了,来喝啤酒吧AMC 8 成绩 2016
9岁, 如何报名考amc10singapore math 到底怎么样啊?
求科普: 奥数 vs. Math Olympiad vs. Math Count vs. Math circle vs. 超前学数学给初中学生家长的一些建议 (转载)
分数应用题7,8岁的女孩子暑假在家都干什么呢?
代数小朋友不理解怎么办?鸡兔同笼其实非常简单
教绝对值方程:Khan Academy vs AMC10 答案 vs 分段函数替换?BSO: 儿子解射雕瑛姑九宫格
系统学习的反面是什么?混BBS要抱着贡献和学习的心
相关话题的讨论汇总
话题: 数学话题: 鸡兔同笼话题: amc话题: 孩子话题: 代数